Download as pdf or txt
Download as pdf or txt
You are on page 1of 192

Polity 2

1. Consider the following statements with respect to the legislative power of the Indian Parliament:
1. The laws made by the Parliament are applicable to properties of Indian citizen in any part of the
world.
2. State legislatures can make laws with respect to any of the matters mentioned in the concurrent
list except criminal law and procedure.
3. In case of a conflict between central and state legislation with respect to laws made on a
concurrent list entry, the central law prevails.
Which of the statements given above are correct?
(a) 1 and 2 only
(b) 1 and 3 only
(c) 2 and 3 only
(d) 1, 2 and 3

Answer: B
Explanation:
Statement 1 is correct: The Parliament can make laws for the whole or any part of the territory of India.
The territory of India includes the states, the union territories, and any other area for the time being
included in the territory of India. The Parliament alone can make ‘extraterritorial legislation’. Thus, the
laws of the Parliament are also applicable to the Indian citizens and their property in any part of the
world.
Statement 2 is incorrect: Both the Parliament and state legislature can make laws with respect to any
of the matters enumerated in the Concurrent List. This list has at present 52 subjects (originally 47
subjects) like criminal law and procedure, civil procedure, marriage and divorce, population control and
family planning, electricity, labour welfare, economic and social planning, drugs, newspapers, books and
printing press, and others.
Statement 3 is correct: The Constitution under Schedule VII sets out the various subjects on which the
Parliament and State may legislate, under List I and List II respectively. Under List III, also known as the
Concurrent List, both the Parliament and the states have the power to make laws.
It is under Article 254 that the Constitution provides that in case both the Parliament and the state
make a law upon a matter in the Concurrent List and the laws are such that they are irreconcilable, then
the law made by the Parliament shall prevail and the law made by the state shall be deemed to be
repugnant to the extent of its repugnancy with the Central law.
It is due to this Article that the power of the Parliament to legislate upon matters contained in List III i.e.,
the Concurrent List is supreme. The Article gives an overriding effect to any statue which the Parliament
is competent to enact and which has been enacted by it.
Sources:
Indian Polity by M. Laxmikanth, 6th Edition - Chapter- 14.
https://www.legalserviceindia.com/legal/article-964-the-doctrine-of-repugnancy-in-the-indian-
constitution.html

2. Consider the following statements:


1. Both the Parliament and state legislatures can make laws with respect to the residuary subjects.
2. Under the Government of India Act of 1935, the federal legislature was provided with exclusive power
to make laws on the residuary subjects.
3. Under no circumstances can state laws prevail over central laws in the case of a conflict between
central and state legislation with respect to laws made on a Concurrent List matter.
Which of the statements given above is/are not correct?
(a) 1 and 3 only
(b) 2 only
(c) 3 only
(d) 1, 2, and 3

Answer: D
Explanation:
Statement 1 is incorrect: The matters of national importance and the matters which require uniformity
of legislation nationwide are included in the Union List. The matters of regional and local importance and
the matters which permit diversity of interest are specified in the State List. The matters on which
uniformity of legislation throughout the country is desirable but not essential are enumerated in the
concurrent list.
According to Article 248 and Entry 97 of List I, the power to make laws with respect to residuary subjects
(i.e., the matters which are not enumerated in any of the three lists) is exclusively vested in the
Parliament. This residuary power of legislation includes the power to levy residuary taxes.
Statement 2 is incorrect: In US, only the powers of the Federal Government are enumerated in the
Constitution and the residuary powers are left to the states. In Canada, on the other hand, there is a
double enumeration– Federal and Provincial, and the residuary powers are vested in the Centre. The
Government of India Act of 1935 provided for a three-fold enumeration, viz., federal, provincial and
concurrent. The present Constitution follows the scheme of this Act but with one difference, that is, under
this Act, the residuary powers were given neither to the federal legislature nor to the provincial
legislature but to the governor general of India. In this respect, India follows the Canadian precedent.
Statement 3 is incorrect: In case of a conflict between the Central law and a State law on a subject
enumerated in the Concurrent List, the Central law prevails over the state law. But, there is an
exception. If the state law has been reserved for the consideration of the President and has received
his assent, then the state law prevails in that state. But, it would still be competent for the Parliament to
override such a law by subsequently making a law on the same matter.

Source: Indian Polity by M. Laxmikanth, 6th Edition- Chapter- 14.

3. Consider the following statements:


1. The president of India can summon a session of parliament at such a place as he/she thinks fit.
2. The constitution of India provides for three sessions of the parliament in a year but it is not
mandatory to conduct all three sessions
3. There is no minimum number of days that the parliament is required to meet in a year
Which of the above-given statements is/are correct?
(a) 1 only
(b) 2 only
(c) 1 and 3 only
(d) 2 and 3 only

Answer: C
Explanation:
Statement 1 is correct: Article 85 says the President can summon a session of Parliament “at such time
and place as he thinks fit”. A session can be called on the recommendation of the government, which
decides its date and duration. But, the maximum gap between two sessions of Parliament cannot be more
than six months which indicates the Parliament should meet at least twice a year.
Statement 2 is incorrect: There are usually three sessions in a year, by convention and not by the
Constitution, Parliament meets for three sessions in a year:
 the Budget Session (February to May);
 the Monsoon Session (July to August); and
 the Winter Session (November to December).
Statement 3 is correct: The Constitution does not specify when or for how many days Parliament should
meet.
Article 85 only requires that there should not be a gap of more than six months between two sessions of
Parliament. During a session, the House meets every day to transact business. The period spanning
between the prorogation of a House and its reassembly in a new session is called ‘recess’.

Source: UPSC CSE PYQ 2020

4. The Government enacted the Panchayat Extension to Scheduled Areas (PESA) Act in 1996. Which one
of the following is not identified as its objective?
(a) To provide self-governance
(b) To recognize traditional rights
(c) To create autonomous regions in tribal areas
(d) To free tribal people from exploitation

Answer: C
Explanation:
Option (c) is correct: The PESA Act was enacted in 1996 “to provide for the extension of the provisions of
Part IX of the Constitution relating to the Panchayats to the Scheduled Areas”.
Under the PESA Act, Scheduled Areas are those referred to in Article 244(1), which says that the provisions
of the Fifth Schedule shall apply to the Scheduled Areas and Scheduled Tribes in states other than Assam,
Meghalaya, Tripura, and Mizoram. The Fifth Schedule provides for a range of special provisions for these
areas. The main rationale behind the Act is to preserve the tribal population from exploitation with the
active involvement of the Gram Sabha. Some other objectives of the PESA Act are:
 To provide self-governance for the bulk of the tribal population
 To have village governance with participatory democracy and to make the gram sabha a nucleus
of all activities
 To recognize traditional rights and safeguard and preserve the traditions and customs of tribal
communities.
 To empower panchayats at the appropriate levels with specific powers conducive to tribal
requirements
Creating autonomous regions in tribal areas is not an objective of the PESA Act.

Source: UPSC CSE PYQ 2013

5. Consider the following pairs:


Article of Constitution Provision
1. Article 249 The Parliament can make laws on State List subjects in national interest
if the Rajya Sabha passes a resolution by 2/3rd majority of the of the
members present and voting.
2. Article 250 If requested by two or more states, the Parliament can make laws on
the State List with respect to those states only.
3. Article 252 The Parliament can legislate with respect to any matter in the State List
if a Proclamation of Emergency is in operation.
4. Article 253 The Parliament can make laws on any matter in the State List for
implementing the international treaties.
How many pairs given above is/are correctly matched?
(a) Only one pair
(b) Only two pairs
(c) Only three pairs
(d) All four pairs

Answer: B
Explanation:
Pair 1 is correctly matched: Article 249 states that if the Rajya Sabha declares that it is necessary in the
national interest that Parliament should make laws with respect to goods and services tax or a matter
in the State List, then the Parliament becomes competent to make laws on that matter. Such a resolution
must be supported by two-thirds (2/3rd) of the members present and voting. The resolution remains in
force for one year; it can be renewed any number of times but not exceeding one year at a time. The laws
cease to have effect on the expiration of six months after the resolution has ceased to be in force. This
provision does not restrict the power of a state legislature to make laws on the same matter. But, in case
of inconsistency between a state law and a parliamentary law, the latter is to prevail.
Pair 2 is incorrectly matched: Article 250 states that the Parliament acquires the power to legislate with
respect to goods and services tax or matters in the State List, while a proclamation of national
emergency is in operation. The laws become inoperative on the expiration of six months after the
emergency has ceased to operate. Here also, the power of a state legislature to make laws on the same
matter is not restricted. But, in case of repugnancy between a state law and a parliamentary law, the latter
is to prevail.
Pair 3 is incorrectly matched: Article 252 states that when the legislatures of two or more states pass
resolutions requesting the Parliament to enact laws on a matter in the State List, then the Parliament
can make laws for regulating that matter. A law so enacted applies only to those states which have passed
the resolutions. However, any other state may adopt it afterwards by passing a resolution to that effect
in its legislature. Such a law can be amended or repealed only by the Parliament and not by the
legislatures of the concerned states. The effect of passing a resolution under the above provision is that
the Parliament becomes entitled to legislate with respect to a matter for which it has no power to make
a law. On the other hand, the state legislature ceases to have the power to make a law with respect to
that matter. The resolution operates as abdication or surrender of the power of the state legislature
with respect to that matter and it is placed entirely in the hands of Parliament which alone can then
legislate with respect to it. Some examples of laws passed under the above provision are Prize
Competition Act, 1955; Wild Life (Protection) Act, 1972; Water (Prevention and Control of Pollution) Act,
1974; Urban Land (Ceiling and Regulation) Act, 1976; and Transplantation of Human Organs Act, 1994.
Pair 4 is correctly matched: Article 253 states that the Parliament can make laws on any matter in the
State List for implementing the international treaties, agreements or conventions. This provision
enables the Central government to fulfil its international obligations and commitments. Some examples
of laws enacted under the above provision are United Nations (Privileges and Immunities) Act, 1947;
Geneva Convention Act, 1960; Anti-Hijacking Act, 1982 and legislations relating to environment and TRIPS.
Source: Indian Polity by M. Laxmikanth, 6th Edition- Chapter- 14.

6. Consider the following statements:


1. The Center can give directions to the states with regard to the maintenance of those means of
communication that are declared to be of national or military importance.
2. Article 365 acts as a screen to prevent any hasty resort to the drastic action under Article 356.
Which of the statements given above is/are correct?
(a) 1 only
(b) 2 only
(c) Both 1 and 2
(d) Neither 1 nor 2

Answer: C
Explanation:
Statement 1 is correct: Under Article 256, the Centre is empowered to give directions to the states with
regard to the exercise of their executive power in the following matters:
• the construction and maintenance of means of communication (declared to be of national or
military importance) by the state;
• the measures to be taken for the protection of the railways within the state;
• the provision of adequate facilities for instruction in the mother tongue at the primary stage of
education to children belonging to linguistic minority groups in the state; and
• the drawing up and execution of the specified schemes for the welfare of the Scheduled Tribes in
the state.
Statement 2 is correct: Dr. B.R. Ambedkar in the Constituent assembly had supported the incorporation
of Article 365 as it conferred power to the Union to take action when there was failure to carry out
directions under Articles 256 & 257. The Council observed that the remedy provided in Article 365 is less
harsh than that in Article 356. Retention of Article 365 is necessary as it acts as a screen to prevent any
hasty resort to drastic action under Article 356. It was decided to retain Article 365 and to accept
Commission’s recommendation for a cautious approach before its application.
• Article 356 empowers the President to issue a proclamation, if he is satisfied that a situation has arisen
in which the government of a state cannot be carried on in accordance with the provisions of the
Constitution. The president can act either on a report of the governor of the state or even without the
governor’s report.
• Article 365 says that whenever a state fails to comply with or to give effect to any direction from
the Centre, it will be lawful for the president to hold that a situation has arisen in which the
government of the state cannot be carried on in accordance with the provisions of the Constitution.

Sources:
Indian Polity by M. Laxmikanth, 6th Edition- Chapter- 14, 16.
https://archive.pib.gov.in/archive/releases98/lyr2003/raug2003/28082003/r280820037.html

7. . Consider the following urban local bodies:


1. Municipal corporation
2. Cantonment board
3. Township
4. Municipal council
5. Notified area committee
Which of the above have been provided to be set up by the 74th Constitutional Amendment Act, 1993?
(a) 1 and 4 only
(b) 1, 4 and 5 only
(c) 2, 3 and 5 only
(d) 1, 2, 3, 4 and 5

Answer: A
Explanation:
The 74th Constitutional Amendment Act provided for the constitution of the following three types of
municipalities in every state.
● A Nagar Panchayat (by whatever name called) for a transitional area.
● A municipal council for a smaller urban area.
● A municipal corporation for a larger urban area.
Statement 1 is correct: Municipal corporations are created for the administration of big cities like Delhi,
Mumbai, Kolkata, Hyderabad, Bangalore and others. They are established in the states by the acts of the
concerned state legislatures, and in the union territories by the acts of the Parliament of India. There
may be one common Act for all the municipal corporations in a state or a separate Act for each municipal
corporation.
Statement 2 is incorrect: A cantonment board is established for municipal administration for the civilian
population in the cantonment area7. It is set up under the provisions of the Cantonments Act of 2006 –
a legislation enacted by the Central government. It works under the administrative control of the Defence
ministry of the Central government. Unlike the other urban local bodies, a cantonment board is created
as well as administered by the Central government.
Statement 3 is incorrect: A Township is a type of urban government established by the large public
enterprises to provide civic amenities to its staff and workers who live in the housing colonies built near
the plant. The enterprise appoints a town administrator to look after the administration of the township.
He is assisted by some engineers and other technical and non-technical staff. Thus, the township form of
urban government has no elected members. In fact, it is an extension of the bureaucratic structure of the
enterprises.
Statement 4 is correct: The municipalities are established for the administration of towns and smaller
cities. Like the corporations, they are also set up in the states by the acts of the concerned state
legislatures and in the union territory by the acts of the Parliament of India. They are also known by
various other names like municipal council, municipal committee, municipal board, borough municipality,
city municipality and others.
Statement 5 is incorrect: A notified area committee is created for the administration of two types of
areas – a fast developing town due to industrialisation, and a town which does not yet fulfill all the
conditions necessary for the constitution of a municipality, but which otherwise is considered important
by the state government. Since it is established by a notification in the government gazette, it is called
a notified area committee.

Source: Indian Polity by M Laxmikanth 6th edition, Chapter 39

8. Which of the following conditions must be fulfilled by a political party for it to be recognised as a
national party?
1. It should secure six percent of valid votes polled in any four or more states at a general election
to the Lok Sabha or to the legislative assembly
2. It should win two percent of seats in the Lok Sabha at a general election and these candidates are
elected from three states
3. It should be recognised as a state party in four states.
Select the correct answer using the code given below:
(a) 1 and 2 only
(b) 2 and 3 only
(c) 1 and 3 only
(d) 1, 2 and 3

Answer: B
Explanation:
Option (b) is correct: The Election Commission registers political parties for the purpose of elections and
grants them recognition as national or state parties on the basis of their poll performance.
A party is recognised as a national party if any of the following conditions is fulfilled:
● If it secures six percent of valid votes polled in any four or more states at a general election to
the Lok Sabha or to the legislative assembly; and, in addition, it wins four seats in the Lok Sabha
from any state or states; or (Both the conditions must be satisfied at the same time) (Statement
1 is incorrect)
● If it wins two percent of seats in the Lok Sabha at a general election; and these candidates are
elected from three states (Statement 2 is correct); or
● If it is recognised as a state party in four states (Statement 3 is correct).

Source: Indian Polity by M Laxmikanth 6th edition, Chapter 69

9. With reference to judicial review of proclamation of President’s Rule, consider the following
statements:
1. The 38th Constitutional Amendment Act of 1975 brought the satisfaction of the President under
the purview of judicial review.
2. The court cannot go into the correctness of the material but it can see whether it is relevant to
the action.
3. Even if the court holds the presidential proclamation to be unconstitutional and invalid, it does
not have the power to revive the state legislative assembly if it was dissolved.
Which of the statements given above is/are not correct?
(a) 1 and 3 only
(b) 1 and 2 only
(c) 2 only
(d) 1, 2 and 3

Answer: A
Explanation:
Statement 1 is incorrect: The 38th Constitutional Amendment Act of 1975 made the satisfaction of the
President in invoking Article 356 final and conclusive which could not be challenged in any court on any
ground. But this provision was subsequently deleted by the 44th Amendment Act of 1978 implying that
the satisfaction of the President is not beyond judicial review.
Statement 2 is correct but Statement 3 is incorrect: In S.R. Bommai case (1994), the following
propositions have been laid down by the Supreme Court on imposition of President’s Rule in a state under
Article 356:
1. The presidential proclamation imposing President’s Rule is subject to judicial review.
2. The satisfaction of the President must be based on relevant material. The action of the President
can be struck down by the court if it is based on irrelevant or extraneous grounds or if it was found
to be malafide or perverse.
3. Burden lies on the Centre to prove that relevant material exists to justify the imposition of the
President’s Rule.
4. The court cannot go into the correctness of the material or its adequacy but it can see whether
it is relevant to the action.
5. If the court holds the presidential proclamation to be unconstitutional and invalid, it has power
to restore the dismissed state government and revive the state legislative assembly if it was
suspended or dissolved.
6. The state legislative assembly should be dissolved only after the Parliament has approved the
presidential proclamation. Until such approval is given, the President can only suspend the
assembly. In case the Parliament fails to approve the proclamation, the assembly would get
reactivated.
7. Secularism is one of the ‘basic features’ of the Constitution. Hence, a state government pursuing
anti-secular politics is liable to action under Article 356.
8. The question of the state government losing the confidence of the legislative assembly should be
decided on the floor of the House and until that is done the ministry should not be unseated.
9. Where a new political party assumes power at the Centre, it will not have the authority to dismiss
ministries formed by other parties in the states.
10. The power under Article 356 is an exceptional power and should be used only occasionally to
meet the requirements of special situations.

Source: Indian Polity by M Laxmikanth 6th edition, Chapter-16

10. Consider the following statements with respect to the Union Executive in India:
1. The individual ministers apart from the Prime Minister hold office during the pleasure of the Prime
Minister.
2. The Council of Ministers shall be collectively responsible to the Parliament.
Which of the statements given above is/are not correct?
(a) 1 only
(b) 2 only
(c) Both 1 and 2
(d) Neither 1 nor 2

Answer: C
Explanation:
Statement 1 is incorrect: Article 75(2) contains the principle of individual responsibility. It states that the
council of ministers hold office during the pleasure of the President, which means that the President can
remove a minister even at a time when the council of ministers enjoys the confidence of the Lok Sabha.
However, the President removes a minister only on the advice of the Prime Minister. In case of a
difference of opinion or dissatisfaction with the performance of a minister, the Prime Minister can ask
him to resign or advise the President to dismiss him. The Council of Ministers (COM) consists of three
categories of ministers, namely cabinet ministers, ministers of state and deputy ministers. The Prime
Minister is at the top of all these ministers and is part of COM.
Statement 2 is incorrect: Article 75(3) is the bedrock foundation of the parliamentary system of
government and provides that the Council of Ministers shall be collectively responsible to the Lok Sabha
(not Parliament). They act as a team, and swim and sink together. The principle of collective responsibility
implies that the Lok Sabha can remove the ministry (i.e., the Council of Ministers headed by the Prime
Minister) from office by passing a vote of no confidence.

Source: Indian Polity by M Laxmikanth 6th edition, Chapter-17, 20

11. The emergency provisions of the Indian Constitution have been materially borrowed from
(a) Government of India Act, 1935
(b) Weimar Constitution of Germany
(c) South African Constitution
(d) British Constitution

Answer: A
Explanation:
Option (a) is correct: The Constitution of India has borrowed most of its provisions from the constitutions
of various other countries as well as from the Government of India Act of 1935. In fact, Dr. B.R. Ambedkar
proudly acclaimed that the Constitution of India has been framed after ‘ransacking all the known
Constitutions of the World’. In fact, the most profound influence and material source of the Constitution
is the Government of India Act, 1935. The Federal Scheme, Judiciary, Governors, Emergency provisions
and administrative details, the Public Service Commissions and most of the administrative details are
drawn from this Act. More than half of the provisions of the Constitution are identical to or bear a close
resemblance to the Act of 1935.
Not to be confused with the source of ‘suspension of Fundamental Rights’ during an Emergency which is
taken from the Weimar Constitution of Germany.

Source:
Indian Polity by M Laxmikanth 6th edition, Chapter-3

12. Consider the following statements with reference to Article 352 of the Indian Constitution:
1. It defines the term ‘National Emergency’ as an emergency due to war, external aggression or
armed rebellion.
2. The President can declare a national emergency even before the actual occurrence of war or
external aggression or armed rebellion.
3. The 44th Constitutional Amendment enabled the President to limit the operation of a national
emergency to a specific part of India.
4. The President can proclaim a national emergency only after receiving a written recommendation
from the Council of Ministers.
Which of the statements given above is/are correct?
(a) 2 only
(b) 1 and 3 only
(c) 3 and 4 only
(d) 2 and 3 only

Answer: A
Explanation:
Statement 1 is incorrect: Under Article 352, the Constitution stipulates an emergency due to war, external
aggression or armed rebellion. This is popularly known as ‘National Emergency’. However, the
Constitution does not use the expression ‘National Emergency’. It rather employs the expression
‘proclamation of emergency’ to denote an emergency of this type.
Statement 2 is correct: The President can declare a national emergency even before the actual
occurrence of war or external aggression or armed rebellion, if he is satisfied that there is an imminent
danger.
Statement 3 is incorrect: The 42nd Amendment Act of 1976 enabled the President to limit the operation
of a National Emergency to a specific part of India.
Statement 4 is incorrect: The President, however, can proclaim a national emergency only after receiving
a written recommendation from the Cabinet (not Council of Minsiters). This provision was added the
44th Constitutional Amendment.

Source: Indian Polity by M Laxmikanth 6th edition, Chapter-16

13. With respect to the Indian Prime Minister, consider the following statements:
1. The Constitution of India has fixed the tenure of the Prime minister for five years.
2. A person who is not a member of either House of Parliament cannot be appointed as Prime
Minister.
3. As per parliamentary convention, a person must prove his majority in the Lok Sabha before he is
appointed as the Prime Minister.
Which of the statements given above is/are not correct?
(a) 1 only
(b) 2 and 3 only
(c) 3 only
(d) 1, 2 and 3
Answer: D
Explanation:
Statement 1 is incorrect: The term of the Prime Minister is not fixed by the Constitution and he holds
office during the pleasure of the president. However, this does not mean that the President can dismiss
the Prime Minister at any time. So long as the Prime Minister enjoys the majority support in the Lok Sabha,
he cannot be dismissed by the President. However, if he loses the confidence of the Lok Sabha, he must
resign or the President can dismiss him.
Statement 2 is incorrect: The India Constitution provides that the Prime Minister can be a member of
either Houses of parliament. Moreover, in 1997, the Supreme Court held that a person who is not a
member of either House of Parliament can be appointed as Prime Minister for six months, within which,
he should become a member of either House of the Parliament; otherwise, he ceases to be the Prime
Minister.
Statement 3 is incorrect: In 1980, the Delhi High Court held that the Constitution does not require that
a person must prove his majority in the Lok Sabha before he is appointed as the Prime Minister. The
President may first appoint him the Prime Minister and then ask him to prove his majority in the Lok Sabha
within a reasonable period.

Source: Indian Polity by M. Laxmikanth, 6th Edition- Chapter- 19.

14. Consider the following statements:


1. The President can appoint only those persons as ministers who are recommended by the Prime
Minister.
2. When an incumbent Prime Minister dies, it does not necessarily mean that the council of ministers
automatically gets dissolved.
3. The Prime Minister can recommend dissolution of the Lok Sabha to the President at any time.
Which of the statements given above are correct?
(a) 1 and 2 only
(b) 1 and 3 only
(c) 2 and 3 only
(d) 1, 2, and 3

Answer: B
Explanation:
Statement 1 is correct: The Prime Minister recommends persons who are to be appointed as ministers by
the President. The President can appoint only those persons as ministers who are recommended by the
Prime Minister.
Statement 2 is incorrect: The Prime Minister stands at the head of the council of ministers, the other
ministers cannot function when the Prime Minister resigns or dies. In other words, the resignation or
death of an incumbent Prime Minister automatically dissolves the council of ministers and thereby
generates a vacuum.
Statement 3 is correct: The Prime Minister stands at the head of the council of ministers. The Prime
Minister advises the President with regard to summoning and proroguing of the sessions of the
Parliament. He can also recommend dissolution of the Lok Sabha to President at any time.

Source: Indian Polity by M. Laxmikanth, 6th Edition- Chapter- 19.

15. With reference to the Constitution of India, which of the following statements are correct?
1. Article 358 suspends Fundamental Rights under Article 19 completely while Article 359 empowers
the President to suspend the enforcement of all other Fundamental Rights.
2. Article 358 extends to the entire country whereas Article 359 may extend to the entire country or
a part of it.
3. Fundamental Rights under Article 19 can be suspended only when the National Emergency is
declared on the ground of war or external aggression.
Select the correct answer using the code given below:
(a) 1 and 2 only
(b) 2 and 3 only
(c) 1 and 3 only
(d) 1, 2 and 3

Answer: B
Explanation:
Articles 358 and 359 describe the effect of a National Emergency on Fundamental Rights. Article 358
deals with the suspension of the Fundamental Rights guaranteed by Article 19, while Article 359 deals
with the suspension of other Fundamental Rights (except those guaranteed by Articles 20 and 21). The
differences between Articles 358 and 359 can be summarized as follows:
Statement 1 is incorrect: Article 358 automatically suspends the Fundamental Rights under Article 19
as soon as the emergency is declared. On the other hand, Article 359 does not automatically suspend
any Fundamental Right. It empowers the President to suspend the enforcement of other specified
Fundamental Rights except the ones under Articles 20 and 21.
Statement 2 is correct: Article 358 extends to the entire country whereas Article 359 may extend to the
entire country or a part of it.
Statement 3 is correct: The 44th Constitutional Amendment Act of 1978 restricted the scope of Article
358 in two ways. Firstly, the six Fundamental Rights under Article 19 can be suspended only when the
National Emergency is declared on the ground of war or external aggression and not on the ground of
armed rebellion. Secondly, only those laws which are related with the Emergency are protected from
being challenged and not other laws. Also, the executive action taken only under such a law is protected.

Source: Indian Polity by M Laxmikanth 6th edition, Chapter-16

16. Consider the following statements:


1. The distribution of legislative powers between the Centre and the states is rigid.
2. The Parliament can delegate any of the executive functions of the Centre to any state, even
without the consent of that state.
3. The mutual delegation of functions between the Centre and the state can take place only through
a legislation.
Which of the statements given above is/are correct?
(a) 1 only
(b) 1 and 3 only
(c) 2 and 3 only
(d) 1, 2, and 3

Answer: A
Explanation:
Statement 1 is correct: The distribution of legislative powers between the Centre and the states is rigid.
Consequently, the Centre cannot delegate its legislative powers to the states and a single state cannot
request the Parliament to make a law on a state subject. Such a rigid division in the executive sphere
may lead to occasional conflicts between the two. Hence, the Constitution provides for inter-government
delegation of executive functions in order to mitigate rigidity and avoid a situation of deadlock.
Statement 2 is incorrect: The President [not Parliament] may, with the consent of the state government,
entrust to that government any of the executive functions of the Centre. Conversely, the governor of a
state may, with the consent of the Central government, entrust to that government any of the executive
functions of the state. This mutual delegation of administrative functions may be conditional or
unconditional. The Constitution also makes a provision for the entrustment of the executive functions
of the Centre to a state without the consent of that state. But, in this case, the delegation is by the
Parliament and not by the president.
Statement 3 is incorrect: The mutual delegation of functions between the Centre and the state can take
place either under an agreement or by a legislation. While the Centre can use both the methods, a state
can use only the first method.

Source: Indian Polity by M. Laxmikanth, 6th Edition- Chapter- 14.

17. Consider the following pairs:


Article of Constitution Provision
1. Article 268 Taxes levied by the Union but collected and appropriated by the States.
2. Article 269 Taxes levied and collected by the Centre but distributed between the
Centre and the states.
3. Article 270 Taxes levied and collected by the Centre but assigned to the states.
4. Article 271 The Parliament cannot impose the surcharge on the Goods and Services
Tax.
How many pairs given above is/are correctly matched?
(a) Only one pair
(b) Only two pairs
(c) Only three pairs
(d) All four pairs

Answer: B
Explanation:
Pair 1 is correctly matched: Article 268 provides for the taxes levied by the Centre but collected and
appropriated by the States. This category includes the stamp duties on bills of exchange, cheques,
promissory notes, policies of insurance, transfer of shares and others. The proceeds of these duties levied
within any state do not form a part of the Consolidated Fund of India, but are assigned to that state.
Pair 2 is incorrectly matched: Article 269 provides for the taxes levied and collected by the Centre but
assigned to the states. The following taxes fall under this category:
i. Taxes on the sale or purchase of goods (other than newspapers) in the course of inter-state trade
or commerce.
ii. Taxes on the consignment of goods in the course of inter-state trade or commerce.
The net proceeds of these taxes do not form a part of the Consolidated Fund of India. They are assigned
to the concerned states in accordance with the principles laid down by the Parliament.
Pair 3 is incorrectly matched: Article 270 provides for the Taxes levied and collected by the Centre but
distributed between the Centre and the states. This category includes all taxes and duties referred to in
the Union List except the following:
i. Duties and taxes referred to in Articles 268, 269 and 269-A (The Goods and Services Tax (GST) on
supplies in the course of inter-state trade or commerce);
ii. Surcharge on taxes and duties referred to in Article 271 commerce.
iii. Any cess levied for specific purposes.
The manner of distribution of the net proceeds of these taxes and duties is prescribed by the President
on the recommendation of the Finance Commission.
Pair 4 is correctly matched: The Parliament can at any time levy the surcharges on taxes and duties
referred to in Articles 269 and 270 (mentioned above). The proceeds of such surcharges go to the Centre
exclusively. In other words, the states have no share in these surcharges. However, the Goods and
Services Tax (GST) is exempted from this surcharge. In other words, this surcharge cannot be imposed
on the GST.

Source: Indian Polity by M. Laxmikanth, 6th Edition- Chapter-14.

18. Consider the following statements:


1. The discretionary grants are made by the Central government to the states which are in need of
financial assistance and are charged upon the Consolidated Fund of India.
2. The statutory grants under Article 275 are given to the states on the recommendation of the
Finance Commission.
3. The receipts from coinage and currency are the sources of non-tax revenues for the Centre.
Which of the statements given above are correct?
(a) 1 and 2 only
(b) 1 and 3 only
(c) 2 and 3 only
(d) 1, 2, and 3

Answer: C
Explanation:
Statement 1 is incorrect: Article 275 empowers the Parliament (not Central government) to make
grants-in-aid to the states which are in need of financial assistance and not to every state. Also, different
sums may be fixed for different states. These sums are charged on the Consolidated Fund of India every
year. Apart from this general provision, the Constitution also provides for specific grants for promoting
the welfare of the scheduled tribes in a state or for raising the level of administration of the scheduled
areas in a state including the State of Assam.
Statement 2 is correct: The statutory grants under Article 275 (both general and specific) are given to
the states on the recommendation of the Finance Commission.
Statement 3 is correct: The receipts from the following form the major sources of non-tax revenues of
the Centre: (i) posts and telegraphs; (ii) railways; (iii) banking; (iv) broadcasting (v) coinage and currency;
(vi) central public sector enterprises; (vii) escheat and lapse; and (viii) others.

Source: Indian Polity by M. Laxmikanth, 6th Edition- Chapter- 14.

19. Consider the following statements:


1. The American president is not regarded as a constituent part of the legislature.
2. In United Kingdom, the Crown is an integral part of the institution of Parliament.
Which of the statements given above is/are correct?
(a) 1 only
(b) 2 only
(c) Both 1 and 2
(d) Neither 1 nor 2

Answer: C
Explanation:
Statement 1 is correct: Under the Presidential system of America, the President is the Chief head of the
Executive. The administration is vested in him. The Presidential system of America is based upon the
separation of the Executive and the Legislature. So that the President and his Secretaries cannot be
members of the Congress (American legislature), which consists of the Senate (Upper House) and the
House of Representatives (Lower House).
Statement 2 is correct: The Parliament is the highest legislative authority in the United Kingdom. Made
up of the House of Commons, House of Lords and the King (who is the UK's current hereditary monarch).
Along with the House of Commons and the House of Lords, the Crown is an integral part of the institution
of Parliament. The King plays a constitutional role in opening and dissolving Parliament and approving
Bills before they become law. The Crown opens Parliament through the State Opening (marking the
beginning of the Parliamentary year). The Crown also dissolves Parliament before a general election. The
Crown informs Parliament of the government's policy ideas and plans for new legislation in a speech
delivered from the throne in the House of Lords. Although the King makes the speech, the government
draws up the content.
Similarly, the President of India is not a member of either House of Parliament and does not sit in the
Parliament to attend its meetings, yet he is an integral part of the Parliament. This is because a bill
passed by both the Houses of Parliament cannot become law without the President’s assent.
Source:
Indian Polity by M. Laxmikanth -6th Edition-Chapter- 22
https://www.parliament.uk/about/how/role/relations-with-other-institutions/parliament-
crown/#:~:text=Along%20with%20the%20House%20of,Bills%20before%20they%20become%20law

20. Which of the following statements is/are correct?


1. The Constitution of India empowers the President to determine the manner in which delimitation is
to be carried out.
2. Currently, the total number of existing seats as allocated to various states in the Lok Sabha and State
assemblies is based on the 1971 Census.
3. The 87th Amendment Act of 2003 empowered the government to undertake readjustment and
rationalization of territorial constituencies in the states on the basis of the population figures of 1971
census.
Select the correct answer using the code given below:
(a) 1 only
(b) 1 and 3 only
(c) 2 only
(d) 1, 2, and 3
Answer: C
Explanation:
Statement 1 is incorrect: The Constitution of India has empowered the Parliament to determine the
authority and the manner in which delimitation is to be made. Accordingly, the Parliament has enacted
the Delimitation Commission Acts in 1952, 1962, 1972 and 2002 for this purpose.
Statement 2 is correct: The 42nd Amendment Act of 1976 froze the allocation of seats in the Lok Sabha
to the states and the division of each state into territorial constituencies till the year 2000 at the 1971
level. Articles 82 and 170 of the Constitution were amended to impose a moratorium on the number of
seats allocated to states and fixing of boundaries of constituencies until the publication of 2001 Census
figures.
However, in 2001, the 84th Constitutional Amendment Act extended the freeze on the number of
constituencies by another 25 years – to 2026. In other words, the total number of existing seats as
allocated to various states in the ‘house of the people’ (the Lok Sabha) and state assemblies on the basis
of 1971 Census shall remain unchanged till the first census to be taken after the year 2026.

Statement 3 is incorrect: The 84th Amendment Act of 2001 also empowered the government to
undertake readjustment and rationalisation of territorial constituencies in the states on the basis of the
population figures of 1991 census. Later, the 87th Amendment Act of 2003 provided for the delimitation
of constituencies on the basis of 2001 census and not 1991 census. However, this can be done without
altering the number of seats allotted to each state in the Lok Sabha.
The ‘freeze on constituencies’ led to another set of problems. Over the years, the proportion of Scheduled
Caste and Scheduled Tribe population, relative to the general population, grew from 15 per cent to 16.6
per cent, and 7.5 to 8.6 per cent respectively. As per the mandate of Article 330 of the Constitution read
with Section 3 of the Representation of People Act, 1961, the number of seats to be reserved for them
had to be adjusted. This was the context in which the 87th Amendment was carried out to provide for the
refixing of the reserved seats on the basis of 2001 census and not 1991 census.
Additional Information:
After the Reorganisation of Jammu and Kashmir Act of 2019, the state was reconstituted as the UTs of
Ladakh and J&K. A Delimitation Commission was established under Justice Ranjana Desai to delimit
constituencies in J&K on the basis of the 2011 census. Based on this, Jammu will have 43 and Kashmir
will have 47 seats in the assembly, and for the first time, nine seats will be reserved for Scheduled Tribes
and seven for Scheduled Castes. Another 24 seats have been kept for constituencies in Pakistan-occupied
Kashmir (PoK).

Sources:
https://theprint.in/opinion/is-india-ready-for-delimitation-of-constituencies-time-has-come-for-
reorganisation-of-states/793024/
https://www.thehindu.com/news/national/law-ministry-notifies-may-20-as-date-for-delimitation-order-
implementation/article65440305.ece
Polity by M. Laxmikanth -6th Edition-Chapter-22
21. With respect to the recommendations of various commissions, consider the following statements:
1. The Administrative Reforms Commission in 1966 recommended the establishment of an Inter-
State Council under Article 263 of the Constitution.
2. The Sarkaria commission recommended that the governor cannot dismiss the council of ministers
so long as it commands a majority in the assembly.
3. The Punchhi Commission recommended that a person who is appointed as the governor must be
from outside the state.
Which of the statements given above are correct?
(a) 1 and 2 only
(b) 1 and 3 only
(c) 2 and 3 only
(d) 1, 2, and 3
Answer: D
Explanation:
Statement 1 is correct: The Central government appointed a six-member Administrative Reforms
Commission (ARC) in 1966 under the chairmanship of Morarji Desai (followed by K Hanumanthayya). It
recommended the establishment of an Inter-State Council under Article 263 of the Constitution. No
action was taken by the Central government on the recommendations of the ARC.
Statement 2 is correct: In 1983, the Central government appointed a three-member Commission on
Centre-state relations under the chairmanship of R.S. Sarkaria, a retired judge of the Supreme Court.
The commission was tasked with examining and reviewing the existing arrangements between the Centre
and states in all spheres and recommend appropriate changes and measures.
It made 247 recommendations to improve Centre-state relationship. One of such recommendations was
that the governor cannot dismiss the council of ministers so long as it commands a majority in the
assembly.
Statement 3 is correct: The Second commission on Centre-State Relations was set-up by the Government
of India in April 2007 under the Chairmanship of Madan Mohan Punchhi, former Chief Justice of India. It
recommended that, while selecting Governors, the Central Government should adopt the following
strict guidelines as recommended in the Sarkaria Commission report and follow its mandate in letter and
spirit:
• He should be eminent in some walk of life
• He should be a person from outside the state
• He should be a detached figure and not too intimately connected with the local politics of the state.
• He should be a person who has not taken too great a par politics generally and particularly in the
recent past.
Source: Indian Polity by M. Laxmikanth, 6th Edition- Chapter- 14.

22. Consider the following statements:


1. The Inter-State Water Disputes Act empowers the President to set up an ad hoc tribunal for the
adjudication of inter-state river disputes.
2. The President can establish Inter State Council any time to serve the public interest.
3. Each Chief Minister acts as a chairman of the Zonal Council by rotation for a period of one year at
a time.
Which of the statements given above is/are correct?
(a) 1 and 2 only
(b) 2 only
(c) 2 and 3 only
(d) 3 only

Answer: B
Explanation:
Statement 1 is incorrect: The Inter-State Water Disputes Act empowers the Central government to set
up an ad hoc tribunal for the adjudication of a dispute between two or more states in relation to the
waters of an inter-state river or river valley. The decision of the tribunal would be final and binding on
the parties to the dispute. Neither the Supreme Court nor any other court is to have jurisdiction in respect
of any water dispute which may be referred to such a tribunal under this Act. So far (2019), the Central
government has set up nine inter-state water dispute tribunals.
Statement 2 is correct: Article 263 of the Constitution contemplates the establishment of an Inter-State
Council to effect coordination between the states and between Centre and states. The President can
establish such a council if at any time it appears to him that the public interest would be served by its
establishment. He can define the nature of duties to be performed by such a council and its organisation
and procedures.
Statement 3 is incorrect: The Zonal Councils are statutory (and not Constitutional) bodies. They are
established by an Act of the Parliament, that is, States Reorganisation Act of 1956. The Act divided the
country into five zones (Northern, Central, Eastern, Western and Southern) and provided a zonal council
for each zone. Each zonal council consists of the following members:
• Home minister of Central government.
• Chief ministers of all the States in the zone.
• Two other ministers from each state in the zone.
• Administrator of each union territory in the zone.
Besides, the following persons can be associated with the zonal council as advisors (i.e., without the right
to vote in the meetings): (i) a person nominated by the Planning Commission; (ii) chief secretary of the
government of each state in the zone; and (iii) development commissioner of each state in the zone.
The Home minister of Central government is the common chairman of the five zonal councils. Each Chief
Minister acts as a vice-chairman of the council by rotation, holding office for a period of one year at a
time.
Source: Indian Polity by M. Laxmikanth, 6th Edition- Chapter- 14.

23. With respect to the Vice President of India, consider the following statements:
1. Both elected and nominated members of the two Houses of Parliament participate in the election
of the Vice President.
2. He can be removed by a resolution passed by the Rajya Sabha with a 2/3rd majority of the total
strength.
3. The Vice President, while acting as the Chairman of the Rajya Sabha, can cast a vote in the case of
an equality of votes.
Which of the statements given above is/are correct?
(a) 1 and 3 only
(b) 1 and 2 only
(c) 2 only
(d) 1, 2 and
Answer: A
Explanation:
Statement 1 is correct: The Vice President of India is elected by the members of an electoral college
consisting of both elected and nominated members of both Houses of Parliament. Here it is to be noted
that the electoral college for the election of the Vice President does not include the members of the state
legislative assemblies.
Statement 2 is incorrect: The Vice-President holds office for a term of five years from the date on which
he enters upon his office. However, he can resign from his office at any time by addressing the resignation
letter to the President. He can also be removed from the office before completion of his term. A formal
impeachment is not required for his removal. According to the provisions of Article 67(c), he can be
removed by a resolution passed by a majority of all the then members of the Rajya Sabha and agreed
to by the Lok Sabha. This means that this resolution should be passed in the Rajya Sabha by an effective
majority and in the Lok Sabha by a simple majority.
Statement 3 is correct: Unlike the Speaker, who is a member of the Lower House, the Chairman (Vice
President) is not a member of the Upper House. But like the Speaker, the Chairman also cannot vote in
the first instance. He too can cast a vote in the case of an equality of votes.
Source: Indian Polity by M. Laxmikanth, 6th Edition- Chapter-18, 22.

24. Consider the following statements regarding the Vice President of India:
1. The Vice President can perform the duties of the chairman of Rajya Sabha while discharging the
functions of the acting President.
2. Unlike the American Vice President, the Indian Vice President only acts as acting President when the
office of the President falls vacant.
Which of the statements given above is/are correct?
(a) 1 only
(b) 2 only
(c) Both 1 and 2
(d) Neither 1 and 2
Answer: B
Explanation:
Statement 1 is incorrect: The Vice President India can act as President when a vacancy occurs in the
office of the President due to his resignation, impeachment, death or otherwise. He can act as President
only for a maximum period of six months within which a new President has to be elected. Further, when
the sitting President is unable to discharge his functions due to absence, illness or any other cause, the
Vice-President discharges his functions until the President resumes his office.
While acting as President or discharging the functions of President, the Vice-President does not perform
the duties of the office of the chairman of Rajya Sabha. During this period, those duties are performed
by the Deputy Chairman of Rajya Sabha.
Statement 2 is correct: The office of the Indian Vice President is modelled on the lines of the American
Vice-President, there is a difference. The American Vice-President succeeds to the presidency when it
falls vacant, and remains President for the unexpired term of his predecessor. The Indian Vice-
President, on the other hand, does not assume the office of the President when it falls vacant for the
unexpired term. He merely serves as an acting President until the new President assumes charge.
Source: Indian Polity by M. Laxmikanth, 6th Edition- Chapter-18.

25. Consider the following statements:


1. As per the Parliamentary convention, the President is bound by the advice rendered by the council of
ministers.
2. The 91st Constitutional Amendment Act has fixed the minimum strength of the Council of Ministers
as 15% of the total strength of the Lok Sabha.
3. A member of either house of the Parliament who is disqualified on the ground of defection also gets
disqualified to be appointed as a minister.
Which of the statements given above are not correct?
(a) 1 and 2 only
(b) 1 and 3 only
(c) 2 and 3 only
(d) 1, 2, and 3
Answer: A
Explanation:
Statement 1 is incorrect: Article 74 provides for a council of ministers with the Prime Minister at the head
to aid and advise the President in the exercise of his functions. Moreover, the 42nd Constitutional
Amendment Act of 1976 provided that the President shall, in the exercise of his functions, act in
accordance with the advice rendered by the council of ministers. The 44th Constitutional Amendment
Act of 1978 further added a proviso to this article to the effect that the president may require the council
of ministers to reconsider such advice and the president shall act in accordance with the advice tendered
after such reconsideration. Furthermore, in 1971, the Supreme Court held that ‘even after the dissolution
of the Lok Sabha, the council of ministers does not cease to hold office. Article 74 is mandatory and,
therefore, the President cannot exercise the executive power without the aid and advise of the council of
ministers. Any exercise of executive power without the aid and advice will be unconstitutional as being
violative of Article 74’.
Statement 2 is incorrect: The total number of ministers, including the Prime Minister, in the Council of
Ministers shall not exceed 15% of the total strength of the Lok Sabha. This provision was added by the
91st Constitutional Amendment Act of 2003. So, the maximum strength of the council of ministers has
been fixed, not minimum.
Statement 3 is correct: The 91st Constitutional Amendment Act of 2003 has also provided that, a
member of either house of Parliament belonging to any political party who is disqualified on the ground
of defection shall also be disqualified to be appointed as a minister.
Source: Indian Polity by M. Laxmikanth -6th Edition-Chapter-20.

26. With respect to the Cabinet Committees, consider the following statements:
1. The Prime Minister acts as the chairman of all the Cabinet Committees of the Parliament.
2. The Cabinet Committees can also consist of non-cabinet ministers, not necessarily cabinet
ministers only.
Which of the statements given above is/are correct?
(a) 1 only
(b) 2 only
(c) Both 1 and 2
(d) Neither 1 nor 2

Answer: B
Explanation:
Statement 1 is incorrect: The Cabinet Committees are extra-constitutional in emergence. In other words,
they are not mentioned in the Constitution. However, the Rules of Business provide for their
establishment. They are of two types–standing and ad hoc. The former are of a permanent nature while
the latter are of a temporary nature. The ad hoc committees are constituted from time to time to deal
with special problems. They are disbanded after their task is completed.
The following four are the more important Cabinet Committees:
 The Political Affairs Committee deals with all policy matters pertaining to domestic and foreign
affairs.
 The Economic Affairs Committee directs and coordinates the governmental activities in the economic
sphere.
 Appointments Committee decides all higher-level appointments in the Central Secretariat, Public
Enterprises, Banks and Financial Institutions.
 Parliamentary Affairs Committee looks after the progress of government business in the Parliament.
The Political Affairs, Economic Affairs, and Appointments Committees are chaired by the Prime
Minister, while the Parliamentary Affairs Committee is chaired by the Home Minister. In either case, if
the Prime Minister is a member of a committee, he invariably presides over it.
Statement 2 is correct: Cabinet Committees are set up by the Prime Minister according to the exigencies
of the time and requirements of the situation. Hence, their number, nomenclature, and composition
varies from time to time. Their membership varies from three to eight. They usually include only Cabinet
Ministers. However, the non-cabinet Ministers are not debarred from their membership.
Source: Polity by M. Laxmikanth, 6th Edition-Chapter-21.

27. With reference to the Parliament of India, which of the following Parliamentary Committees
scrutinizes and reports to the House whether the powers to make regulations, rules, sub-rules, by-laws,
etc. conferred by the Constitution or delegated by the Parliament are being properly exercised by the
Executive within the scope of such delegation?
(a) Committee on Government Assurances
(b) Committee on Subordinate Legislation
(c) Rules Committee
(d) Business Advisory Committee

Answer: B
Explanation:
Broadly, parliamentary committees are of two kinds–Standing Committees and Ad Hoc Committees. The
former is permanent (constituted every year or periodically) and works on a continuous basis, while the
latter is temporary and ceases to exist on completion of the task assigned to them.
Option (b) is correct: This committee investigates and informs the House as to whether the Executive is
correctly using the authority granted to it by the Constitution or delegated by the Parliament to enact
regulations, rules, sub-rules, and bye-laws. The committee has 15 members in both Houses.
This committee reviews the guarantees, promises, and undertakings that ministers occasionally make
on the House floor and provides a report on how well they have been followed through. It has a total of
10 members in the Rajya Sabha and 15 members in the Lok Sabha. This committee examines the rules of
procedure and how business is conducted in the House and makes recommendations for any required
changes or additions. This committee controls the House's agenda and schedule. It allots time for the
government to conduct legislative and other matters that are brought before the House.

Source: UPSC CSE PYQ 2018

28. With reference to the Indian judiciary, consider the following statements:
1. Any retired judge of the Supreme Court of India can be called back to sit and act as a Supreme
Court judge by the Chief Justice of India with the prior permission of the President of India.
2. A High Court in India has the power to review its own judgment as the Supreme Court does.
Which of the statements given above is/are correct?
(a) 1 only
(b) 2 only
(c) Both 1 and 2
(d) Neither 1 nor 2

Answer: A
Explanation:
Statement 1 is correct: The Chief Justice of India may request that a retired Supreme Court judge or a
retired high court judge (duly qualified for appointment as a Supreme Court judge) act as a Supreme
Court judge for a limited time. He can only do so with the prior approval of both the President and the
person to be appointed. Such a judge is entitled to whatever allowances the president deems appropriate.
He will also have all of the jurisdiction, powers, and privileges of a Supreme Court justice.
Statement 2 is incorrect: A High Court, as a court of record, has the authority to correct its own judgment,
order, or decision, despite the fact that no specific power of review is granted to it by the Constitution.

Source: UPSC CSE PYQ 2021

29. Consider the following statements regarding the veto powers of the Indian President:
1. The President cannot exercise suspensive veto in case of a money bill.
2. The President has no veto power in the case of a Constitutional Amendment bill.
3. Unlike the American President who has to resend the bill within 10 days, the Indian President has
no such time-rule.
Which of these statements is/are not correct?
(a) 1 only
(b) 2 only
(c) 1 and 3 only
(d) None of the above

Answer: D
Explanation:
Statement 1 is correct: The President cannot exercise suspensive veto in case of money bills. He/She can
either withhold his assent or ratify it. But he cannot send it back for reconsideration.
Statement 2 is correct: In the case of Constitutional Amendment bills, the President has no choice but to
give his assent. The President has no veto power in respect of a constitutional amendment bill. The 24th
Constitutional Amendment Act of 1971 made it obligatory for the President to give his assent to a
constitutional amendment bill.
Statement 3 is correct: The facts about the pocket veto power of the Indian President are as follows.
1. The bill can be kept pending by the President for an indefinite period when he/she exercises
his/her pocket veto.
2. He/She neither rejects the bill nor returns the bill for reconsideration.
3. The Constitution does not prescribe any time-limit to the President within which he has to act
upon the bill.
Therefore, the President uses his pocket veto where he doesn’t have to act upon the bill. Unlike the
American President who has to resend the bill within 10 days, the Indian President has no such time-
rule.

Source: Indian Polity by M Laxmikanth 6th edition, Chapter-17

30. Consider the following statements regarding the President of India:


1. S/He has discretionary powers to dissolve the Lok Sabha if the Council of Ministers loses its
majority.
2. The original Constitution did not bind the President by the advice of the Council of Ministers but
it was done through the 44th Constitutional Amendment.
Which of the statements given above is/are correct?
(a) 1 only
(b) 2 only
(c) Both 1 and 2
(d) Neither 1 nor 2

Answer: A
Explanation:
Statement 1 is correct: Though the President has no constitutional discretion, he has some situational
discretion. In other words, the President can act on his discretion (that is, without the advice of the
ministers) under the following situations:
1. Appointment of Prime Minister when no party has a clear majority in the Lok Sabha or when the
Prime Minister in office dies suddenly and there is no obvious successor.
2. Dismissal of the Council of Ministers when it cannot prove the confidence of the Lok Sabha.
3. Dissolution of the Lok Sabha if the Council of Ministers has lost its majority.
Statement 2 is incorrect: The 42nd Constitutional Amendment Act of 1976 (enacted by the Indira Gandhi
Government) made the President bound by the advice of the Council of Ministers headed by the Prime
Minister. The 44th Constitutional Amendment Act of 1978 (enacted by the Janata Party Government
headed by Morarji Desai) authorized the President to require the Council of Ministers to reconsider such
advice either generally or otherwise. However, he ‘shall’ act in accordance with the advice tendered after
such reconsideration. In other words, the President may return a matter once for reconsideration of his
ministers, but the reconsidered advice shall be binding.

Source: Indian Polity by M Laxmikanth 6th edition, Chapter-17

31. Consider the following differences between First Past the Post (FPTP) system and the Proportional
Representation (PR) system of election:
1. In the FPTP system, every constituency elects one representative whereas in the PR system, more
than one representative may be elected from one constituency.
2. In the FPTP system, a voter votes for a candidate whereas in the PR system, a voter votes for the
party.
3. In the FPTP system, a party may get more seats than proportion of votes in the legislature whereas
in the PR system every party gets seats in the legislature in proportion to the percentage of votes
that it gets.
Which of the statements given above are correct?
(a) 1 and 2 only
(b) 2 and 3 only
(c) 1 and 3 only
(d) 1, 2 and 3

Answer: D
Explanation:
Option (d) is correct: Basic differences between First Past the Post (FPTP) and Proportional
Representation are:
Basis for comparison First Past the Post Proportional Representation

Meaning First Past the Post is a voting Proportional Representation is


system, wherein people cast an electoral device in which
their votes to the candidate of seats are allotted to the
their choice and the one getting political parties on the basis of
the highest number of votes the number of votes polled for
wins. them.

Constituency Entire country is segregated Large geographical areas are


into various geographical units, termed as constituencies.
i.e., constituencies.

Representative One representative is elected One or more representatives


from each constituency. can be elected from one
constituency.

Voting Votes are casted for the Votes are casted for the party.
candidate.

Seats Votes may or may not be equal A party gets seats, as per the
to the seats obtained. proportion of votes, it gets.

Majority Winning candidate may not get Winning candidate gets the
the majority of the votes. majority of the votes.

Accountability Exists Does not exist

Clashing of Ideas Does not prevail May prevail

Source: NCERT Class XI - Indian Constitution at Work - Chapter 3


32. Which of the following are express grounds for the disqualification of Members of Parliament as
provided under Article 102 the Constitution of India?
1. He/she is disqualified for being elected if he/she gets disqualified under any law made by the
Parliament.
2. He/she is disqualified for being elected if he/she is an undischarged insolvent.
3. He/she is disqualified for being elected if he/she fails to lodge an account of his/her election
expenses within time.
4. He/she is disqualified for being elected if convicted for promoting enmity between different
groups.
Select the correct answer using the code given below:
(a) 1 and 2 only
(b) 1, 2 and 3 only
(c) 2 and 3 only
(d) 3 and 4 only

Answer: A
Explanation:
Statements 1 and 2 are correct: Article 102 of the Indian Constitution establishes conditions for a
member of either House of Parliament to be disqualified. If the member fails to comply with the stated
rules, he/she will be disqualified.
1. He/She holds a profit-making office within the Government of India or any State government
(except that of a Minister or any other office exempted by Parliament),
2. He/She is found to be of unsound mind by a competent court,
3. He/She is discovered to be an undischarged insolvent,
4. He/She is not an Indian citizen, or if he has deliberately gained the citizenship of another country,
or if he has pledged allegiance or adherence to another country;
5. Any law passed by Parliament disqualifies him/her.
Statement 3 is incorrect: The disqualification criteria on account of failing to lodge an account of his
expenses in an election is not mentioned in the Constitution (Article 102) but given in the Representation
of the People Act (1951).
Statement 4 is incorrect: If a person is convicted in promoting enmity among different groups and in
offence of bribery, he must be disqualified as prescribed in the Representation of the People Act (1951)
and not under Article 102 of the Indian Constitution.

Source: Indian Polity by M Laxmikanth 6th edition, Chapter-22, 72

33. Under the Tenth Schedule of the Constitution, which one of the following would not generally lead to
disqualification?
(a) If a member of a House voluntarily gives up the membership of his political party
(b) If a member of a House votes contrary to the directions of his political party.
(c) If an independent candidate joins a political party after the election.
(d) If a nominated member joins a party within six months after he becomes a member of the house.

Answer: D
Explanation:
Option (d) is correct: The provisions for disqualification provided under the anti-defection law (Tenth
Schedule) of the Indian Constitution are as follows –
● If a member of a house belonging to a political party:
○ Voluntarily gives up the membership of his political party, or
○ Votes, or does not vote in the legislature, contrary to the directions of his political party.
However, if the member has taken prior permission, or is condoned by the party within
15 days from such voting or abstention, the member shall not be disqualified.
● If an independent candidate joins a political party after the election.
● If a nominated member joins a party six months after (not within) he becomes a member of the
legislature.

Source: Indian Polity by M Laxmikanth 6th edition, Chapter-76

34. Consider the following statements:


1. The Inter-State Council is established by an Act of the Parliament, on the request of concerned
states.
2. The President can define the nature of duties to be performed by the Inter-State Council and its
organization and its procedure.
3. The Parliament assigns the duties to the Inter-State Council and provides the guidelines which it
has to follow while performing its duties.
4. The President can establish an Inter-State Council if at any time it appears to him that the public
interest would be served.
Which of the statements given above are not correct?
(a) 1 and 3
(b) 2 and 4 only
(c) 3 and 4 only
(d) 2, 3 and 4

Answer: A
Explanation:
Statement 1 is incorrect: According to the provisions of Article 263 of the Indian Constitution, Inter-State
council is not established by an Act of Parliament or the Parliament, but by the President, at any time it
appears to him that public interest would be served.
Statement 2 is correct: According to the provisions of Article 263 of the Indian Constitution, the President
is empowered to define the nature of the duties to be performed by the Inter-State Council along with
its organization and procedures.
Statement 3 is incorrect: The Parliament cannot assign duties or provide guidelines for the functioning
of the Inter-State Council.
Statement 4 is correct: The President can establish an Inter-State Council for enquiring into and advising
upon Inter-state disputes, if any time it appears to him that the public interests would be served by the
establishment of such a council.

Source: Indian Polity by M Laxmikanth 6th edition, Chapter-14;


Introduction to the Constitution of India by D.D Basu, 23rd edition
35. Consider the following statements with respect to National Emergency in India:
1. The 42nd Constitutional Amendment Act of 1976 made the declaration of a National Emergency
immune from judicial review.
2. The Supreme Court in the Minerva Mills case held that the proclamation of a national emergency
can be challenged in a court on the grounds of malafide.
3. The proclamation of Emergency must be approved by both the Houses of Parliament within two
months from the date of its issue.
Which of the statements given above is/are correct?
(a) 1 only
(b) 2 only
(c) 3 only
(d) 1 and 2 only

Answer: B
Explanation:
Statement 1 is incorrect: The 38th Constitutional Amendment Act of 1975 made the declaration of a
National Emergency immune from the judicial review. But, this provision was subsequently deleted by
the 44th Constitutional Amendment Act of 1978.
Statement 2 is correct: In the Minerva Mills case, 1980, the Supreme Court held that the proclamation
of a national emergency can be challenged in a court on the ground of malafide or that the declaration
was based on wholly extraneous and irrelevant facts or is absurd or perverse.
Statement 3 is incorrect: The proclamation of Emergency must be approved by both the Houses of
Parliament within one month from the date of its issue. Originally, the period allowed for approval by
the Parliament was two months, but was reduced by the 44th Constitutional Amendment Act of 1978.
However, if the proclamation of emergency is issued at a time when the Lok Sabha has been dissolved or
the dissolution of the Lok Sabha takes place during the period of one month without approving the
proclamation, then the proclamation survives until 30 days from the first sitting of the Lok Sabha after its
reconstitution, provided the Rajya Sabha has in the meantime approved it.

Source: Indian Polity by M Laxmikanth 6th edition, Chapter-16

36. Consider the following statements with respect to effects of National Emergency on Centre-State
relations:
1. During a national emergency, the Parliament becomes empowered to make laws on any subject
mentioned in the State List.
2. The laws made by Parliament on the state subjects during a National Emergency become
inoperative six months after the emergency has ceased to operate.
3. The legislative powers of the Parliament not only extend to a state where the Emergency is in
operation but also to any other state.
Which of the statements given above are correct?
(a) 1 and 2 only
(b) 2 and 3 only
(c) 1 and 3 only
(d) 1, 2 and 3
Answer: D
Explanation:
Statement 1 is correct: During a national emergency, the Parliament becomes empowered to make laws
on any subject mentioned in the State List. Although the legislative power of a state legislature is not
suspended, it becomes subject to the overriding power of the Parliament. Thus, the normal distribution
of the legislative powers between the Centre and states is suspended, though the state Legislatures are
not suspended. In brief, the Constitution becomes unitary rather than federal.
Statement 2 is correct: The laws made by Parliament on the state subjects during a National Emergency
become inoperative six months after the emergency has ceased to operate. Notably, while a
proclamation of national emergency is in operation, the President can issue ordinances on the state
subjects also, if the Parliament is not in session.
Statement 3 is correct: The 42nd Constitutional Amendment Act of 1976 provided that the legislative
consequences and effects extend not only to a state where the Emergency is in operation but also to
any other state.

Source 1: Indian Polity by M Laxmikanth 6th edition, Chapter-16


Source 2: Introduction to the Constitution of India by D.D Basu 23rd edition

37. Consider the following statements with respect to President’s rule in India:
1. Article 365 empowers the President to issue a proclamation, if he is satisfied that a situation has
arisen in which the government of a state cannot be carried on in accordance with the provisions
of the Constitution.
2. Article 356 provides that whenever a state fails to comply with or to give effect to any direction
from the Centre, it will be lawful for the President to hold that a situation has arisen in which the
government of the state cannot be carried on in accordance with the provisions of the
Constitution.
3. Every resolution approving the proclamation of President’s Rule or its continuation can be passed
by either House of Parliament by a simple majority.
Which of the statements given above is/are not correct?
(a) 1 only
(b) 1 and 2 only
(c) 2 and 3 only
(d) 1, 2 and 3

Answer: B
Explanation:
The President’s Rule in a state can be proclaimed on two grounds – one mentioned in Article 356 itself
and another in Article 365:
Statement 1 is incorrect: Article 356 empowers the President to issue a proclamation, if he is satisfied
that a situation has arisen in which the government of a state cannot be carried on in accordance with
the provisions of the Constitution. Notably, the President can act either on a report of the governor of
the state or otherwise too (ie, even without the governor’s report).
Statement 2 is incorrect: Article 365 says that whenever a state fails to comply with or to give effect to
any direction from the Centre, it will be lawful for the President to hold that a situation has arisen in
which the government of the state cannot be carried on in accordance with the provisions of the
Constitution.
Statement 3 is correct: Every resolution approving the proclamation of President’s Rule or its
continuation can be passed by either House of Parliament only by a simple majority, that is, a majority
of the members of that House present and voting.

Source: Indian Polity by M Laxmikanth 6th edition, Chapter-16

38. With reference to the powers acquired by the President during President’s Rule, which of the following
statements hold true?
1. He/She dismisses the state council of ministers headed by the chief minister.
2. He/She dissolves the state legislative assembly.
3. He/She can declare that the powers of the state legislature are to be exercised by the Parliament.
Select the correct answer using the code given below:
(a) 1 only
(b) 1 and 3 only
(c) 2 and 3 only
(d) 1, 2 and 3

Answer: B
Explanation:
Article 355 imposes a duty on the Centre to ensure that the government of every state is carried on in
accordance with the provisions of the Constitution. It is this duty in the performance of which the Centre
takes over the government of a state under Article 356 in case of failure of constitutional machinery in
state. This is popularly known as ‘President’s Rule’. It is also known as ‘State Emergency’ or ‘Constitutional
Emergency’.
Statement 1 is correct: When the President’s Rule is imposed in a state, the President dismisses the state
council of ministers headed by the chief minister. The state Governor, on behalf of the President, carries
on the state administration with the help of the chief secretary of the state or the advisors appointed by
the President. This is the reason why a proclamation under Article 356 is popularly known as the
imposition of ‘President’s Rule’ in a state.
Statement 2 is incorrect: Further, the President either suspends or dissolves the state legislative
assembly. The Parliament passes the state legislative bills and the state budget. The legislative assembly
may or may not be dissolved.
Statement 3 is correct: He/She can declare that the powers of the state legislature are to be exercised
by the Parliament. He/She can take all other necessary steps including the suspension of the
constitutional provisions relating to any body or authority in the state.

Source: Indian Polity by M Laxmikanth 6th edition, Chapter-16

39. Which of the following statements is/are correct regarding Financial Emergency under Article 360 of
the Constitution of India?
1. A proclamation of Financial Emergency shall cease to operate at the expiration of one month, if
not approved by the resolutions of both the Houses of Parliament by then.
2. Financial Emergency once approved by the Parliament continues indefinitely till it is revoked.
3. During Financial Emergency, the President can issue directions for the reduction of salaries and
allowances of all government employees, but excluding that of the Judges of the Supreme Court
and the High Courts.
Select the correct answer using the code given below:
(a) 1 and 2 only
(b) 2 only
(c) 2 and 3 only
(d) 1, 2 and 3

Answer: B
Explanation:
Article 360 empowers the President to proclaim a Financial Emergency if he is satisfied that a situation
has arisen due to which the financial stability or credit of India or any part of its territory is threatened.
Statement 1 is incorrect: A proclamation declaring financial emergency must be approved by both the
Houses of Parliament within two months from the date of its issue. However, if the proclamation of
Financial Emergency is issued at a time when the Lok Sabha has been dissolved or the dissolution of the
Lok Sabha takes place during the period of two months without approving the proclamation, then the
proclamation survives until 30 days from the first sitting of the Lok Sabha after its reconstitution, provided
the Rajya Sabha has in the meantime approved it.
Statement 2 is correct: Once approved by both the Houses of Parliament, the Financial Emergency
continues indefinitely till it is revoked. This implies two things:
1. There is no maximum period prescribed for its operation; and
2. Repeated parliamentary approval is not required for its continuation.
Statement 3 is incorrect: The President may issue directions for the reduction of salaries and allowances
of (a) all or any class of persons serving the Union; and (b) the judges of the Supreme Court and the High
Court. Thus, during the operation of a financial emergency, the Centre acquires full control over the states
in financial matters.

Sources:
Indian Polity by M Laxmikanth, 6th edition, Chapter-16
Introduction to the Constitution of India by D.D Basu, 23rd edition

40. Who among the following are a part of the Union Executive of India?
1. President of India
2. Prime Minister of India
3. Vice-President of India
4. Attorney General of India
5. Cabinet Secretary of India
Select the correct answer using the code given below:
(a) 1 and 3 only
(b) 2, 4 and 5 only
(c) 1, 2, 3 and 4 only
(d) 1, 2, 3, 4 and 5

Answer: C
Explanation:
Option (c) is correct: The executive is the organ that implements the laws made by the legislature and
implements the will of the state. The Union Executive of India consists of
1. the President of India,
2. the Vice-President of India,
3. the Prime Minister of India,
4. the Council of Ministers and
5. the Attorney General of India.
Cabinet Secretary of India is not part of Union Executive.

Source: Indian Polity by M Laxmikanth 6th edition, Chapter-17, 20

41. Consider the following statements regarding the election of the President in India:
1. It is held in accordance with the system of proportional representation by means of the single
transferable vote.
2. Total value of votes of all MLAs is the same as the total value of votes of all MPs.
3. Elected members of the legislative assemblies of the Union Territories of Delhi and Puducherry
also form a part of the electoral college.
Which of the statements given above are correct?
(a) 1 and 2 only
(b) 2 and 3 only
(c) 1 and 3 only
(d) 1, 2 and 3

Answer: D
Explanation:
Statement 1 is correct: The President’s election is held in accordance with the system of proportional
representation by means of the single transferable vote and the voting is by secret ballot. This system
ensures that the successful candidate is returned by the absolute majority of votes.
Statement 2 is correct: The Constitution provides that there shall be uniformity in the scale of
representation of different states as well as parity between the states as a whole and the Union at the
election of the President. Thus, to be consistent with this principle, the value of votes of all MLAs is the
same as the value of votes of all MPs. The value of votes of MLAs from two different states can vary
because the value of the vote of an MLA is directly proportional to the population of his state and inversely
proportional to the number of seats in his legislative assembly.
Statement 3 is correct: The President is elected indirectly by the people of India by members of electoral
college consisting of:
1. the elected members of both the Houses of Parliament;
2. the elected members of the legislative assemblies of the states; and
3. the elected members of the legislative assemblies of the Union Territories of Delhi and
Puducherry.

Source: Indian Polity by M Laxmikanth 6th edition, Chapter-17


42. Consider the following statements:
1. Attorney General of India and Solicitor General of India are the only officers of the Government
who are allowed to participate in the meetings of the Parliament of India.
2. According to the Constitution of India, the Attorney General of India submits his resignation when
the Government which appointed him resigns.
Which of the statements given above is/are correct?
(a) 1 only
(b) 2 only
(c) Both 1 and 2
(d) Neither 1 nor 2

Answer: D
Explanation:
Statement 1 is incorrect: As per Article 76 of the Constitution, only the Attorney General has the right to
participate in the Parliamentary proceedings. This right is not extended to the Solicitor General of India.
Statement 2 is incorrect: Attorney General is not appointed by the Government. Instead, he is appointed
by the President in consultation with the Government. However, traditionally, the Attorney General
resigns from office once the government resigns, as he is appointed on its advice.

Source: UPSC CSE PYQ 2022

43. Consider the following statements:


1. Central Administrative Tribunal (CAT) was set up during the Prime Ministership of Lal Bahadur
Shastri.
2. The Members for CAT are drawn from both judicial and administrative streams.
Which of the statements given above is/are correct?
(a) 1 only
(b) 2 only
(c) Both 1 and 2
(d) Neither 1 nor 2

Answer: B
Explanation:
Statement 1 is not correct: The Central Administrative Tribunal had been established under Article 323
-A of the Constitution for adjudication of disputes and complaints with respect to recruitment and
conditions of service of persons appointed to public services and posts in connection with the affairs of
the Union or other authorities under the control of the Government. It was set up during the Prime
Ministership of Rajiv Gandhi.
Statement 2 is correct: The Central Administrative Tribunal has been established as a specialist body
comprising of Administrative Members and Judicial Members who by virtue of their specialized
knowledge are better equipped to dispense speedy and effective justice.

Source: UPSC CSE PYQ 2009

44. Consider the following statements:


1. The President shall make rules for the more convenient transaction of the business of the
Government of India, and for the allocation among Ministers of the said business.
2. All executive actions of the Government of India shall be expressed to be taken in the name of
the Prime Minister.
Which of the statements given above is/are correct? (2014)
(a) 1 only
(b) 2 only
(c) Both 1 and 2
(d) Neither 1 nor 2

Answer: A
Explanation:
Statement 1 is correct: According to Article 77(3), the President shall make rules for the more convenient
transaction of the business of the Government of India, and for the allocation among Ministers of the
said business.
Statement 2 is incorrect: In India, the president is the de jure head of the state. All executive actions of
the Government of India shall be expressed to be taken in the name of the President, not the prime
minister. The Prime Minister is chief communicator between the Council of Ministers and the President.
He advises the President and advice of the Council of Ministers is binding on the President.

Source: UPSC CSE PYQ 2014

45. Which one of the following statements regarding the Panchayati Raj Elections is not correct?
(a) Reservation of at least one-third of the total number of seats and one third of the total number
of offices of chairpersons for women is mandatory.
(b) The chairperson of Panchayats at the intermediate and district levels is elected indirectly.
(c) Fresh elections to constitute a Panchayat shall be completed before the expiry of a period of six
months from the date of its dissolution.
(d) The State Election Commission makes provisions with respect to matters relating to elections to
the Panchayats.

Answer: D
Explanation:
The provisions regarding the Panchayati Raj Elections are provided in the 73rd constitutional amendment
Act, 1992:
Option (a) is correct: The Act provides for the reservation of not less than one-third of the total number
of seats for women (including the number of seats reserved for women belonging to the SCs and STs).
Further, not less than one-third of the total number of offices of chairpersons in the Panchayats at each
level shall be reserved for women.
Option (b) is correct: All the members of Panchayats at the village, intermediate and district levels shall
be elected directly by the people. Further, the chairperson of Panchayats at the intermediate and district
levels shall be elected indirectly by and from amongst the elected members thereof. However, the
chairperson of a Panchayat at the village level shall be elected in such a manner as the state legislature
determines.
Option (c) is correct: The Act provides for a five-year term of office to the Panchayat at every level.
However, it can be dissolved before the completion of its term. Further, fresh elections to constitute a
Panchayat shall be completed
● before the expiry of its duration of five years; or
● in case of dissolution, before the expiry of a period of six months from the date of its dissolution.
Option (d) is incorrect: The superintendence, direction and control of the preparation of electoral rolls
and the conduct of all elections to the Panchayats shall be vested in the state election commission. But,
the provisions with respect to all matters relating to elections to the Panchayats are made by the state
legislature.

Source: Indian Polity by M Laxmikanth 6th edition, Chapter 38

46. Consider the following statements with reference to the power of Gram Sabha under the PESA
(Provisions of the Panchayats (Extension to Scheduled Areas) Act, 1996 Act:
1. Every Gram Sabha shall be competent to safeguard and preserve the traditions and customs of
the people, their cultural identity, community resources and the customary mode of dispute
resolution.
2. The recommendations of the Gram Sabha or the Panchayats at the appropriate level shall be
mandatory for grant of prospecting license or mining lease for major minerals in the Scheduled
Areas.
3. Every Gram Sabha shall be responsible for the identification of beneficiaries under the poverty
alleviation and other programmes.
Which of the statements given above is/are correct?
(a) 1 only
(b) 1 and 3 only
(c) 2 and 3 only
(d) 1, 2 and 3

Answer: B
Explanation:
Option (b) is correct: The features (or the provisions) of the PESA Act are as follows:
● State legislation on the Panchayats in the Scheduled Areas shall be in consonance with the
customary law, social and religious practices and traditional management practices of community
resources.
● A village shall ordinarily consist of a habitation or a group of habitations or a hamlet or a group of
hamlets comprising a community and managing its affairs in accordance with traditions and
customs.
● Every village shall have a Gram Sabha consisting of persons whose names are included in the
electoral rolls for the Panchayat at the village level.
● Every Gram Sabha shall be competent to safeguard and preserve the traditions and customs of
the people, their cultural identity, community resources and the customary mode of dispute
resolution.
● Every Gram Sabha shall
○ approve of the plans, programmes and projects for social and economic development
before they are taken up for implementation by the Panchayat at the village level; and
○ be responsible for the identification of beneficiaries under the poverty alleviation and
other programmes.
● Every Panchayat at the village level shall be required to obtain from the Gram Sabha a certification
of the utilization of funds for the above plans, programmes and projects.
● The reservation of seats in the Scheduled Areas in every Panchayat shall be in proportion to the
population of the communities for whom the reservation is sought to be given under Part IX of
the Constitution.
● However, the reservation for the Scheduled Tribes shall not be less than one-half of the total
number of seats. Further, all seats of Chairpersons of Panchayats at all levels shall be reserved for
the Scheduled Tribes.
● The recommendations of the Gram Sabha or the Panchayats at the appropriate level shall be
mandatory for grant of prospecting license or mining lease for minor minerals (not major) in
the Scheduled Areas.

Source: Indian Polity by M Laxmikanth 6th edition, Chapter 38

47. With reference to the President of India, which of the following statements is/are not correct?
1. Oath for the President is prescribed in Schedule III of the Constitution along with oath for the
Council of Ministers.
2. The President is completely immune from civil and criminal proceedings during his term in office.
3. The President can be impeached on the grounds of violation of the Constitution as defined in the
Constitution.
Select the answer using code given below.
(a) 1 only
(b) 1 and 2 only
(c) 2 and 3 only
(d) 1, 2 and 3

Answer: D
Explanation:
Statement 1 is incorrect: The oath of the Council of Ministers is present in Schedule III of the Constitution.
The Council of Ministers takes the oath of office and secrecy. However, the oath of the President to
‘preserve, protect and defend the Constitution’ is provided in Article 60 of the Indian Constitution.
Statement 2 is incorrect: During his term of office, he is immune from any criminal proceedings, even in
respect of his personal acts. He cannot be arrested or imprisoned. However, after giving two months’
notice, civil proceedings can be instituted against him during his term of office in respect of his personal
acts.
Statement 3 is incorrect: The President can be removed from office by a process of impeachment for
‘violation of the Constitution’ as per Article 61. However, the Constitution does not define the meaning
of the phrase ‘violation of the Constitution’.

Source: Indian Polity by M Laxmikanth 6th edition, Chapter-17


48. With reference to the impeachment of the President, which one of the statements given below is not
correct?
(a) It can be initiated in the Lok Sabha as well as Rajya Sabha.
(b) Nominated members of both the houses participate in this process even though they are not a
part of the electoral college to elect the President.
(c) The President can be removed from office by this process for ‘proved misbehavior or incapacity’.
(d) Elected members of legislative assemblies of the states do not participate in this even though they
are a part of the electoral college to elect the President.

Answer: C
Explanation:
Option (a) is correct: The impeachment proceedings can be initiated by either house of Parliament.
These charges should be signed by one fourth of members of the house that had initiated the
impeachment proceedings. For Example, in Lok Sabha it should be signed by one-fourth of 543 members.
Options (b) and (d) are correct: An impeachment is a quasi-judicial procedure in the Parliament. In this
context, two things should be noted:
1. The nominated members of either House of Parliament can participate in the impeachment of
the President though they do not participate in his election;
2. The elected members of the legislative assemblies of states and the Union Territories of Delhi
and Puducherry do not participate in the impeachment of the President though they participate
in his election.
Option (c) is incorrect: The President can be removed from office by a process of impeachment for
‘violation of the Constitution’ as per Article 61. However, the Constitution does not define the meaning
of the phrase ‘violation of the Constitution’.

Source: Indian Polity by M Laxmikanth 6th edition, Chapter-17

49. With reference to the Departmental Standing Committees, consider the following statements:
1. Each standing committee consists of equal number of members from both the Lok Sabha and the
Rajya Sabha.
2. No minister is eligible to be nominated as a member of any of the standing committees.
3. These committees secure the accountability of the Council of Ministers to the Parliament by taking
into consideration the day-to-day administration of the concerned ministries.
Which of the statements given above is/are correct?
(a) 1 and 2 only
(b) 2 only
(c) 3 only
(d) 2 and 3 only

Answer: B
Explanation:
On the recommendation of the Rules Committee of the Lok Sabha, 17 Departmentally-Related Standing
Committees (DRSCs) were set up in the Parliament in 1993. In 2004, seven more such committees were
setup, thus increasing their number from 17 to 24.
Statement 1 is incorrect: The 24 departmental standing committees cover under their jurisdiction all the
ministries / departments of the Central Government. Each standing committee consists of 31 members
(21 from Lok Sabha and 10 from Rajya Sabha). The members of the Lok Sabha are nominated by the
Speaker from amongst its own members, just as the members of the Rajya Sabha are nominated by the
Chairman from amongst its members.
Statement 2 is correct: A minister is not eligible to be nominated as a member of any of the
departmental standing committees. In case a member, after his nomination to any of the departmental
standing committees, is appointed a minister, he then ceases to be a member of the committee.
Statement 3 is incorrect: The main objective of these departmental standing committees is to secure
more accountability of the Executive (i.e., the Council of Ministers) to the Parliament, particularly
financial accountability. However, there are limitations imposed upon the functioning of these standing
committees in a sense that they should not consider the matters of day- to-day administration of the
concerned ministries / departments.
Source: Polity by M. Laxmikanth- 6th Edition-Chapter- 23.

50. With reference to the appointment of Judges, consider the following statements:
1. The word ‘collegium’ is not mentioned anywhere in the Constitution of India.
2. The First Judges case ruled that the advice tendered by CJI during the process of consultation is binding
upon the President in the matter of appointments of Judges of the Supreme Court.
3. The Third Judges case held that the sole opinion of the Chief Justice of India does not constitute the
consultation process in the matters of appointment of the judges of the Supreme Court.
Which of the statements given above are correct?
(a) 1 and 2 only
(b) 1 and 3 only
(c) 2 and 3 only
(d) 1, 2 and 3
Answer: B
Explanation:
Statement 1 is correct: The Collegium of judges is the Supreme Court’s invention. It does not figure in
the Constitution. Article 124 simply provides that the judges of the Supreme Court and High Courts are
to be appointed by the President and speaks of a process of consultation.
Statement 2 is incorrect: In the First Judges case (1982), the Supreme Court held that consultation does
not mean concurrence and it only implies exchange of views. But, in the Second Judges case (1993), the
Supreme Court reversed its earlier ruling and changed the meaning of the word consultation to
concurrence. Hence, it ruled that the advice tendered by the Chief Justice of India is binding on the
President in the matters of appointment of the judges of the Supreme Court.
Statement 3 is correct: In the Third Judges case (1998), the Supreme Court opined that the consultation
process to be adopted by the Chief justice of India in the matters of appointment of the judges of the
Supreme Court requires ‘consultation of plurality of judges’. The sole opinion of the Chief Justice of India
does not constitute the consultation process in the matters of appointment of the judges of the Supreme
Court. The Supreme Court, in this case expanded the Collegium to a five-member body, comprising the
CJI and four of his senior-most colleagues. The Supreme Court held that the recommendation made by
the chief justice of India without complying with the norms and requirements of the consultation
process are not binding on the government. And even if two judges give an adverse opinion, he should
not send the recommendation to the government.
Source: Polity by M. Laxmikanth- 6th Edition-Chapter- 26.

51. Consider the following Panchayati Raj committees:


1. Balwant Rai Mehta Committee
2. L.M. Singhvi Committee
3. Ashok Mehta Committee
4. G.V.K.Rao Committee
Which of the following is the correct chronological order of the formation of committees given above?
(a) 1 – 3 – 2 – 4
(b) 1 – 3– 4 – 2
(c) 3 – 1 – 2 – 4
(d) 3 – 1 – 4 – 2

Answer: A
Explanation:
Option (b) is correct: In January 1957, the Government of India appointed a committee to examine the
working of the Community Development Programme (1952) and the National Extension Service (1953)
and to suggest measures for their better working. The chairman of this committee was Balwant Rai G
Mehta. The committee submitted its report in November 1957 and recommended the establishment of
the scheme of ‘democratic decentralization’, which ultimately came to be known as Panchayati Raj.
In December 1977, the Janata Government appointed a committee on Panchayati raj institutions under
the chairmanship of Ashok Mehta. It submitted its report in August 1978 and made 132
recommendations to revive and strengthen the declining Panchayati raj system in the country.
The Committee to review the existing Administrative Arrangements for Rural Development and Poverty
Alleviation Programmes under the chairmanship of G.V.K. Rao was appointed by the Planning
Commission in 1985. The Committee came to the conclusion that the developmental process was
gradually bureaucratised and divorced from the Panchayati Raj. It made some key recommendations to
strengthen the weakening of PRIs by bureaucratisation of development administration aptly called as
‘grass without roots’.
In 1986, Rajiv Gandhi government appointed a committee to prepare a concept paper on ‘Revitalisation
of Panchayati Raj Institutions for Democracy and Development’ under the chairmanship of L.M. Singhvi.

Question Hack: The mnemonic to remember the committees in chronological order – BAGS (Balwant –
Ashok – G.V.K.Rao – Singhvi). Do not confuse G with Gadgil committee.
Source: Indian Polity by M Laxmikanth 6th edition, Chapter 38

52. Considers the following statements regarding the features of the 73rd Constitutional Amendment Act,
1992:
1. It added a new Part-XI to the Constitution of India.
2. It added a new ninth Schedule to the Constitution of India.
3. It gave a practical shape to Article 40 of the Directive Principles of State Policy.
4. It gave a constitutional status to the Panchayati Raj Institutions for the first time ever.
Which of the statements given above is/are correct?
(a) 4 only
(b) 1 and 3 only
(c) 3 and 4 only
(d) 2, 3 and 4 only

Answer: C
Explanation:
Statements 1 and 2 are incorrect: The 73rd Constitutional Amendment Act added a new Part-IX to the
Constitution of India. This part is entitled as ‘The Panchayats’ and consists of provisions from Articles 243
to 243 O. In addition, the Act also added a new Eleventh Schedule to the Constitution. This schedule
contains 29 functional items of the Panchayats. It deals with Article 243-G.
Statement 3 is correct: The Act has also given a practical shape to Article 40 of the Constitution which
says that, “The State shall take steps to organize village Panchayats and endow them with such powers
and authority as may be necessary to enable them to function as units of self-government.” This article
forms a part of the Directive Principles of State Policy.
Statement 4 is correct: The Act gave constitutional status to the Panchayati raj institutions for the first
time. It brought them under the purview of the justiciable part of the Constitution. In other words, the
state governments are under constitutional obligation to adopt the new Panchayati raj system in
accordance with the provisions of the Act. Consequently, neither the formation of Panchayats nor the
holding of elections at regular intervals depend on the will of the state government any more.

Source: Indian Polity by M Laxmikanth 6th edition, Chapter 38

53. Which one of the statements given below is not correct?


(a) The House can be prorogued only after the House is adjourned sine die.
(b) The President is a part of the Union Executive as well as the Parliament.
(c) The Constitution no longer mandates the nomination of members from the Anglo-Indian
community to the Lok Sabha.
(d) When a President is impeached, the Vice-President acts as the President till the new President
takes the oath.

Answer: A
Explanation:
Option (a) is incorrect: A few days after the House is adjourned sine die by the presiding officer, the
President issues a notification for the prorogation of the session. However, the President can also
prorogue the House even when the House is in session.
Option (b) is correct: The President of India is not a member of either of the Houses and does not sit in
the Parliament to attend its meetings but he/she is an integral part of the Parliament. He/She is also a
part of the Union Executive and is the highest formal authority in the country.
Option (c) is correct: Recently passed, the 104th Constitutional Amendment extends by 10 years the
deadline for the abolition of the reservation of seats in the Lok Sabha and State Legislative Assemblies for
members of Scheduled Castes and Scheduled Tribes. The Amendment, however, does not extend the
period of reservation of the two Lok Sabha seats and seats in State Legislative Assemblies reserved for
members of the Anglo-Indian Community. Thus, the practice of the President of India (Governor in the
states) nominating two members of the Anglo-Indian Community (one in the state) on the
recommendation of the Prime Minister of India (CM in states) was effectively abolished.
Option (d) is correct: When the President is impeached, vacancy must be filled within 6 months. The Vice
President acts as the President till the new President takes oath for the office.

Source: Indian Polity by M Laxmikanth 6th edition, Chapter-17

54. Consider the following provisions of the Indian Constitution:


1. A bill seeking a Constitutional Amendment cannot be introduced by a private member.
2. Prior approval of the President is required before the introduction of a Money Bill.
3. A provision for joint sitting exists in case of the Ordinary bills and Money bills but not in the case
of Constitutional Amendment bills.
Which of the statements given above is/are not correct?
(a) 1 only
(b) 2 only
(c) 1 and 3 only
(d) None of the above

Answer: C
Explanation:
Statement 1 is incorrect: A Constitutional Amendment bill can be introduced by both a minister as well
as a private member (member of the House who is not a minister) and does not require any prior
permission of the President too.
Statement 2 is correct: The Constitution lays down a special procedure for the passing of money bills in
the Parliament. A money bill can only be introduced in the Lok Sabha and that too on the
recommendation of the president. Every such bill is considered to be a government bill and can be
introduced only by a minister.
Statement 3 is incorrect: Article 108 of the Indian Constitutions provides for the joint sitting in the case
of a deadlock between the two Houses. The President can summon both the Houses to meet in a joint
sitting for the purpose of deliberating and voting on the bill. It must be noted here that the provision of
joint sitting is applicable to ordinary bills or financial bills only and not to money bills or Constitutional
Amendment bills. In the case of a money bill, the Lok Sabha has overriding powers, while a Constitutional
Amendment bill must be passed by each House separately.

Source: Indian Polity by M Laxmikanth 6th edition, Chapter-22

55. Consider the following statements regarding clemency powers of the President vis-a-vis that of the
Governor:
1. The President can use his/her pardoning powers in case the punishment or sentence is by a Court
Martial while the Governor cannot do so.
2. The scope of pardoning power of the President under Article 72 is wider than the pardoning power
of the Governor under Article 161.
3. The pardoning power of the President is beyond the purview of judicial review and cannot be
challenged in any court of law.
Which of the statements given above is/are correct?
(a) 1 and 2 only
(b) 2 and 3 only
(c) 1 and 3 only
(d) 1, 2 and 3

Answer: A
Explanation:
The Constitution of India grants clemency powers to the President and Governors of States through
Articles 72 and 161 respectively. These powers allow for correction of any possible judicial errors and to
provide relief to those they regard as being punished unduly harshly.
Statement 1 is correct: Under Article 72(1) of the Constitution, the President is empowered to grant
pardons, reprieves, respites or remissions of punishment, or to suspend, remit or commute the sentence
of any individual who has been convicted of offences in all cases where:
1. The punishment or sentence is by a Court Martial;
2. The punishment or sentence is for an offence against any law relating to a matter to which the
executive power of the Union extends;
3. The sentence is a sentence of death.
Statement 2 is correct: Similarly, under Article 161, the Governor of State is empowered to grant pardons,
reprieves, respites or remissions of punishment or to suspend, remit or commute the sentence of any
person convicted of any offence against any law relating to a matter to which the executive power of the
State extends.
Thus, the scope of pardoning power of the President under Article 72 is wider than the pardoning power
of the Governor under Article 161.
Statement 3 is incorrect: The pardoning powers of the President and Governor are bound by the advice
of their respective Councils of Ministers. The Supreme Court has held that the pardoning power is subject
to judicial review in case of mala fide, irrelevant, irrational or discriminating pardons.

Source: Indian Polity by M Laxmikanth 6th edition, Chapter-17

56. Consider the following statements with respect to the Anti-Defection law:
1. The 52nd Constitutional Amendment Act provided for the disqualification of members of
Parliament on grounds of defection.
2. If one-third members of the legislature party split from the party, it does not entail anti-defection.
3. The decision with regard to disqualification cannot be subject to judicial review.
Which of the statements given above is/are correct?
(a) 1 and 2 only
(b) 2 only
(c) 1 only
(d) 2 and 3 only

Answer: C
Explanation:
Statement 1 is correct: Anti-Defection law – The Tenth Schedule was inserted in the Constitution in 1985
by the 52nd Amendment Act. It lays down the process by which legislators may be disqualified on grounds
of defection by the Presiding Officer of a legislature based on a petition by any other member of the
House.
Statement 2 is incorrect: Disqualification provisions –
● If a member of a house belonging to a political party:
○ Voluntarily gives up the membership of his political party, or
○ Votes, or does not vote in the legislature, contrary to the directions of his political party.
However, if the member has taken prior permission, or is condoned by the party within
15 days from such voting or abstention, the member shall not be disqualified.
● If an independent candidate joins a political party after the election.
● If a nominated member joins a party six months after he becomes a member of the legislature.
Exceptions under the law:
● Legislators may change their party without the risk of disqualification in certain circumstances.
The law allows a party to merge with or into another party provided that at least two-thirds of its
legislators are in favour of the merger.
● In such a scenario, neither the members who decide to merge nor the ones who stay with the
original party will face disqualification.
● The provision of the Tenth Schedule pertaining to an exemption from disqualification in case of
the split by one-third members of the legislature party has been deleted by the 91st
Amendment Act of 2003. It means that the defectors have no more protection on grounds of
splits.
Statement 3 is incorrect: The law initially stated that the decision of the Presiding Officer is not subject to
judicial review. This condition was struck down by the Supreme Court in 1992, thereby allowing appeals
against the Presiding Officer’s decision in the High Court and Supreme Court. However, Supreme Court
has held that there may not be any judicial intervention until the Presiding Officer gives his order.

Source: Indian Polity by M Laxmikanth 6th edition, Chapter-76

57. Consider the following statements with reference to the devices of parliamentary proceedings in India:
1. The first hour of every parliamentary sitting is reserved for the Zero Hour.
2. Unlike the Question Hour, the Zero Hour is not mentioned in the Rules of Procedure.
Which of the statements given above is/are correct?
(a) 1 only
(b) 2 only
(c) Both 1 and 2
(d) Neither 1 nor 2

Answer: B
Explanation:
Statement 1 is incorrect: Question Hour – The first hour of every parliamentary sitting is slotted for this.
During this time, the members ask questions and the ministers usually give answers. The questions are of
three kinds, namely, starred, unstarred and short notice.
1. A starred question (distinguished by an asterisk) requires an oral answer and hence
supplementary questions can follow.
2. An unstarred question, on the other hand, requires a written answer and hence, supplementary
questions cannot follow.
3. A short notice question is one that is asked by giving a notice of less than ten days. It is answered
orally.
In addition to the ministers, the questions can also be asked to the private members. Thus, a question
may be addressed to a private member if the subject matter of the question relates to some Bill, resolution
or other matter connected with the business of the House for which that member is responsible. The
procedure in regard to such a question is the same as that followed in the case of questions addressed to
a minister.
Statement 2 is correct: Unlike the Question hour, the Zero hour is not mentioned in the Rules of
Procedure. Zero hour is an informal device available to the members of the Parliament to raise matters
without any prior notice. The zero hour starts immediately after the question hour and lasts until the
agenda for the day (i.e., regular business of the House) is taken up. In other words, the time gap between
the question hour and the agenda is known as zero hour. It is an Indian innovation in the field of
parliamentary procedures and has been in existence since 1962.

Source: Indian Polity by M Laxmikanth 6th edition, Chapter-22

58. Consider the following pairs:


Motion Usage
1. Closure Inaugural address by the President
2. Adjournment To cut short the debate on any matter
3. No-Confidence Can be used in the Lok Sabha to remove the Ministry from office
4. Motion of Thanks To draw attention to a matter of public importance by the President
How many of the pairs given above is/are correctly matched?
(a) Only one pair
(b) Only two pairs
(c) Only three pairs
(d) All four pairs

Answer: A
Explanation:
No discussion on a matter of general public importance can take place except on a motion made with the
consent of the Presiding officer. The House expresses its decisions or opinions on various issues through
the adoption or rejection of motions moved by either ministers or private members.
Pair 1 is incorrectly matched: Closure Motion – It is a motion moved by a member to cut short the debate
on a matter before the House. If the motion is approved by the House, debate is stopped forthwith and
the matter is put to vote.
Pair 2 is incorrectly matched: Adjournment Motion – It is introduced in the Parliament to draw attention
of the House to a definite matter of urgent public importance, and needs the support of 50 members to
be admitted. As it interrupts the normal business of the House, it is regarded as an extraordinary device.
It involves an element of censure against the government and hence Rajya Sabha is not permitted to make
use of this device. The discussion on an adjournment motion should last for not less than two hours and
thirty minutes.
Pair 3 is correctly matched: No-Confidence Motion – Article 75 of the Constitution says that the council
of ministers shall be collectively responsible to the Lok Sabha. It means that the ministry stays in office so
long as it enjoys the confidence of the majority of the members of the Lok Sabha. In other words, the Lok
Sabha can remove the ministry from office by passing a no confidence motion. The motion needs the
support of 50 members to be admitted.
Pair 4 is incorrectly matched: Motion of Thanks – The first session after each general election and the
first session of every fiscal year is addressed by the President. In this address, the president outlines the
policies and programmes of the government in the preceding year and ensuing year. This address of the
president, which corresponds to the ‘speech from the Throne in Britain’, is discussed in both the Houses
of Parliament on a motion called the ‘Motion of Thanks’. At the end of the discussion, the motion is put
to vote. This motion must be passed in the House. Otherwise, it amounts to the defeat of the
government. This inaugural speech of the president is an occasion available to the members of
Parliament to raise discussions and debates to examine and criticize the government and administration
for its lapses and failures.

Source: Indian Polity by M Laxmikanth 6th edition, Chapter-22

59. For election to the Lok Sabha, a nomination paper can be filed by.
(a) anyone residing in India.
(b) a resident of the constituency from which the election is to be contested.
(c) any citizen of India whose name appears in the electoral roll of a constituency.
(d) any citizen of India.

Answer: C
Explanation:
Option (c) is correct: The nomination papers can be filed by anyone who is 25 years of age or older and
is eligible to contest an election to a Lok Sabha seat.
The candidate must be a registered elector of a constituency and a citizen of India. However, if s/he is a
registered voter in a particular state, s/he can contest from any seat in any state. The candidate cannot
be nominated if they have been convicted for some offence and are in prison, or if s/he is out on bail,
pending disposal of his/her appeal. As per the guidelines of the Election Commission, such a person is
disqualified from contesting elections. A person, who has held an office under the Government of India
or under the Government of any State has been dismissed, wants to contest election, he is required to
produce along with his nomination paper, the certificate issued by the Election Commission as per Section
9(2) of the R. P. Act, 1951.

Source: UPSC CSE PYQ 2017

60. Department of Border Management is a Department of which one of the following Union Ministries?
(a) Ministry of Defence
(b) Ministry of Home Affairs
(c) Ministry of Shipping, Road Transport and Highway's
(d) Ministry of Environment and forests
Answer: B
Explanation:
Option (b) is correct: Department of Border Management is under the Ministry of Home Affairs. The
Ministry of Home Affairs (MHA) discharges multifarious responsibilities, the important among them
being- internal security, border management, Centre-State relations, administration of Union Territories,
management of Central Armed Police Forces, disaster management, etc.

Source: UPSC CSE 2008

61. With reference to the provisions laid down in the Constitution, which one of the following statements
is not correct?
(a) The decision of the Speaker of the Lok Sabha regarding whether a Bill is a Money Bill or not is final.
(b) The Speaker’s post goes to a member of the majority party while the Deputy Speaker’s position
to that of the Opposition.
(c) The power to prorogue the House of the People lies with the President while the power to adjourn
it lies with the Lok Sabha Speaker.
(d) Whenever the Lok Sabha is dissolved, the Speaker does not vacate his office and continues till the
newly-elected Lok Sabha meets.

Answer: B
Explanation:
Option (a) is correct: According to Article 110(3), the decision of the Speaker of the Lok Sabha is final in
case a question arises on whether a Bill is a Money bill or not. His decision in this regard cannot be
questioned in any court of law or in either House of the Parliament or even by the President. The Speaker
endorses a bill as a money bill when such bill is transmitted to the Rajya Sabha and when it is presented
to the President for his assent.
Option (b) is incorrect: There is no Constitutional provision which provides that the Speaker’s post goes
to the majority party and the Deputy Speaker’s to the Opposition. The Speaker and Deputy Speaker are
elected by the members of Parliament. They can be of any political party. Since 11th Lok Sabha there was
a trend that the Speaker comes from the ruling party while the deputy speaker from the main opposition
party.
Option (c) is correct: A sitting of Parliament can be terminated by adjournment or adjournment sine die
or prorogation or dissolution (in the case of the Lok Sabha). An adjournment suspends the work in a sitting
for a specified time, which may be hours, days or weeks. Adjournment sine die means terminating a sitting
of Parliament for an indefinite period. It is the President who has the power to dissolve the House of
People as well as prorogue it. The Speaker can only adjourn the sessions of the lower house.
Option (d) is correct: The Speaker holds office from the date of his/her election till immediately before
the first meeting of the next Lok Sabha (for 5 years). The Speaker once elected is eligible for re-election.
Whenever the Lok Sabha is dissolved, the Speaker does not vacate his office and continues till the newly-
elected Lok Sabha meets.

Source: Indian Polity by M Laxmikanth 6th edition, Chapter-22

62. Consider the following statements:


1. The Quorum for Lok Sabha is one-tenth of the total number of members including the Speaker.
2. The Speaker of the Lok Sabha is the ex-officio chairman of the Public Accounts Committee.
3. The Speaker can be removed only by a resolution passed by the Lok Sabha by an effective majority
(i.e., a majority of all the then members of the House)
Which of the statements given above is/are correct?
(a) 1 only
(b) 1 and 3 only
(c) 2 and 3 only
(d) 1, 2 and 3

Answer: B
Explanation:
Statement 1 is correct: Quorum is the minimum number of members required to be present in the
House before it can transact any business. It is one-tenth of the total number of members in each House
including the Presiding Officer as per Article 100(3). It means that there must be at least 55 members
present in the Lok Sabha and 25 members present in the Rajya Sabha, if any business is to be conducted.
If there is no quorum during a meeting of the House, it is the duty of the Presiding officer either to adjourn
the House or to suspend the meeting until there is a quorum.
Statement 2 is incorrect: The Committee on Public Accounts is the oldest Parliamentary Committee and
was first constituted in 1921. It was constituted under the GOI act 1919. The Committee consists of 22
members, 15 members are elected by the Lok Sabha and 7 Members from the Rajya Sabha. The Speaker
is empowered to appoint the Chairman of the Committee from amongst its members. According to
convention, the chairman of this committee is appointed from the opposition party.
Statement 3 is correct: The Speaker is provided with a security of tenure. He can be removed only by a
resolution passed by the Lok Sabha by an effective majority (i.e., a majority of all the then members of
the House) as per Article 94(c) and not by an ordinary majority (i.e., a majority of the members present
and voting in the House). This motion of removal can be considered and discussed only when it has the
support of at least 50 members.

Sources:
Constitution of India [Article 100(3), 94(c)]
Indian Polity by M Laxmikanth 6th edition, Chapter-23

63. With reference to the Deputy Speaker of Lok Sabha, consider the following statements:
1. According to Constitutional provisions, the Speaker’s post goes to the majority party and the
Deputy Speaker’s to the Opposition.
2. When he is the Presiding Officer of the House in absence of the Speaker, he has the same powers
as that of the speaker.
3. If he is a member of a parliamentary committee then he automatically becomes the chairman of
that committee.
4. While the Speaker takes an oath for the office, the Deputy Speaker does not take any oath for the
office.
How many of the statements given above is/are correct?
(a) Only one statement
(b) Only two statements
(c) Only three statements
(d) All four statements

Answer: B
Explanation:
The Deputy Speaker is elected by the House after the Speaker on a day decided by the Speaker. His terms
of office and grounds for removal and procedure for removal, provision for salary and allowances are the
same as that of the Speaker. He resigns by writing to the Speaker.
Statement 1 is incorrect: As per the Constitution of India, the Speaker and Deputy Speaker in Lok Sabha
are elected among its members. Up to the 10th Lok Sabha, both the Speaker and the Deputy Speaker
were usually from the ruling party. Since the 11th Lok Sabha, there has been a consensus that the
Speaker comes from the ruling party (or ruling alliance) and the post of Deputy Speaker goes to the
main opposition party. However, there is no such constitutional mandate.
Statement 2 is correct: When he is the Presiding Officer of the House in absence of the Speaker, he has
the same powers as that of the Speaker. He isn’t subordinate to the Speaker but directly responsible to
the House.
Statement 3 is correct: Whenever the Speaker is present, the Deputy Speaker becomes an ordinary
member of the house. However, if he is a member of a parliamentary committee then he automatically
becomes the chairman of that committee.
Statement 4 is incorrect: Both the Speaker and the Deputy Speaker do not take any oath for office.

Source: Indian Polity by M Laxmikanth 6th edition, Chapter-22

64. Consider the following statements with regard to Vice-President’s role as Chairman of the Rajya Sabha:
1. The Chairman is not a member of the Parliament.
2. The Chairman cannot vote in the first instance like the Speaker, but unlike Speaker he/she does
not have a casting vote in case of an equality of votes
3. The Chairman takes his/her salaries and allowances in the capacity of the Vice President.
Which of the statements given above is/are correct?
(a) 1 only
(b) 2 and 3 only
(c) 1 and 3 only
(d) 1, 2 and 3

Answer: A
Explanation:
Statement 1 is correct and 2 is incorrect: Unlike the Speaker (who is a member of the House), the
Chairman is not a member of the House. But like the Speaker, the Chairman also cannot vote in the first
instance. He too can cast a vote in the case of an equality of votes. The Vice-President cannot preside
over a sitting of the Rajya Sabha as its Chairman when a resolution for his removal is under consideration.
However, he can be present and speak in the House and can take part in its proceedings, without voting,
even at such a time (while the Speaker can vote in the first instance when a resolution for his removal is
under consideration of the Lok Sabha).
Statement 3 is incorrect: The Constitution has not fixed any emoluments for the Vice-President in
that capacity. He draws his regular salary in his capacity as the ex-officio Chairman of the Rajya
Sabha. During any period when the Vice-President acts as President or discharges the functions of the
President, he is not entitled to any salary or allowance payable to the Chairman of the Rajya Sabha. But
he is paid the salary and allowance of the President during such a time.

Source: Indian Polity by M Laxmikanth 6th edition, Chapter-18, 22

65. Consider the following statements with respect to the Panchayati Raj Institutions:
1. The Gram Sabha acts as the foundation of the Panchayati raj system.
2. There is a three-tier system of Panchayati raj in every state – Panchayats at the village, district
and constituency levels.
Which of the statements given above are not correct?
(a) 1 only
(b) 2 only
(c) Both 1 and 2
(d) Neither 1 nor 2

Answer: B
Explanation:
Statement 1 is correct: The 73rd Constitutional Amendment Act provides for a Gram Sabha as the
foundation of the Panchayati raj system. It is a body consisting of persons registered in the electoral rolls
of a village within the area of Panchayat at the village level. Thus, it is a village assembly consisting of all
the registered voters in the area of a Panchayat. It may exercise such powers and perform such functions
at the village level as the legislature of a state determines.
Statement 2 is incorrect: The Act also provides for a three-tier system of Panchayati raj in every state,
that is, Panchayats at the village, intermediate, and district levels. Thus, the Act brings about uniformity
in the structure of Panchayati raj throughout the country. However, a state having a population not
exceeding 20 lakh may not constitute Panchayats at the intermediate level.

Source: Indian Polity by M Laxmikanth 6th edition, Chapter 38

66. Consider the following statements:


1. Direct Election to all seats in Panchayats at the village, intermediate and district level.
2. Fixing tenure of five years at all levels of Panchayats.
3. Providing reservation of seats for members of backward classes at any level.
4. Establishment of state election commission
5. Devolution of Powers and responsibilities upon Panchayats.
How many of the above is/are not the compulsory provisions under the 73rd Constitutional Amendment
Act?
(a) Only one statement
(b) Only two statements
(c) Only three statements
(d) Only four statements

Answer: C
Explanation:
Option (c) is correct: The 73rd constitutional amendment Act of 1992 came into force in India to provide
constitutional status to the Panchayati Raj institutions. It provided for compulsory as well as voluntary
provisions to be framed by the state legislature. The compulsory provisions of the Act are:
● Organization of Gram Sabha
● Creation of a three-tier Panchayati Raj Structure at the district, block and Village levels
● Direct Election to all seats in Panchayats at the village, intermediate and district level
● All the seats in a Panchayat shall be filled by persons chosen by direct elections from territorial
constituencies in the Panchayat area
● The minimum age for contesting elections to Panchayats is 21 years
● Reservation for women in Panchayats up to 33%
● Reservation of seats for SC/ST, in Panchayats, in proportion to their population
● Creation of a State Election Commission to conduct elections
● Fixed 5 years tenure of Panchayats at all levels
● Each state is to constitute a State Finance Commission every five years to review the financial
position of the Panchayat
Providing reservation of seats for members of backward classes as well as Devolution of Powers and
responsibilities upon Panchayats comes under voluntary Provision of 73rd Amendment Act

Source: Indian Polity by M Laxmikanth 6th edition, Chapter 38

67. Consider the following pairs:


Electoral tool Year of introduction
1. Electronic Voting Machine 1998-99
2. None of the Above (NOTA) 2013-14
3. Voter verifiable paper audit trail 2013-14
4. Model Code of Conduct 1991
How many of the above pairs is/are correctly matched?
(a) Only one pair
(b) Only two pairs
(c) Only three pairs
(d) All four pairs

Answer: D
Explanation:
Pair 1 is correctly matched: The Electronic Voting Machines (EVMs) were used for the first time in 1998
on experimental basis in selected constituencies in the elections to the Assemblies of Rajasthan, Madhya
Pradesh and Delhi. The EVMs were used for the first time in the general elections (entire state) to the
Assembly of Goa in 1999.
Pair 2 is correctly matched: The provision for NOTA has been made since General Election to State
Legislative Assemblies of Chhattisgarh, Madhya Pradesh, Mizoram, NCT of Delhi and Rajasthan in 2013
and continued in the General Election to State Legislative Assemblies of Andhra Pradesh, Arunachal
Pradesh, Odisha and Sikkim in 2014 along with the General Elections to the Sixteenth Lok Sabha (2014).
Pair 3 is correctly matched: 3. Voter verifiable paper audit trails (VVPATs) were first used in a by-
election to the Noksen Assembly Constituency of Nagaland held in 2013. Thereafter, VVPATs have been
used in selected constituencies during every General Election to State Legislative Assemblies. VVPATs
were used in eight selected Parliamentary Constituencies in the country in the 2014 Lok Sabha Election.
EVMs with VVPAT ensure the accuracy and transparency of the voting system.
Pair 4 is correctly matched: The Model Code of Conduct was agreed to by all the political parties in 1968.
The Election Commission first effectively put to use the Model Code of Conduct in the year 1991 to
ensure fair elections and a level playing field.

Source: Indian Polity by M Laxmikanth 6th edition, Chapter 73

68. Which one of the following electoral reforms is not correct?


(a) The exit poll results can be broadcast only after the final phase of the elections get over.
(b) A candidate is not allowed to contest election from more than one constituency in a particular
Lok Sabha election.
(c) A person is not eligible to vote at any election if he/she is confined in a prison.
(d) For contesting an election, a person must be registered as a voter in the corresponding
constituency.

Answer: B
Explanation:
Statement 1 is correct: Restriction on exit polls – The EC issued a statement before the 2019 Lok Sabha
elections saying that exit poll results could be broadcast only after the final phase of the elections were
over. This was done to avoid prospective voters from being misguided or prejudiced in any manner.
Statement 2 is incorrect: As per Section33(7) of Representation People Act, 1951, a person cannot
contest from more than two constituencies for a Lok Sabha/Vidhan Sabha election.
Statement 3 is correct: According to Section 62(5) of the Representation of the People Act, 1951, no
person shall vote at any election if he is confined in a prison, whether under a sentence of imprisonment
or transportation or otherwise, or is in the lawful custody of the police.
Statement 4 is correct: For contesting an election as a candidate, a person must be registered as a voter.
Section 4(d) of Representation People Act, 1951 precludes a person from contesting unless he is an
elector in any Parliamentary Constituency. Section 5 (c) of R. P. Act, 1951 has a similar provision for
Assembly Constituencies.
Question Hack: PM Narendra Modi himself contested elections from 2 different constituencies in the
2014 elections.

Source: Indian Polity by M Laxmikanth 6th edition, Chapter 73

69. According to the Constitution, a Money Bill can contain which of the following provisions?
1. The imposition of fines or other pecuniary penalties.
2. The regulation of the borrowing of money by the Union government.
3. The imposition of any tax by local authority for local purposes.
4. The appropriation of money out of the Consolidated Fund of India.
5. The withdrawal of money from the Contingency Fund of India.
Select the correct answer using the code given below:
(a) 1, 2 and 3 only
(b) 2 and 4 only
(c) 2, 4, and 5 only
(d) 1, 4, and 5 only
Answer: C
Explanation:
Option (c) is correct: According to Article 110 of the Constitution, a bill is deemed to be a money bill if it
contains ‘only’ provisions dealing with all or any of the following matters:
 The imposition, abolition, remission, alteration or regulation of any tax.
 The regulation of the borrowing of money by the Union government.
 The custody of the Consolidated Fund of India or the contingency fund of India, the payment of
moneys into or the withdrawal of money from any such fund.
 The appropriation of money out of the Consolidated Fund of India.
 Declaration of any expenditure charged on the Consolidated Fund of India or increasing the
amount of any such expenditure.
 The receipt of money on account of the Consolidated Fund of India or the public account of India
or the custody or issue of such money, or the audit of the accounts of the Union or of a state.
 Any matter incidental to any of the matters specified above.
However, a bill is not to be deemed to be a money bill by reason only that it provides for:
 The imposition of fines or other pecuniary penalties, or
 The demand or payment of fees for licenses or fees for services rendered; or
 The imposition, abolition, remission, alteration or regulation of any tax by any local authority or
body for local purposes.
Source: Indian Polity by M. Laxmikanth- 6th Edition-Chapter- 22.

70. Consider the following statements:


1. The Speaker Pro Tem, who administers the oath to the newly elected members of Lok Sabha is elected
at the first meeting of the newly-elected Lok Sabha.
2. The office of ‘whip’ is based on the parliamentary conventions while the office of the Leader the
Opposition is mentioned in the Parliamentary Statute.
Which of the statements given above is/are correct?
(a) 1 only
(b) 2 only
(c) Both 1 and 2
(d) Neither 1 nor 2
Answer: B
Explanation:
Statement 1 is incorrect: The Speaker Pro Tem is appointed by the President, usually, the seniormost
member of the Lok Sabha. The President himself administers oath to the Speaker Pro Tem. The Speaker
Pro Tem has all the powers of the Speaker. He presides over the first sitting of the newly-elected Lok
Sabha. His main duty is to administer oath to the new members. He also enables the House to elect the
new Speaker. When the new Speaker is elected by the House, the office of the Speaker Pro Tem ceases
to exist. Hence, this office is a temporary office, existing for a few days.
Statement 2 is correct: Though the offices of the leader of the House and the leader of the Opposition
are not mentioned in the Constitution of India, they are mentioned in the Rules of the House and
Parliamentary Statute respectively. The office of ‘whip’, on the other hand, is mentioned neither in the
Constitution of India nor in the Rules of the House nor in a Parliamentary Statute. It is based on the
conventions of the parliamentary government.
Source: Indian Polity by M. Laxmikanth- 6th Edition-Chapter- 22.

71. Arrange the stages through which the budget passes in the Parliament in the correct sequential order:
1. Scrutiny by departmental committees.
2. Passing of finance bill.
3. Voting on demands for grants.
4. Passing of appropriation bill.
Select the correct answer using the code given below:
(a) 1 – 2 – 3 – 4
(b) 1 – 3 – 4 – 2
(c) 2 – 1 – 3 – 4
(d) 1 – 4 – 3 – 2

Answer: B
Explanation:
Option (b) is correct: The budget goes through the following six stages in the Parliament:
 Presentation of budget: Conventionally, the budget is presented to the Lok Sabha by the finance
minister on the last working day of February. Since 2017, the presentation of the budget has been
advanced to 1st of February. The finance minister presents the budget with a speech known as the
‘budget speech’. Further, there shall be no discussion of the budget on the day on which it is
presented to the House.
 General discussion on budget: It begins a few days after its presentation. It takes place in both the
Houses of Parliament and lasts usually for three to four days. During this stage, the Lok Sabha can
discuss the budget as a whole or on any question of principle involved therein but no cut motion can
be moved nor can the budget be submitted to the vote of the House.
 Scrutiny by departmental committees: After the general discussion on the budget is over, the both
the House of Parliament gets adjourned for about three to four weeks. During this gap period, the
24 departmental standing committees of Parliament examine and discuss in detail the demands for
grants of the concerned ministers and prepare reports on them. These reports are submitted to both
the Houses of Parliament for consideration.
 Voting on demands for grants: In the light of the reports of the departmental standing committees,
the Lok Sabha takes up voting of demands. The voting is confined to the votable part of the budget
the expenditure charged on the Consolidated Fund of India is not submitted to the vote (it can only
be discussed). The demands are presented ministry wise. Each demand is voted separately by the Lok
Sabha. During this stage, the members of Parliament can discuss the details of the budget. They can
also move motions to reduce any demand for grant. Such motions are called as ‘cut motion’. A
demand becomes a grant after it has been duly voted. The Rajya Sabha, which can only discuss it and
has no power to vote on the demands for grants.
 Passing of appropriation bill: An appropriation bill is introduced to provide for the appropriation, out
of the Consolidated Fund of India, all money required to meet the grants voted by the Lok Sabha and
the expenditure charged on the Consolidated Fund of India. The Appropriation Bill becomes the
Appropriation Act after it is assented to by the President. This act authorises (or legalises) the
payments from the Consolidated Fund of India.
 Passing of finance bill: The Finance Bill is introduced to give effect to the financial proposals of the
Government of India for the following year. It is subjected to all the conditions applicable to a Money
Bill. Unlike the Appropriation Bill, the amendments (seeking to reject or reduce a tax) can be moved
in the case of finance bill. According to the Provisional Collection of Taxes Act of 1931, the Finance Bill
must be enacted (i.e., passed by the Parliament and assented to by the president) within 75 days. The
Finance Act legalises the income side of the budget and completes the process of the enactment of
the budget.

Source: Polity by M. Laxmikanth- 6th Edition-Chapter- 22.


72. With reference to the Union Government, consider the following statements:
1. N. Gopalaswamy Iyengar Committee suggested that a minister and a secretary be designated
solely for pursuing the subject of administrative reform and promoting it.
2. In 1970, the Department of Personnel was constituted on the recommendation of the
Administrative Reforms Commission, 1966, and this was placed under the Prime Minister's
charge.
Which of the statements given above is/are correct?
(a) 1 only
(b) 2 only
(c) Both 1 and 2
(d) Neither 1 nor 2

Answer: B
Explanation:
Statement 1 is incorrect: Sh. N. Gopalaswamy Ayyangar recommended the grouping of ministries and
the improvement of personnel capabilities in his Report titled 'Reorganization of the Machinery of
Central Government' in 1950. It makes no mention of a minister and a secretary being designated solely
for the purpose of pursuing and promoting administrative reform.
Statement 2 is correct: The Department of Personnel was established in the Cabinet Secretariat in 1970,
based on the recommendations of the Administrative Reforms Commission. It was assigned to the Prime
Minister's Office.

Source: UPSC CSE 2021

73. With reference to Union Government, consider the following statements:


1. The Constitution of India provides that all Cabinet Ministers shall be compulsorily the sitting
members of Lok Sabha only.
2. The Union Cabinet Secretariat operates under the direction of the Ministry of Parliamentary
Affairs.
Which of the statements given above is/are correct?
(a) 1 only
(b) 2 only
(c) Both 1 and 2
(d) Neither 1 nor 2

Answer: D
Explanation:
Statement 1 is incorrect: The original text of the Constitution of India does not mention the word Cabinet.
It was inserted in Article 352 of the Constitution in 1978 by the 44th Constitutional Amendment Act.
Therefore, the Constitution does not provide anything regarding all Cabinet Ministers being
compulsorily the sitting members of Lok Sabha only.
Statement 2 is incorrect: The Union Cabinet Secretariat operates under the direction of the Prime
Minister (not Ministry of Parliamentary Affairs).
About the Cabinet Secretariat:
 It is responsible for the administration of the Government of India (Transaction of Business) Rules,
1961 and the Government of India (Allocation of Business) Rules 1961, facilitating smooth
transaction of business in Ministries/ Departments of the Government.
 This Secretariat provides Secretarial assistance to the Cabinet and its Committees
 It assists in decision-making in Government by ensuring Inter-Ministerial coordination, ironing out
differences amongst Ministries/ Departments and evolving consensus through the
instrumentality of the standing/ ad hoc Committees of Secretaries.

Source: UPSC CSE PYQ 2009

74. Consider the following statements:


1. All the money received by the government in repayment of loans forms the part of Consolidated Fund
of India.
2. The payments made from the Public Account of India are mostly in the nature of banking transactions.
3. The Contingency Fund of India, which is operated by executive action, is held by the Finance Secretary
on behalf of the President.
Which of the statements given above is/are correct?
(a) 1 and 3 only
(b) 2 only
(c) 2 and 3 only
(d) 1, 2 and 3

Answer: D
Explanation:
Statement 1 is correct: Article 266(1) provides for the formation of one Consolidated Fund of India. It is
a fund to which all receipts are credited and all payments are debited. It includes:
o All revenues received by the Government of India.
o All loans raised by the Government by the issue of treasury bills, loans or ways and means of
advances.
o All money received by the government in repayment of loans
All the legally authorised payments on behalf of the Government of India are made out of this fund. No
money out of this fund can be appropriated (issued or drawn) except in accordance with a parliamentary
law.
Statement 2 is correct: All other public money (other than those which are credited to the Consolidated
Fund of India) received by or on behalf of the Government of India shall be credited to the Public Account
of India as per the provisions of Article 266(2). This includes provident fund deposits, judicial deposits,
savings bank deposits, departmental deposits and remittances. This account is operated by executive
action, that is, the payments from this account can be made without parliamentary appropriation. Such
payments are mostly in the nature of banking transactions.
Statement 3 is correct: The Constitution under Article 267 authorised the Parliament to establish a
‘Contingency Fund of India’, into which amounts determined by law are paid from time to time.
Accordingly, the Parliament enacted the contingency fund of India Act in 1950. This fund is placed at the
disposal of the president, and he can make advances out of it to meet unforeseen expenditure pending
its authorisation by the Parliament. The fund is held by the Finance Secretary on behalf of the president.
Like the Public Account of India, it is also operated by executive action.
Source: Polity by M. Laxmikanth- 6th Edition-Chapter- 22

75. Which of the following bills does not lapse on the dissolution of the Lok Sabha?
1. A bill passed by the Lok Sabha but pending in the Rajya Sabha
2. A bill pending in the Rajya Sabha but not passed by the Lok Sabha
3. A bill passed by both Houses but pending assent of the president
Select the correct answer using the code given below:
(a) 1 only
(b) 1 and 2 only
(c) 2 only
(d) 2 and 3 only

Answer: D
Explanation:
Option (d) is correct: When the Lok Sabha is dissolved, all business including bills, motions, resolutions,
notices, petitions and so on pending before it or its committees lapse. They (to be pursued further) must
be reintroduced in the newly-constituted Lok Sabha. However, some pending bills and all pending
assurances that are to be examined by the Committee on Government Assurances do not lapse on the
dissolution of the Lok Sabha. The position with respect to lapsing of bills is as follows:
 A bill pending in the Lok Sabha lapses (whether originating in the Lok Sabha or transmitted to it by the
Rajya Sabha).
 A bill passed by the Lok Sabha but pending in the Rajya Sabha lapses.
 A bill not passed by the two Houses due to disagreement and if the president has notified the holding
of a joint sitting before the dissolution of Lok Sabha, does not lapse.
 A bill pending in the Rajya Sabha but not passed by the Lok Sabha does not lapse.
 A bill passed by both Houses but pending assent of the president does not lapse.
 A bill passed by both Houses but returned by the president for reconsideration of Houses does not
lapse.
Source: Polity by M. Laxmikanth- 6th Edition-Chapter- 22

76. Consider the following statements with respect to the Financial Committees of the Parliament:
1. Both the Public Accounts Committee and the Estimates Committee were set up first in 1921 under
the provisions of the Government of India Act of 1919.
2. A minister cannot be elected as a member of either the Public Accounts Committee or the Estimates
Committee.
3. The chairman of the Committee on Public Undertakings can be appointed from either of the House of
Parliament.
4. Unlike the Public Accounts Committee, the recommendations of Estimates Committee and the
Committee on Public Undertakings are not binding on the ministries.
How many statements given above is/are correct?
(a) Only one statement
(b) Only two statements
(c) Only three statements
(d) All four statements
Answer: A
Explanation:
Statement 1 is incorrect: The Public Accounts Committee was set up first in 1921 under the provisions
of the Government of India Act of 1919 and has since been in existence. At present, it consists of 22
members (15 from the Lok Sabha and 7 from the Rajya Sabha). Whereas, The origin of this Estimates
committee can be traced to the standing financial committee set up in 1921. The first Estimates
Committee in the post-independence era was constituted in 1950 on the recommendation of John
Mathai, the then finance minister. Originally, it had 25 members but in 1956 its membership was raised
to 30. All the thirty members are from Lok Sabha only. The Rajya Sabha has no representation in this
committee.
Statement 2 is correct: The members of PAC (15 from the Lok Sabha and 7 from the Rajya Sabha) are
elected every year by the members of the Parliament from amongst its members according to the
principle of proportional representation by means of single transferable vote. The members of the
Estimates committee come solely from the Lok Sabha. These 30 members are elected every year by the
members of the Lok Sabha from amongst its members according to the principle of proportional
representation by means of single transferable vote. A minister cannot be elected as a member of either
the Public Accounts Committee or the Estimates Committee.
The chairman of the Public Accounts Committee is appointed from amongst its members by the Speaker.
Until 1966 - 67, the chairman of the committee belonged to the ruling party. However, since 1967 a
convention has developed whereby the chairman of the committee is selected invariably from the
Opposition. Whereas, the chairman of the Estimates Committee is appointed by the Speaker from
amongst its members and he is invariably from the ruling party.
Statement 3 is incorrect: The chairman of the Committee on Public Undertakings is appointed by the
Speaker from amongst its members who are drawn from the Lok Sabha only. Thus, the members of the
committee who are from the Rajya Sabha cannot be appointed as the chairman.
Statement 4 is incorrect: The recommendations of all three financial committees, i.e., the Public
Accounts Committee, the Estimates Committee and the Committee on Public Undertakings are not
binding the on the ministries.

Source: Polity by M. Laxmikanth- 6th Edition-Chapter- 23.

77. ‘Meri Policy Mere Hath’ campaign is associated with which one of the following schemes?
(a) Pradhan Mantri Fasal Bima Yojana (PMFBY)
(b) Pradhan Mantri Kaushal Vikas Yojana 3.0 (PMKVY 3.0)
(c) Pradhan Mantri Kisan Samman Nidhi (PM-KISAN)
(d) Pradhan Mantri Suraksha Bima Yojana(PMSBY)

Answer: A
Explanation:
Option (a) is correct: "Meri Policy, Mere Haath" initiative is a doorstep delivery of Pradhan Mantri Fasal
Bima Yojana (crop insurance policies) to those farmers across the country who have taken this insurance
in all implementing States. The campaign aims to ensure all farmers are well aware and equipped with
all information on their policies, land records, the process of claim and grievance redressal under PMFBY.
Option (b) is incorrect: The Ministry of Skill Development and Entrepreneurship (MSDE) has launched the
third phase of its flagship scheme Pradhan Mantri Kaushal Vikas Yojana 3.0 (PMKVY 3.0) in January 2021.
It will encourage and promote skill development throughout the country to address the industry needs,
meet the market demands, impart skills in services and in new-age job roles especially during post
pandemic.
Option (c) is incorrect: Pradhan Mantri Kisan Samman Nidhi (PM-KISAN) is a central sector scheme
launched on 24th February, 2019 to supplement financial needs of land holding farmers. Financial benefit
of Rs 6000/- per year in three equal installments, every four month is transferred into the bank accounts
of farmers’ families across the country through Direct Benefit Transfer (DBT) mode. The scheme was
initially meant for small and marginal farmers (SMFs) having landholding upto 2 hectares but scope of
the scheme was extended to cover all landholding farmers with effect from 01.06.2019.
Option (d) is incorrect: Pradhan Mantri Suraksha Bima Yojana (PMSBY) is a one-year accidental insurance
scheme renewable from year to year offering coverage for death or disability due to accident.
Individuals in the age group of 18-70 years having a savings bank or a post office account are entitled to
enroll under the scheme. The risk coverage under the scheme isRs.2 lakh for accidental death and full
disability and Rs. 1 lakh for partial disability.
Sources:
https://pib.gov.in/PressReleaseIframePage.aspx?PRID=1799297
https://pib.gov.in/PressReleasePage.aspx?PRID=1788919
https://pib.gov.in/PressReleasePage.aspx?PRID=1800851
https://pib.gov.in/PressReleaseIframePage.aspx?PRID=1823754

78. Consider the following statements:


1. Both green bonds and climate bonds are fixed or regular income financial instruments.
2. Unlike the Green bonds, which can be issued by both government and private corporations, the
Climate bonds can only be issued by government or multilateral agencies.
3. Blue bonds are a subset of Green bonds, those used particularly to finance projects related to ocean
conservation.
Which of the statements given above is/are correct?
(a) 1 and 2 only
(b) 2 only
(c) 1 and 3 only
(d) 2 and 3 only

Answer: C
Explanation:
Statement 1 is correct: Climate bonds are fixed-income financial instruments linked to climate change
solutions. They are issued in order to raise finance for climate change solutions, for example mitigation
or adaptation related projects. These might be greenhouse gas emission reduction projects ranging from
clean energy to energy efficiency, or climate change adaptation projects. Most climate bonds are use-of-
proceeds bonds, where the issuer promise to the investors that all the raised funds will only go to specified
climate-related programs or assets, such as renewable energy plants or climate mitigation funding
programs.
On the other hand, according to World bank, green bonds are financial instruments that finance green
projects and provide investors with regular or fixed income payments. Green bonds were created to
fund projects that have positive environmental and/or climate benefits. Over the last 14 years, green
bonds have become an important tool to address the impacts of climate change and related challenges.
The very first green bond was issued in 2007 from multilateral institutions European Investment Bank
(EIB) and World Bank.
Statement 2 is incorrect: Green bonds are issued by multilateral agencies such as the World Bank,
corporations, government agencies and municipalities. In the period between 2007 and 2012,
supranational organisations such as the European Investment Bank and the World Bank, as also
governments, accounted for most of the green bond issue. Since then, corporate interest has risen
sharply. In 2014, bonds issued by corporations in the energy and utilities, consumer goods, and real estate
sectors accounted for a third of the market. Moreover, like normal bonds, climate bonds can be issued
by governments, multi-national banks or private corporations.
Statement 3 is correct: Blue bonds are a subset of Green bonds, those used specifically to finance
projects related to ocean conservation. This includes managing plastic waste, but also promoting marine
biodiversity by ensuring sustainable, clean and ecologically-friendly developments. Recently Financial
Markets regulator SEBI has proposed the concept of blue bonds as a mode of sustainable finance, saying
such securities can be utilised for various blue economy-related activities, including oceanic resource
mining and sustainable fishing.
Sources:
https://bondvigilantes.com/blog/2021/02/green-bonds-blue-bonds-esg-bonds-galore-a-beginners-
guide-for-fixed-income-
investors/#:~:text=Blue%20bonds%20are%20a%20subset,clean%20and%20ecologically%2Dfriendly%20
developments.
https://economictimes.indiatimes.com/markets/stocks/news/sebi-proposes-blue-bonds-concept-for-
sustainable-financing-activities/articleshow/93433121.cms?from=mdr
https://www.climatebonds.net/resources/understanding
https://www.thehindu.com/business/what-are-green-bonds/article7070840.ece
https://www.worldbank.org/en/news/feature/2021/12/08/what-you-need-to-know-about-ifc-s-green-
bonds

79. With respect to Virtual Digital Assets, Consider the following statements:
1. The Budget 2022–2023 made it mandatory for the buyer of a virtual digital asset to deduct 1 percent
of the amount paid to the seller as TDS.
2. According to the Income Tax Act 1961, "virtual digital asset" means any token generated through
blockchain technology.
3. The Budget 2022–2023 has provided a 30% tax on income from the transfer of any virtual digital asset.
Which of the statements given above are correct?
(a) 1 and 2 only
(b) 1 and 3 only
(c) 2 and 3 only
(d) 1, 2 and 3

Answer: B
Explanation:
Statement 1 and 3 are correct: The Finance Minister during Budget 2022 and 2023 announced that any
income from transfer of any virtual digital asset shall be taxed at the rate of 30 per cent. Moreover the
30% tax on virtual assets cannot be mentioned as a replacement of any other income (meaning it cannot
be claimed for deduction). In addition to this, a 1% TDS that will be charged on payments made using
digital assets in order to keep a track on transactions.
Statement 2 is incorrect: Cryptocurrencies and non-fungible tokens (NFTs) are presently unregulated in
India. According to the Income Tax Act 1961, ‘virtual digital asset’ means any information or code or
number or token (not being Indian currency or foreign currency), generated through cryptographic
means or otherwise, by whatever name called, providing a digital representation of value exchanged with
or without consideration, with the promise or representation of having inherent value, or functions as a
store of value or a unit of account including its use in any financial transaction or investment, but not
limited to investment scheme; and can be transferred, stored or traded electronically. The Income Tax
Act does not mention either blockchain or DLT in the definition of VDAs.
Sources:
https://economictimes.indiatimes.com/wealth/tax/budget-2022-levies-30-tax-and-tds-on-crypto-
assets/articleshow/89267756.cms?utm_source=contentofinterest&utm_medium=text&utm_campaign=
cppst
https://economictimes.indiatimes.com/wealth/tax/budget-2022-levies-30-tax-and-tds-on-crypto-
assets/articleshow/89267756.cms
https://www.forbes.com/advisor/in/investing/cryptocurrency/cryptocurrency-tax-in-india/

80. With reference to Saffron Cultivation in India, consider the following statements:
1. Saffron cultivation has long been restricted to a few districts in Jammu and Kashmir.
2. The North East Center for Technology Application and Reach is responsible for the implementation of
the Saffron Bowl project in the North Eastern States.
3. For the first time in northeast India, saffron was successfully cultivated in the state of Sikkim.
4. Recently, some of the districts in Assam and Meghalaya have been identified as suitable for saffron
cultivation.
Which of the statements given above are correct?
(a) 1 and 3 only
(b) 2 and 4 only
(c) 1, 2 and 3 only
(d) 2, 3 and 4 only
Answer: C
Explanation:
Statement 1 is correct: Saffron cultivation is believed to have been introduced in Kashmir by Central Asian
immigrants around 1st Century BCE. In ancient Sanskrit literature, saffron is referred to as ‘bahukam’.
Saffron cultivation has long been restricted to a limited geographical area in J&K, mainly Pampore,
followed by Budgam, Srinagar and Kishtwar districts. The saffron available in Kashmir is of three types
— ‘Lachha Saffron’, with stigmas just separated from the flowers and dried without further processing;
‘Mongra Saffron’, in which stigmas are detached from the flower, dried in the sun and processed
traditionally; and ‘Guchhi Saffron’, which is the same as Lachha, except that the latter’s dried stigmas are
packed loosely in air-tight containers while the former has stigmas joined together in a bundle tied with a
cloth thread. India cultivates about 6 to 7 tonne of saffron annually, but in order to meet the 100 tonne
demand, most of the saffron is being imported from Iran.

Statement 2 is correct: The saffron bowl which was so far confined to Kashmir is being expanded to the
Northeast through the Saffron Bowl Project to explore the feasibility of growing saffron in the region with
the same quality. North East Centre for Technology Application and Reach (NECTAR) is responsible for
implementation of Saffron Bowl project in North Eastern States.
Statement 3 is correct but statement 4 is incorrect: The Northeast saw the successful cultivation of
saffron for the first time in Yangang village of South Sikkim. Recently, the North East Centre for
Technology Application and Reach (NECTAR) under Saffron Bowl project has identified few locations in
Arunachal Pradesh and Meghalaya for saffron cultivation.
Sources:
https://indianexpress.com/article/india/an-experiment-to-grow-saffron-in-sikkim-with-help-from-jk-
7034311/
https://www.thehindu.com/news/national/other-states/kashmir-saffron-gets-gi-
tag/article31484569.ece
https://dst.gov.in/nectar-brings-saffron-bowl-northeast-boosts-technology-sustainable-solutions-
regions-challenges

81. With reference to Battle of Saraighat, consider the following statements:


1. Lachit Borphukan led the Ahom against the Mughals in the Battle of Saraighat.
2. The Battle of Saraighat was a decisive naval battle between the Ahoms and the Mughal warriors.
3. The Battle of Saraighat prevented the Mughals from taking back Assam.
Which of the statements given above is/are correct?
(a) 1 and 2 only
(b) 1 and 3 only
(c) 3 only
(d) 1, 2 and 3
Answer: D
Explanation:
Statement 1 is correct: Ahom kings ruled large parts of what is now Assam, and parts of what are
neighbouring states today, for nearly 600 years between the 13th and 19th centuries. Lachit Borphukan
was a commander in the Ahom Kingdom known for its leadership in the 1671 battle of Saraighat that
thwarted a drawn out attempt by the Mughal warriors.
Statement 2 and 3 are correct: The Battle of Saraighat was considered one of the greatest naval battles
on a river to secure victory over the Mughals. Unlike in Alaboi, where he was forced to fight on land
instead of a naval battle, Lachit in Saraighat enticed the Mughals into a naval battle. According to the
historian H K Barpujari, Ahom forces combined a frontal attack and a surprise attack from behind. They
lured the Mughal fleet into moving ahead by feigning an attack with a few ships from the front. The
Mughals vacated the waters behind them, from where the main Ahom fleet attacked and achieved a
decisive victor. The battle prevented Mughals from taking back Assam. Lachit Borphukan died in April
1672, almost two centuries before the Indian freedom movement had begun. Today, Lachit Barphukan’s
victory is honoured with a gold medal given every year to the best cadet graduating from the National
Defence Academy (NDA).
Sources:
https://www.thehindu.com/news/national/other-states/pm-modi-slammed-for-calling-17th-century-
ahom-general-a-freedom-fighter/article34066126.ece
https://indianexpress.com/article/explained/explained-ahom-warrior-lachit-borphukan-and-the-battles-
of-alaboi-saraighat-7791371/

82. Consider the following statements:


1. Integrated Coastal Zone Management Project was launched by the Environment Ministry in
collaboration with the UNEP to study impact of shoreline change and sea level rise in the country.
2. Recently, the Coastal Vulnerability Index was released by the Indian National Centre for Ocean
Information Services.
3. The Coastal Vulnerability Index uses the relative risk that physical changes will occur as sea-level rises
which are quantified based on parameters like tidal range, wave heigh, coastal slope, etc.
Which of the statements given above is/are correct?
(a) 1 and 2 only
(b) 2 only
(c) 3 only
(d) 2 and 3 only
Answer: D
Explanation:
Statement 1 is incorrect: The Integrated Coastal Zone Management Project is an effort to study the
7,500-km coastline and impact of shoreline change and sea level rise in the country. It is a World Bank
[not UNEP] assisted project, in identified stretches of Gujarat, Odisha and West Bengal from 2010 to
2020 with a purpose of protecting and conserving coastal and marine environment of the country. This
includes development of Integrated Coastal Zone Management Plan(ICZM), prevention of soil erosion,
shelterbelt plantation, mangrove plantation, strengthening requirements for ecosystem monitoring,
biodiversity conservation and sustainable livelihood of coastal communities. The World Bank’s
contributes soft loan/IDA credit which comprises around 78% of total project cost. This ICZM project
assumes special significance in the context of climate change since one of the definitive findings of the
IPCC relates to the increase in mean sea levels as a result of global warming.
Statement 2 and 3 are correct: Indian National Centre for Ocean Information Services(INCOIS), which is
an autonomous body under Ministry of Earth Sciences (MoES), has been issuing alerts on Potential Fishing
Zone, Ocean State Forecast, Tsunami Early Warning, Storm Surge Early Warning, High Wave Alerts, etc.,
through dedicated ocean modeling, observations, computation facilities and the marine data center.
INCOIS has carried out coastal vulnerability assessment for entire Indian coast at states level to bring
out an Atlas comprising 156 maps on 1:1,00,000 scales to prepare a Coastal Vulnerability Index (CVI).
While the maps determine the coastal risks due to future sea-level rise based on the physical and
geological parameters for the Indian coast, the CVI uses the relative risk that physical changes will occur
as sea-level rises are quantified based on parameters like: tidal range; wave height; coastal slope;
coastal elevation; shoreline change rate; geomorphology; and historical rate of relative sea-level
change.
Sources:
https://pib.gov.in/newsite/erelcontent.aspx?relid=60204
https://pib.gov.in/PressReleasePage.aspx?PRID=1812037
https://www.thehindu.com/news/national/telangana/incois-prepares-coastal-vulnerability-
index/article38396960.ece

83. Consider the following statements regarding Hydrogen:


1. Blue hydrogen is a cleaner and cheaper alternative to grey hydrogen.
2. Grey hydrogen accounts for nearly 95% of the hydrogen produced in the world today.
3. Pink hydrogen is considered green because no CO2 emissions occur during its production.
4. National Hydrogen Mission aims to produce 25 million tonnes of Green hydrogen by 2025.
5. Hydrogen has the highest energy content of any common fuel by weight, but it has the lowest energy
content by volume.
Which of the statements given above are correct?
(a) 1, 3 and 4 only
(b) 2, 3, and 5 only
(c) 2, 4 and 5 only
(d) 1, 2, 3 and 5 only
Answer: B
Explanation:
Statement 1 is incorrect: Blue hydrogen is similar to grey hydrogen, except that most of the CO2 emissions
are sequestered (stored in the ground) using carbon capture and storage. Blue hydrogen is a cleaner
alternative to grey hydrogen, but is expensive since carbon capture technology is used.
Statement 2 and 3 are correct: Hydrogen is primarily used in petrochemicals and fertiliser industry and is
produced largely from natural gas, thereby emitting enormous amounts of carbon dioxide. Depending on
the nature of the method of its extraction, Hydrogen is categorised into three categories, namely, grey,
blue and green. Grey hydrogen is hydrogen produced using fossil fuels such as natural gas or coal. Grey
hydrogen accounts for nearly 95% of the hydrogen produced in the world today. The two main
production methods are steam methane reforming and coal gasification. Whereas, Pink hydrogen refers
to hydrogen generated through electrolysis powered by nuclear energy. Pink hydrogen is usually
considered green because it does not produce any CO2 emissions during production.
Statement 4 is incorrect: The National Hydrogen Mission was announced during Budget Speech of 2020-
2021 and was formally launched by Prime Minister on India’s 75th Independence Day. The Mission aims
to aid the government in meeting its climate targets and making India a green hydrogen hub. This will
help in meeting the target of production of 5 million tonnes of Green hydrogen by 2030 and the related
development of renewable energy capacity.
Statement 5 is correct: Hydrogen is emerging as an important source of energy since it has zero carbon
content and is a non-polluting source of energy in contrast to hydrocarbons that have net carbon content
in the range of 75 to 85 per cent. Hydrogen, like electricity, is an energy carrier that must be produced
from another substance. Hydrogen can be produced or separated from a variety of sources including
water, fossil fuels, or biomass and used as a source of energy or fuel. Hydrogen has the highest energy
content of any common fuel by weight (about three times more than gasoline), but it has the lowest
energy content by volume (about four times less than gasoline).
Sources:
https://static.pib.gov.in/WriteReadData/specificdocs/documents/2022/mar/doc202232127201.pdf
https://energyeducation.ca/encyclopedia/Types_of_hydrogen_fuel#cite_ref-hyd2_14-1

84. With reference to advisory jurisdiction of the Indian Supreme Court, consider the following
statements:
1. The Supreme Court must tender its advice to the President on any question of public
importance but not necessarily on disputes arising out of any pre-Constitution treaty.
2. The Supreme Court's advice under the Article 143 is neither a judicial pronouncement nor
binding on the President.
Which of the statements given above is/are correct?
(a) 1 only
(b) 2 only
(c) Both 1 and 2
(d) Neither 1 nor 2
Answer: B
Explanation:
Statement 1 is incorrect: The Constitution, under the Article 143, authorises the president to
seek the opinion of the Supreme Court in the two categories of matters:
 On any question of law or fact of public importance which has arisen or which is likely to
arise.
 On any dispute arising out of any pre-constitution treaty, agreement, covenant,
engagement, sanad or other similar instruments.
In the first case, the Supreme Court may tender or may refuse to tender its opinion to the
president. But, in the second case, the Supreme Court must tender its opinion to the President.
Statement 2 is correct: The opinion expressed by the Supreme Court to the President, under
Article 143, is only advisory and not a judicial pronouncement. Hence, the advice or opinion of
the Supreme Court is not binding on the President, who may follow or may not follow the
opinion tendered by the Supreme Court.
Source: Indian Polity by Laxmikanth, 6th edition, Chapter- 26

85. Which one of the following statements is correct with respect to the original jurisdiction of
the Supreme Court in India?
(a) The Supreme Court has exclusive original jurisdiction to hear disputes between the Centre
and any State by way of appeal.
(b) The original jurisdiction of the Supreme Court does not extend to a dispute arising out of any
pre-Constitution treaty or agreement.
(c) The Indian Supreme Court can adjudicate matters related to inter-state water disputes under
its original jurisdiction.
(d) According to Article 131, the Supreme Court can entertain any matter related to a violation
of the fundamental rights of any Indian citizen by the state agencies.
Answer: B
Explanation:
Option (a) and (d) are incorrect: The Supreme Court has exclusive original jurisdiction, under
Article 131 of the Constitution, over federal cases. Here ‘exclusive’ means that no other Court
in India other than the Supreme Court can decide such disputes and ‘original’ means, the power
to hear such disputes in the first instance, not by way of appeal. These disputes are-
 Between the Centre and one or more states; or
 Between the Centre and any state or states on one side and one or more other states on the
other side; or
 Between two or more states.
Moreover, with regard to the exclusive original jurisdiction of the Supreme Court, two points
should be noted. First, the dispute must involve a question (whether of law or fact) on which the
existence or extent of a legal right depends. Thus, the questions of political nature are excluded
from it. Second, any suit brought before the Supreme Court by a private citizen against the
Centre or a State cannot be entertained under this.
Option (b) is correct but Option (c) is incorrect: The original jurisdiction of the Supreme Court,
under Article 131 of the Constitution, does not extend to the following:
 A dispute arising out of any pre-Constitution treaty, agreement, covenant, engagement,
sanad or other similar instrument.
 A dispute arising out of any treaty, agreement, etc., which specifically provides that the said
jurisdiction does not extent to such a dispute.
 Inter-state water disputes.
 Matters referred to the Finance Commission.
 Adjustment of certain expenses and pensions between the Centre and the states.
 Ordinary dispute of Commercial nature between the Centre and the states.
 Recovery of damages by a state against the Centre.

Question Hack: Option (a) says “by way of appeal” so something by appeal can never be exclusive
hence Option (a) can be eliminated.

Source: Indian Polity by Laxmikanth, 6th edition, Chapter- 26

86. Which one of the following statements with reference to the National Judicial Appointments
Commission (NJAC) is not correct?
(a) It was inserted in the Constitution through the 99th Constitutional Amendment Act of
2014.
(b) Article 124A contains provisions for the composition of the NJAC.
(c) The Union Minister of Law and Justice is the ex-officio chairman of NJAC.
(d) The Constitutional Amendment providing for the NJAC was struck down by the Supreme
Court in the case of Supreme Court Advocates on-Record Association and another Vs
Union of India case.
Answer: C
Explanation:
Option (a) is correct: National Judicial Appointments Commission (NJAC) was inserted in the
Constitution through the 99th Constitutional Amendment Act of 2014. The commission was
formed to:
a) recommend persons for appointment as Chief Justice of India, Judges of the Supreme
Court, Chief Justices of High Courts and other Judges of High Courts;
b) recommend transfer of Chief Justices and other Judges of High Courts from one High
Court to any other High Court; and
c) ensure that the person recommended is of ability and integrity.
Option (b) is correct: A new article, Article 124A, (which provides for the composition of the
NJAC) was to be inserted into the Constitution.
Option (c) is incorrect: According to the 99th Constitutional Amendment Act of 2014, NJAC was
supposed to consist of the following members:
 Chief Justice of India (Chairperson, ex officio)
 Two other senior judges of the Supreme Court next to the Chief Justice of India – ex officio
 The Union Minister of Law and Justice, ex-officio
 Two eminent persons (to be nominated by a committee consisting of the Chief Justice of
India, Prime Minister of India and the Leader of opposition in the Lok Sabha or where
there is no such Leader of Opposition, then, the Leader of single largest Opposition Party
in Lok Sabha), provided that of the two eminent persons, one person would be from the
Scheduled Castes or Scheduled Tribes or OBC or minority communities or a woman. The
eminent persons shall be nominated for a period of three years and shall not be eligible
for re-nomination.
Option (d) is correct: The 99th Constitutional Amendment Act providing for the NJAC has been
struck down by the Supreme Court in the case of Supreme Court Advocates on-Record
Association and another Vs Union of India in its judgment dated 16-10-2015.
Sources: Indian polity by M Laxmikanth, 6th Edition, Chapter-26.
87. Which one of the following statements is correct about the Supreme court’s power to review
under Article 137?
(a) It can examine the constitutionality of legislative enactments and executive orders of the
Central and state governments.
(b) It can review its own judgments and orders.
(c) It can review all the judgments and orders passed by the High Court through appeal.
(d) It can review all international laws and treaties of which India is a signatory.

Answer: B
Explanation:
Statement (a) is incorrect: Judicial review is the power of the Supreme Court to examine the
constitutionality of legislative enactments and executive orders of both the Central and state
governments. On examination, if they are found to be violative of the Constitution (ultra-vires),
they can be declared illegal, unconstitutional, and invalid (null and void) by the Supreme Court.
Consequently, they cannot be enforced by the Government. This power of the Supreme Court
comes from a reading of Articles 13 and 32.
Statement (b) is correct: Article 137 of the Constitution of India grants the Supreme Court the
power to review any of its judgments or orders. This power is however subject to the Rules
made by the Supreme Court under Article 145, as well as the provisions of any law enacted by
parliament. Further, as per the Supreme Court Rules, 2013, a review Petition must be filed within
30 days from the judgment.
Statement (c) is incorrect: The Supreme Court has also a very wide appellate jurisdiction over
all Courts and Tribunals in India. Under appellate jurisdiction, the Supreme Court can hear the
cases delivered by High Court through appeal. This has been provided in the Constitution under
Articles 132, 133, 134 and 134A.
It may, at its discretion also grant special leave to appeal under Article 136 of the Constitution
from any judgment, sentence, or order in any cause or matter passed or made by any Court or
Tribunal in the territory of India.
Statement (d) is incorrect: The Indian judiciary has the power to interpret the obligation of
India in international law by adjudicating domestic cases concerning issues of international
law. The statement is incorrect because the judiciary has no role in reviewing any of the
international treaties though the President of India can seek the opinion of the Supreme Court
on any dispute arising on any treaty or agreement.
Source: Indian Polity by M Laxmikanth, 6th Edition, Chapter-26.

88. Under which Schedule of the Constitution of India can the transfer of tribal land to private parties for
mining be declared null and void?
(a) Third Schedule
(b) Fifth Schedule
(c) Ninth Schedule
(d) Twelfth Schedule
Answer: B
Explanation:
Option (b) is correct: Under 5th Schedule of the Constitution of India the transfer of tribal land to private
parties for mining can be declared null and void.At present, 10 States namely Andhra Pradesh,
Chhattisgarh, Gujarat, Himachal Pradesh, Jharkhand, Madhya Pradesh, Maharashtra, Odisha, Rajasthan
and Telangana have Fifth Schedule Areas. The Fifth Schedule of the Constitution deals with the
administration and control of Scheduled Areas as well as of Scheduled Tribes residing in any State other
than the States of Assam, Meghalaya, Tripura and Mizoram. SC Judgement: Samatha v. State of Andhra
Pradesh & Others (1997) the transfer of tribal land to private parties for mining or other associated
activities can be declared null and void under the fifth schedule

Source: UPSC CSE PYQ 2019

89. With respect to the Supreme Court in India, consider the following statements:
1. The Supreme Court may grant special leave to appeal against any judgement rendered by any
court, except a military court, at its discretion.
2. Article 21 of the Constitution requires the Supreme Court to hear every curative petition filed
by a convict.
3. The Supreme Court in India is not bound by its previous decision and can depart from it in the
interest of community welfare.
Which of the statements given above is/are correct:
(a) 1 and 2 only
(b) 1 and 3 only
(c) 3 only
(d) 2 and 3 only
Answer: B
Explanation:
Statement 1 is correct: While exercising its power under Article 136, the Supreme Court is
authorised to grant in its discretion special leave to appeal from any judgement in any matter
passed by any court or tribunal in the country (except military tribunal and court martial).
Statement 2 is incorrect: A curative petition may be filed after a review plea against the final
conviction is dismissed. It is meant to ensure there is no miscarriage of justice, and to prevent
abuse of process. Every curative petition is decided on the basis of principles laid down by the
Supreme Court in Rupa Ashok Hurra Vs Ashok Hurra & another, 2002. In this case the Supreme
Court has ruled that a curative petition can be entertained if the petitioner establishes there
was a violation of the principles of natural justice, and that he was not heard by the court
before passing an order. It will also be admitted where a judge failed to disclose facts that raise
the apprehension of bias. Moreover, the SC has held that curative petitions must be rare rather
than regular, and be entertained with circumspection. And a curative petition must be
accompanied by certification by a senior advocate, pointing out substantial grounds for
entertaining it. It must be first circulated to a bench of the three senior-most judges, and the
judges who passed the concerned judgment, if available. Only when a majority of the judges
conclude that the matter needs hearing should it be listed as far as possible, before the same
Bench. Moreover, the word ‘curative petition’ is not mention under any Article of the Indian
Constitution.
Statement 3 is correct: Under Article 137 of the Constitution, the Supreme Court has power to
review its own judgement or order. Thus, the Supreme Court is not bound by its previous
decision and can depart from it in the interest of justice or community welfare. In brief, the
Supreme Court is a self-correcting agency. For example, in the Kesavananda Bharati case (1973),
the Supreme Court departed from its previous judgement in the Golak Nath case (1967).

Sources:
Indian Polity by Laxmikanth, 6th edition, Chapter- 26;
https://indianexpress.com/article/what-is/what-is-curative-petition-december-2012-gangrape-
case-sc-6204629/

90. Which of the following statements is/are correct?


1. Both the Gram Nyayalaya and the Lok Adalat are empowered to deal with any criminal and
civil cases at the village level.
2. The judgement passed by the Lok Adalat is final and binding on the parties, while the order
passed by the Gram Nyayalaya is considered a decree and not final in nature.
Select the correct answer using the code given below:
(a) 1 only
(b) 2 only
(c) Both 1 and 2
(d) Neither 1 nor 2

Answer: B
Explanation:
Statement 1 is incorrect: According to the Gram Nyayalayas Act, 2008, the Gram Nyayalaya shall
be a mobile court and shall exercise the powers of both Criminal and Civil Courts. The Gram
Nyayalaya shall try criminal cases, civil suits, claims or disputes which are specified in the First
Schedule and the Second Schedule to the Act thus not every cases. Whereas, the Lok Adalat
can deal with not only the cases pending before a court but also with the disputes at pre-
litigation stage including those Criminal cases which are compoundable offences only. In other
words, the offences which are non- compoundable under any law fall outside the purview of the
Lok Adalat.
Statement 2 is correct: The institution of Lok Adalat has been given statutory status under the
Legal Services Authorities Act, 1987. An award of a Lok Adalat shall be deemed to be a decree of
a Civil Court or an order of any other court. Every award made by a Lok Adalat shall be final and
binding on all the parties to the dispute. No appeal shall lie to any court against the award of
the Lok Adalat. The Gram Nyayalaya Act 2008 has mandated that the Gram Nyayalaya must try
to settle the disputes as far as possible by bringing about conciliation between the parties and
for this purpose, it shall make use of the conciliators to be appointed for this purpose. The
judgment and order passed by the Gram Nyayalaya shall be deemed to be a decree and to avoid
delay in its execution, the Gram Nyayalaya shall follow summary procedure for its execution. An
appeal in criminal cases shall lie to the Court of Session, which shall be heard and disposed of
within a period of six months from the date of filing of such appeal. And an appeal in civil cases
shall lie to the District Court, which shall be heard and disposed of within a period of six months
from the date of filing of the appeal.

Source: Indian Polity by Laxmikanth, 6th edition, Chapter- 36

91. With reference to the Supreme Court of India, consider the following statements:
1. The Constitution authorises the President to appoint places other than Delhi as seat of
the Supreme Court.
2. The Constitutional cases or references made by the President under Article 143 are
decided by a Bench consisting of at least five judges.
3. All proceedings in the Supreme court should be in English, but petitions can be allowed in
Hindi along with English.
Which of the statements given above is/are correct?
(a) 1 and 2 only
(b) 2 only
(c) 2 and 3 only
(d) 1 and 3 only

Answer: B
Explanation:
Statement 1 is incorrect: The Constitution declares Delhi as the seat of the Supreme court. The
Constitution authorises the Chief Justice of India and not the President to appoint places other
than Delhi as seat of the Supreme Court. But yes, he/she can take the decision in this regard only
with the approval of the President.
Statement 2 is correct: The Constitutional cases or references made by the President under
Article 143 are decided by a Bench consisting of at least five judges. All other cases are decided
by single benches and division benches. These judgments are delivered by the open court. All
judgments are by majority vote but if differing, then judges can give dissenting judgments or
opinions.
Statement 3 is incorrect: Unless the Parliament provides otherwise, all proceedings of the
Supreme Court should be in English. And as the Parliament has not made any provision for the
use of Hindi, So Supreme court allows only those petitions or appeals which are in English only.
Source: Indian polity by M Laxmikanth, 6th Edition, Chapter-26.

92. Consider the following statements regarding the Judges of the High Court:
1. They are appointed by the Governor but serve at the pleasure of the President.
2. They can be removed in the same manner and on the same grounds as a judge of the
Supreme Court.
3. A distinguished jurist can be appointed as a Supreme Court Judge but not as a Judge of a
High Court.
Which of the statements given above is/are correct?
(a) 1 and 2 only
(b) 2 and 3 only
(c) 3 only
(d) 1, 2 and 3

Answer: B
Explanation:
Statement 1 is incorrect: The judges of a high court are appointed by the President. The chief
justice is appointed by the President after consultation with the chief justice of India and the
governor of the state concerned. For appointment of other judges, the chief justice of the
concerned high court is also consulted.
They enjoy security of tenure (although no fixed tenure provided in Constitution) and hold
office until they attain the age of 62 years.
Statement 2 is correct: A judge of a high court can be removed from his office by an order of the
President. The President can issue the removal order only after an address by the Parliament has
been presented to him in the same session for such removal. The address must be supported by
a special majority of each House of Parliament (i.e., a majority of the total membership of that
House and majority of not less than two-thirds of the members of that House present and voting).
The grounds of removal are two–proved misbehaviour or incapacity. Thus, a judge of a high court
can be removed in the same manner and on the same grounds as a judge of the Supreme Court.
Statement 3 is correct: A person to be appointed as a judge of a high court, should have the
following qualifications:
 He should be a citizen of India.
 He should have held a judicial office in the territory of India for ten years; or
 He should have been an advocate of a high court (or high courts in succession) for ten
years.
The Constitution makes no provision for the appointment of a distinguished jurist as a judge of
a High court. But a distinguished jurist can become a judge of Supreme court.
Source: Indian Polity by M Laxmikanth, 6th Edition, Chapter-34.

93. Consider the following statements with respect to the Legislative Councils:
1. The members of the legislative council are partly nominated and partly indirectly elected.
2. The maximum strength of the council is fixed at one-third of the total strength of the
assembly and the minimum strength is fixed at 40.
3. 1/12th of the members is elected by teachers and graduates of three years standing each.
Which of the statements given above is/are correct?
(a) 1 and 2 only
(b) 2 and 3 only
(c) 3 only
(d) 1, 2 and 3

Answer: D
Explanation:

Statement 2 is correct: Unlike the members of the legislative assembly, the members of the
legislative council are indirectly elected. The maximum strength of the council is fixed at one-
third of the total strength of the assembly and the minimum strength is fixed at 40. It means
that the size of the council depends on the size of the assembly of the concerned state.
This is done to ensure the predominance of the directly elected House (assembly) in the
legislative affairs of the state. Though the Constitution has fixed the maximum and the minimum
limits, the actual strength of a Council is fixed by Parliament.
Statement 1 and 3 are correct: Manner of Election: Of the total number of members of a
legislative council:
1. 1/3 are elected by the members of local bodies in the state like municipalities, district
boards, etc.,
2. 1/12 are elected by graduates of three years standing and residing within the state,
3. 1/12 are elected by teachers of three years standing in the state, not lower in standard
than secondary school,
4. 1/3 are elected by the members of the legislative assembly of the state from amongst
persons who are not members of the assembly, and
5. the remainder are nominated by the governor from amongst persons who have a special
knowledge or practical experience of literature, science, art, cooperative movement and
social service.
Thus, 5/6 of the total number of members of a legislative council are indirectly elected and 1/6
are nominated by the governor. The members are elected in accordance with the system of
proportional representation by means of a single transferable vote. The bonafide or propriety of
the governor’s nomination in any case cannot be challenged in the courts.
Source: Indian Polity by M. Laxmikanth, 6th Edition, Chapter-33

94. The inherent power of the Supreme Court to pass orders or decrees in order to do complete
justice stems from which of these Articles?
(a) Article 136
(b) Article 137
(c) Article 142
(d) Article 143

Answer: C
Explanation:
Option (c) is correct: The Constitution of India through Article 142 empowers the Supreme
Court of India for passing such “decree or order as may be necessary for doing complete justice
among the parties”. In our Constitution, Article-142 provides that in the process to do complete
justice, Supreme Court has the power to pass any Judgment, decree or order as is necessary. Any
such order made by Supreme Court will be enforceable all over the territory of India. The
instrument of enforcement of such order shall be determined by the law that has been passed
by parliament regarding it. If there is no existing law passed by parliament to enforce the order
passed by SC, then a President’s order shall determine it by prescription. In this Article, Supreme
Court can also give order for the attendance of a person, ask for discovery and production of
documents, and can also investigate and punish for its contempt.
These powers have been given to Supreme Court because the Supreme Court protect the other
court from any type of jurisdictional difficulties and does not harm the court to do justice among
the parties before it.
Sources:
https://blog.ipleaders.in/inherent-powers-supreme-court-high-court-civil-court-criminal-court/
Indian Polity by M Laxmikanth, 6th Edition, Chapter-26

95. India generates 15 million tonnes of plastic waste every year but only one fourth of this is recycled
due to lack of a functioning solid waste management system. This leads to burden on the landfills and
poor socio-economic conditions.
With reference to the Plastic Waste Management, consider the following statements:
1. Under Plastic Waste Management Rules, 2016, it is the primary responsibility Producers, Importers
and Brand Owners to collect used multi-layered plastic sachet or pouches or packaging introduced
by them in the market.
2. Extended Producer Responsibility is applicable only on post-consumer plastic packaging waste.
3. As per EPR guidelines, 2022, Extended Producer Responsibility guidelines allow for the sale and
purchase of surplus extended producer responsibility certificates.
Which of the statements given above are correct?
(a) 1 and 2 only
(b) 2 and 3 only
(c) 1 and 3 only
(d) 1, 2 and 3

Answer: C
Explanation:
Statement 1 is correct: The Plastic Waste Management Rules, 2016, mandate the generators of plastic
waste to take steps to minimize generation of plastic waste, not to litter the plastic waste, ensure
segregated storage of waste at source and hand over segregated waste in accordance with rules. The
rules also mandate the responsibilities of local bodies, gram panchayats, waste generators, retailers
and street vendors to manage plastic waste. The Plastic Waste Management Rules, 2016 cast
Extended Producer Responsibility on Producer, Importer, and Brand Owner. Primary responsibility
for collection of used multi-layered plastic sachet or pouches or packaging is of Producers, Importers
and Brand Owners who introduce the products in the market. They need to establish a system for
collecting back the plastic waste generated due to their products.
Statement 2 is incorrect: Extended Producers Responsibility (EPR) regime is under implementation in
Plastic Waste Management Rules, 2016, Under 2016 Rules, Extended Producer Responsibility shall
be applicable to both pre-consumer and post-consumer plastic packaging waste.
Statement 3 is correct: In order to streamline implementation process of EPR, the Ministry of
Environment, Forest and Climate Change, Government of India, in its fourth Amendment to the Plastic
Waste Management Rules, 2022, notified ‘Guidelines on Extended Producer Responsibility for Plastic
Packaging’ in the Schedule II of the Rules. These guidelines for the first time, proposed the
generation of surplus Extended Producer Responsibility certificates. They allow sale and purchase
of surplus Extended Producer Responsibility certificates to carry forward and offsetting against
previous year Extended Producer Responsibility targets and obligations.

Source: https://eprplastic.cpcb.gov.in/#/plastic/home
96. With reference to Dugong, consider the following statements:
1. They are the largest herbivorous marine mammals in the world thriving primarily on seagrass
beds.
2. They are found throughout the warm latitudes of Indian and western Pacific Ocean.
3. They are protected under Schedule 1 of the Wild Life (Protection) Act, 1972.
4. India’s first Dugong Conservation Reserve has been established in Odisha near Chilika Lake.
Which of the statements given above is/are correct?
(a) 1, 2 and 3 only
(b) 1, 2 and 4 only
(c) 1, 3 and 4 only
(d) 2, 3 and 4 only

Answer: A
Explanation:
Statement 1 is correct: Dugongs are the largest herbivorous marine mammals in the world thriving
primarily on seagrass beds, a major carbon sink of the oceans. The dugong is strictly a marine mammal.
Commonly known as ‘sea cows’. It primarily grazes on sea grasses and therefore spends most of its time
in sea grass beds. Dugongs are threatened by sea grass habitat loss or degradation because of coastal
development or industrial activities that cause water pollution. If there is not enough sea grass to eat
then the dugong does not breed normally. This makes the conservation of their shallow water marine
habitat very important.
Statement 2 is correct: The dugong is a species of sea cow found throughout the warm latitudes of the
Indian and western Pacific Oceans. It is the only member of the Dugongidae family, and its closest living
relatives are the manatees. Dugongs graze on sea grasses in shallow coastal waters of the Indian and
western Pacific Oceans.
Statement 3 is correct: The population of Dugongs is on the decline due to habitat. Its protection status
is:
 Wild Life (Protection) Act, 1972 – Schedule 1
 IUCN Red List – Vulnerable
 CITES – Appendix 1
Statement 4 is incorrect: India’s first ‘Dugong Conservation Reserve’ was notified in Palk Bay (off the
coast of Tamil Nadu). The move was aimed at protecting the Dugong species and its marine habitats.
Conserving dugongs will help to protect and improve seagrass beds and sequestering more atmospheric
carbon. Seagrass beds are also the breeding and feeding grounds for many commercially valuable fishes
and marine fauna. Hence, thousands of fisher families directly depend on dugong habitats for their
income.

Source:
https://www.worldwildlife.org/species/dugong
https://indianexpress.com/article/cities/chennai/indias-first-dugong-conservation-reserve-notified-in-
tamil-nadu-says-govt-8165098/

97. With reference to the ‘Seabed 2030 Project’, which of the following statements are correct?
1. The project was launched at the United Nations Ocean Conference in 2017.
2. Under this project, UNESCO has set the target to map 100% of the seabed by 2030.
3. This project is a flagship programme of the Ocean decade initiative by the UN.
4. The project is aligned with the UN Sustainable Development Goal 17, which aims at strengthening
the means of implementation and revitalise the Global partnership for Sustainable development.
Select the correct answer using the code given below:
(a) 1 and 3 only
(b) 1, 2 and 4 only
(c) 4 only
(d) 2 and 3 only

Answer: A
Explanation:
Statement 1 is correct: Seabed 2030 is a collaborative project between the Nippon Foundation of
Japan and the General Bathymetric Chart of the Oceans (GEBCO). It aims to bring together all available
bathymetric data to produce the definitive map of the world ocean floor by 2030 and make it available to
all. The project was launched at the United Nations (UN) Ocean Conference in June 2017.
Statement 2 is incorrect: When Seabed 2030 was launched five years ago, only 6% of the seabed was
mapped to modern standards, today the figure stands at 20%. UNESCO, in the conference, has
announced that at least 80% of the seabed will be mapped by 2030, compared to 20% currently, in
collaboration with other UN bodies and with the support of its member states and the private sector. The
potential to accelerate seabed mapping is further enhanced by recent technological innovations. While
sonar technology has been used since the 1960s to scan the seabed, sonars now have multiple beams
enabling them to measure water height at several points and in several directions at once, which save
time considerably.
Statement 3 is correct: Seabed 2030 is a flagship programme of Ocean Decade, a major initiative by the
UN to mobilise governments, the private sector, scientists, and civil society to co-design and co-deliver
transformative knowledge-led actions to reverse the decline in the health of the ocean and make a
change in the sustainable management of the marine environment. IOC-UNESCO is the UN body tasked
with leading the implementation of the Ocean Decade.
Statement 4 is incorrect: Seabed data is foundational for determining how the ocean works. Beyond
navigation, the shape of the ocean floor plays a big role in the movement of ocean debris and pollution
on its surfaces and currents. Knowledge about the depths can provide insights into sustainable fisheries
management. Ocean acidification is also directly linked to depth; some areas may experience more
chemical change and be less able to sustain healthy ecosystems than others. Thus, Seabed 2030 is aligned
with the UN's Sustainable Development Goal 14, which aims at to conserve and sustainably use the
oceans, seas and marine resources.

Source:
https://seabed2030.org/aboutus#:~:text=Seabed%202030%20is%20a%20collaborative,make%20it%20a
vailable%20to%20all.

98. ‘The Ministry of Health and Family Welfare, Government of India has formulated the Ayushman Bharat
Digital Mission with the aim to provide the necessary support for the integration of digital health
infrastructure in the country.’ Which of the statements given below is/are correct with reference to
Ayushman Bharat Digital Mission?
1. Ayushman Bharat Digital Mission stems from the National Health Policy, 2017.
2. A personal health record is an electronic application which help patients to maintain and manage
their health information.
3. Universal health coverage in an efficient, accessible, inclusive, affordable, timely and safe
manner is the ultimate vision of Ayushman Bharat Digital Mission.
Which of the statements given above is/are correct?
(a) 1 and 2 only
(b) 2 and 3 only
(c) 1 and 3 only
(d) 1, 2 and 3

Answer: D
Explanation:
Statement 1 is correct: The Ministry of Health and Family Welfare, Government of India has formulated
the Ayushman Bharat Digital Mission with the aim to provide the necessary support for the integration of
digital health infrastructure in the country. This visionary initiative, stemming from the National Health
Policy, 2017 intends to digitize healthcare in India. National Health Authority (NHA) has been entrusted
with the role of designing strategy, building technological infrastructure and implementation of Ayushman
Bharat Digital Mission.
Statement 2 is correct: Under the mission, a personal health record is an electronic application through
which patients can maintain and manage their health information (and that of others for whom they
are authorized) in a private, secure, and confidential environment. The citizens can get a longitudinal
view of the personal health record by creating and linking the health ID with the visited health care
facilities.
Statement 3 is correct: Its vision is to create a national digital health ecosystem that supports universal
health coverage in an efficient, accessible, inclusive, affordable, timely and safe manner. It also
envisions a seamless online platform through the provision of a wide range of data, information and
infrastructure services, duly leveraging open, interoperable, standards-based digital systems, and ensures
the security, confidentiality and privacy of health-related personal information.

Source: https://www.nhp.gov.in/national-digital-health-mission-(ndhm)_pg

99. The term ‘Doxxing’ often heard in news corresponds to which of the following activites?
(a) Tracking user’s browser and download history with the intent to display pop-up or banner
advertisements.
(b) Publishing others’ personal information on the internet with malicious intent.
(c) Tracking nearly everything a person does on their computer.
(d) Delivering mails disguised as legitimate message containing malicious links or attachments.

Answer: B
Explanation:
Option (a) is incorrect: Adware refers to tracking user’s browser and download history with the intent to
display pop-up or banner advertisements. They lure the users into making purchases.
Option (b) is correct: Doxxing is the act of publishing others’ personal information on the internet with
malicious intent which can make them victims of harassment and cyber-attacks. It can impact an
individual emotionally and mentally. Meta’s oversight board suggested Facebook to release stringent
Doxxing rules in February, 2022.
Option (c) is incorrect: Tracking nearly everything a person does on their computer is an act known as
Keyloggers. This includes emails, opened webpages, programs and keystrokes.
Option (d) is incorrect: Delivering mails disguised as legitimate message containing malicious links or
attachments is known as Phishing attack. It delivers the malware executable file to unsuspecting users.

Source: https://www.bbc.com/news/technology-60302683

100. With reference to the India-UAE relations, consider the following statements:
1. In FY 2021-22, the United Arab Emirates is India’s third largest trade partner.
2. UAE is the largest export destination of India.
3. Indians constitute the second largest ethnic community in UAE.
Which of the statements given above is/are correct?
(a) 1 only
(b) 1 and 2 only
(c) 3 only
(d) 1, 2 and 3

Answer: A
Explanation:
Option (a) is correct: India and UAE have shared trade links through the centuries. The trade, which was
dominated by traditional items such as dates, pearl and fishes, underwent a sharp change after the
discovery of oil in UAE. Growing India-UAE economic and commercial relations contribute to the stability
and strength of a rapidly diversifying and deepening bilateral relationship between the two countries.
Both sides are striving to further strengthen these ties for mutual benefits. India-UAE trade valued at US$
59 billion in 2019-20 making UAE, India's third largest trading partner for the year 2019-20 after China
and US. This trend continues in 2021-22 as well. Moreover, UAE is the second largest export destination
of India (after US) with an amount of nearly US$ 29 billion for the year 2019-20. For UAE, India is the
second largest trading partner for the year 2019 with an amount of around US$ 41.43 billion (non-oil
trade). Moreover, Indians constitute the largest ethnic community in India, with about 30% of
population constituting of Indians in all walks of services.

Source: https://www.indembassyuae.gov.in/bilateral-eco-com-relation.php
Polity 2

1. भारतीय संसद क वधायी शि त के संबंध म न न ल खत कथन पर वचार कर:


1. संसद वारा बनाए गए कानून द ु नया के कसी भी ह से म भारतीय
नाग रक क संप य पर लागू होते ह।
2. रा य वधा यका आपरा धक कानून और या को छोड़कर समवत सूची
म उि ल खत कसी भी मामले के संबंध म कानून बना सकती है ।
3. समवत सूची के वषय पर बनाए गए कानून के संबंध म क य और
रा य वधान के बीच संघष क ि थ त म, क य कानून भावी होता है ।
ऊपर दए गए कथन म से कौन-से सह ह?
(a) केवल 1 और 2
(b) केवल 1 और 3
(c) केवल 2 और 3
(d) 1, 2 और 3

उ र: b
या या:
कथन 1 सह है : संसद भारत के पूरे े या उसके कसी भी ह से के लए कानून बना
सकती है । भारत के े म रा य, क शा सत दे श और भारत के े म कुछ समय के
लए शा मल कोई अ य े शा मल है । केवल संसद ह 'अ े ीय कानून' बना सकती है
अतः संसद के कानून द ु नया के कसी भी ह से म भारतीय नाग रक और उनक संप
पर भी लागू होते ह।
कथन 2 गलत है: संसद और रा य वधानमंडल दोन समवत सच
ू ी म सच
ू ीब ध कसी
भी वषय के संबंध म कानन
ू बना सकते ह। इस सच
ू ी म वतमान म आपरा धक कानन

और या , स वल या, ववाह और तलाक, जनसं या नयं ण और प रवार नयोजन,
बजल , म क याण, आ थक और सामािजक नयोजन, स, समाचार प , कताब और
ं टंग ेस जैसे 52 वषय शा मल है (मल
ू प से 47 वषय थे ) ह |
कथन 3 सह है : अनस
ु च
ू ी VII के तहत सं वधान व भ न वषय को नधा रत करता है
िजन पर संसद और रा य मशः सूची I और सूची II के तहत कानून बना सकते ह। सूची
III के तहत, िजसे समवत सच
ू ी के प म भी जाना जाता है , संसद और रा य दोन के
पास कानून बनाने क शि त है ।
अनु छे द 254 के तहत सं वधान म यह ावधान है क य द संसद और रा य दोन
समवत सूची के कसी वषय पर कानून बनाते ह और य द कानून ऐसा ह जो एक दस
ू रे
के असंगत है तो संसद वारा बनाया गया कानून भावी होगा और रा य वारा बनाया
गया कानन
ू क य कानन
ू के तकूल होने क सीमा तक शू य माना जाएगा।
इस अनु छे द के अनुसार सूची III अथात समवत सूची म न हत मामल पर कानून
बनाने क संसद क शि त सव च है । अनु छे द कसी भी व ध को अ धभावी भाव दे ता
है िजसे अ ध नय मत करने के लए संसद स म है और िजसे इसके वारा अ ध नय मत
कया गया है ।
ोत:
एम. ल मीकांत वारा भारतीय राजनी त, छठा सं करण - अ याय- 14।
https://www.legalserviceindia। com/legal/article-964-the- स धांत-का-repugnancy-
in-the- indian-constitution.html

2. न न ल खत कथन पर वचार क िजएः


1. संसद और रा य वधानमंडल दोन ह अव श ट वषय के संबंध म कानून
बना सकते ह।
2. भारत सरकार अ ध नयम 1935 के तहत, संघीय वधा यका को अव श ट वषय
पर कानून बनाने क वशेष शि त दान क गई थी।
3. समवत सूची के वषय म बने कानून के संबंध म क और रा य के कानून
के बीच संघष क ि थ त म कसी भी प रि थ त म रा य के कानून क य कानून
पर हावी नह ं हो सकते ह।
ऊपर दए गए कथन म से कौन-सा/से सह नह ं है /ह ?
(a) केवल 1 और 3
(b) केवल 2
(c) केवल 3
(d) 1, 2, और 3

उ र: d
या या:
कथन 1 गलत है : रा य मह व के मामले और दे श भर म कानून क एक पता क
आव यकता वाले वषयो को संघ सूची म शा मल कया गया है । े ीय और थानीय
मह व के मामले और वे मामले जो हत क व वधता क अनुम त दे ते ह, रा य सूची म
न द ट ह। िजन वषयो पर परू े दे श म कानन
ू क एक पता वांछनीय है ले कन आव यक
नह ं है , उ ह समवत सच
ू ी म शा मल कया गया है ।
सच
ू ी I के अनु छे द 248 और वि ट 97 के अनस
ु ार, वह अव श ट वषय (अथात, तीन
सू चय म से कसी म सच
ू ीब ध नह ं ह) के संबंध म कानन
ू बनाने क शि त वशेष प
से संसद म न हत है । कानन
ू क इस अव श ट शि त म अव श ट कर लगाने क शि त
शा मल है ।
कथन 2 गलत है : अमे रका म, केवल संघीय सरकार क शि तय को सं वधान म व णत
कया गया है अव श ट शि तयां रा य के पास होती ह। दस
ू र ओर, कनाडा म, दोहर
सूची है - संघीय और ांतीय, और अव श ट शि तयाँ क म न हत ह। 1935 के भारत
शासन अ ध नयम म तीन सूची का ावधान कया था जैस,े संघीय, ांतीय और
समवत । वतमान सं वधान इस अ ध नयम के ा प का पालन करता है ले कन एक
अंतर के साथ, इस अ ध नयम के तहत, अव श ट शि तयां न तो संघीय वधा यका को
और न ह ांतीय वधा यका को बि क भारत के गवनर जनरल को द गई थीं। इस
संबंध म, भारत कनाडा का अनुसरण करता है ।
कथन 3 गलत है : समवत सूची म उि ल खत वषय पर क य कानून और रा य के
कानून के बीच संघष के मामले म, क य कानून रा य के कानन
ू पर हावी होता
है । ले कन, एक अपवाद है । य द रा य का कानून रा प त के वचार के लए सुर त
रखा गया है और उसक सहम त ा त कर ल गई है , तो उस रा य म रा य का कानून
लागू होता है। ले कन, संसद इस बात के लए स म है क वह बाद म उसी वषय
पर एक कानून बनाकर ऐसे कानून को ओवरराइड कर सकती है
ोत: एम. ल मीकांत वारा भारतीय राजनी त, छठा सं करण- अ याय- 14।

3. न न ल खत कथन पर वचार क िजएः


1. भारत का रा प त संसद का स ऐसी जगह पर बुला सकता है िजसे वह ठ क समझे।
2. भारत का सं वधान एक वष म संसद के तीन स का ावधान करता है ले कन तीन स
का संचालन करना अ नवाय नह ं है
3. एक वष म संसद क बैठक के लए आव यक यूनतम दन क कोई सं या नह ं है
ऊपर दए गए कथन म से कौन सा/से सह है /ह?
(a) केवल 1
(b) केवल 2
(c) केवल 1 और 3
(d) केवल 2 और 3

उ र: C
या या:
कथन 1 सह है : अनु छे द 85 कहता है क रा प त संसद का स "ऐसे समय और थान पर जैसा
वह उ चत समझे" बुला सकता है । सरकार क सफा रश पर एक स बुलाया जा सकता है , जो
इसक तार ख और अव ध तय करता है । ले कन, संसद के दो स के बीच अ धकतम अंतराल छह
मह ने से अ धक नह ं हो सकता है जो इं गत करता है क संसद को वष म कम से कम दो बार
मलना चा हए।
कथन 2 गलत है : आमतौर पर एक वष म तीन स होते ह, स मेलन वारा और सं वधान वारा
नह ं, संसद एक वष म तीन स के लए मलती है :
• बजट स (फरवर से मई);
• मानसून स (जुलाई से अग त); तथा
• शीतकाल न स (नवंबर से दसंबर)।
कथन 3 सह है : सं वधान यह न द ट नह ं करता है क संसद क बैठक कब या कतने दन के
लए होनी चा हए।
अनु छे द 85 म केवल यह आव यक है क संसद के दो स के बीच छह मह ने से अ धक का
अंतराल नह ं होना चा हए। एक स के दौरान, सदन हर दन काय करने के लए बैठक करता है ।
एक सदन के स ावसान और एक नए स म उसके पुन: संयोजन के बीच क अव ध को 'अवकाश'
कहा जाता है ।

ोत: यूपीएससी सीएसई पीवाई यू 2020

4. सरकार ने 1996 म पंचायत व तार अनस


ु ू चत े (पेसा) अ ध नयम को अ ध नय मत
कया। न न ल खत म से कस एक को इसके उ दे य के प म पहचाना नह ं गया है ?
(a) वशासन दान करने के लए
(b) पारं प रक अ धकार को मा यता दे ने के लए
(c) आ दवासी े म वाय े बनाने के लए
(d) आ दवासी लोग को शोषण से मु त करने के लए

उ र: C
या या:
वक प (c) सह है : पेसा अ ध नयम 1996 म "पंचायत से अनस
ु ू चत े म सं वधान के भाग IX
के ावधान के व तार के लए दान करने के लए" अ ध नय मत कया गया था।
पेसा अ ध नयम के तहत, अनस
ु ू चत े वे ह िज ह अनु छे द 244 (1) म संद भत कया गया है ,
जो कहता है क पांचवीं अनस
ु च
ू ी के ावधान असम, मेघालय, परु ा और मजोरम के अलावा अ य
रा य म अनस
ु ू चत े और अनस
ु ू चत जनजा तय पर लागू ह गे। पांचवीं अनस
ु च
ू ी इन े के
लए वशेष ावधान क एक ख
ंृ ला दान करती है । अ ध नयम के पीछे मु य तक ाम सभा क
स य भागीदार के साथ आ दवासी आबाद को शोषण से बचाना है । पेसा अ ध नयम के कुछ
अ य उ दे य ह:
• जनजातीय आबाद के बड़े ह से के लए वशासन दान करना
• सहभागी लोकतं के साथ ाम शासन रखना और ाम सभा को सभी ग त व धय का क
बनाना
• पारं प रक अ धकार को मा यता दे ना और आ दवासी समुदाय क परं पराओं और र त-
रवाज क र ा करना और उनका संर ण करना।
• जनजातीय आव यकताओं के अनुकूल व श ट शि तय के साथ पंचायत को उपयु त
तर पर सश त बनाना
• जनजातीय े म वाय े बनाना पेसा अ ध नयम का उ दे य नह ं है ।

ोत: यप
ू ीएससी सीएसई पीवाई यू 2013

5. न न ल खत यु म पर वचार क िजएः
सं वधान का ावधान
अनु छे द
1. अनु छे द 249 संसद रा य सूची के वषय पर रा य हत म
कानून बना सकती है य द रा य सभा उपि थत और
मतदान करने वाले सद य के 2/3 बहुमत से ताव
पा रत करती है ।
2. अनु छे द 250 य द दो या दो से अ धक रा य वारा अनुरोध कया
जाता है , तो संसद केवल उन रा य के संबंध म
रा य सूची पर कानून बना सकती है ।
3. अनु छे द 252 य द आपातकाल क उ घोषणा चल रह हो तो संसद
रा य सच
ू ी के कसी भी वषय के संबंध म कानन

बना सकती है ।
4. अनु छे द 253 अंतररा य सं धय को लागू करने के लए संसद
रा य सूची के कसी भी वषय पर कानून बना
सकती है ।
ऊपर दए गए कतने जोड़े सह सुमे लत ह/ह?
(a) केवल एक जोड़ी
(b) केवल दो जोड़े
(c) केवल तीन जोड़े
(d) सभी चार जोड़े

उ र: b
या या:
जोड़ी 1 सह सुमे लत है : अनु छे द 249 म कहा गया है क य द रा य सभा यह घोषणा
करती है क रा य हत म यह आव यक है क संसद व तु एवं सेवा कर या रा य सच
ू ी
के कसी वषय के संबंध म कानून बनाए , तो संसद उस वषय पर कानून बनाने के
लए स म हो जाती है । । इस तरह के ताव को उपि थत और मतदान करने वाले
वारा सम थत होना चा हए
सद य के दो- तहाई (2/3 rd ) . संक प एक वष के लए लागू रहता
है ; इसे बार बार नवीनीकृत कया जा सकता है ले कन एक बार म एक वष से अ धक
समय के लए नह ं। संक प के समा त होने के छह मह ने बाद कानून भावी नह ं होते
ह। यह ावधान उसी वषय पर कानून बनाने के लए रा य वधा यका क शि त को
तबं धत नह ं करता है । ले कन, एक रा य के कानून और एक संसद य कानून के बीच
असंग त के मामले म, बाद वाला मा य होगा।
जोड़ी 2 का गलत मलान कया गया है : अनु छे द 250 म कहा गया है क संसद व तु
और सेवा कर या रा य सूची के वषय के संबंध म कानून बनाने क शि त ा त
करती है , जब रा य आपातकाल क उ घोषणा चल रह है ।ले कन आपातकाल समा त
होने के छह मह ने बाद कानून नि य हो जाते ह। यहां उसी वषय पर कानून बनाने
के लए रा य वधा यका क शि त तबं धत नह ं है । ले कन, एक रा य के कानून और
संसद य कानून के बीच ग तरोध होने पर , संसद का कानून भावी होता है ।
जोड़ी 3 का गलत मलान कया गया है : अनु छे द 252 म कहा गया है क जब दो या दो
से अ धक रा य क वधा यका संसद से रा य सूची म कसी वषय पर कानून बनाने
का अनुरोध करते हुए ताव पा रत करती है , तो संसद उस वषय को व नय मत करने
के लए कानून बना सकती है । इस कार अ ध नय मत कानन ू केवल उन रा य पर लागू
होता है िज ह ने ताव पा रत कए ह। हालाँ क, कोई भी अ य रा य अपनी वधा यका
म इस आशय का ताव पा रत करके बाद म इसे अपना सकता है । इस तरह के कानून
को केवल संसद वारा संशो धत या नर त कया जा सकता है , न क संबं धत रा य के
वधानमंडल वारा। उपरो त ावधान के तहत एक ताव पा रत करने का भाव यह है
क संसद उस वषय के संबंध म कानन
ू बनाने म स म हो जाती है िजसके लए उसे
कानन
ू बनाने क कोई शि त नह ं है । दस
ू र ओर, रा य वधा यका को उस वषय के
संबंध म कानन
ू बनाने क शि त समा त हो जाती है । संक प उस वषय के संबंध म
रा य वधा यका क शि त के याग या समपण के प म काय करता है और इसे परू
तरह से संसद के हाथ म रखा जाता है , जो अकेले ह इसके संबंध म कानून बना सकती
है । उपरो त ावधान के तहत पा रत कानून के कुछ उदाहरण पुर कार त पधा
अ ध नयम, 1955 ह; व य जीवन (संर ण) अ ध नयम, 1972; जल ( दष
ू ण क रोकथाम
और नयं ण) अ ध नयम, 1974; शहर भू म (सीमा और व नयमन) अ ध नयम,
1976; और मानव अंग यारोपण अ ध नयम, 1994।
यु म 4 का सह मलान कया गया है : अनु छे द 253 म कहा गया है क संसद
अंतररा य सं धय , समझौत को लागू करने के लए रा य सच
ू ी के कसी भी वषय
पर कानून बना सकती है । यह ावधान क सरकार को अपने अंतररा य दा य व और
तब धताओं को पूरा करने म स म बनाता है । उपरो त ावधान के तहत अ ध नय मत
कानून के कुछ उदाहरण संयु त रा ( वशेषा धकार और तर ा) अ ध नयम, 1947
ह; िजनेवा क वशन ए ट, 1960; अपहरण नरोधी अ ध नयम, 1982 व पयावरण और
स(TRIPS) से संबं धत वधान।
ोत: एम. ल मीकांत वारा भारतीय राजनी त, छठा सं करण- अ याय- 14।

6. न न ल खत कथन पर वचार क िजएः


1. क रा य को संचार के उन साधन के रखरखाव के संबंध म नदश दे
सकता है िज ह रा य या सै य मह व का घो षत कया गया है ।
2. अनु छे द 365 अनु छे द 356 के तहत ज दबाजी म उठाये गए कसी भी
कठोर नणय को रोकने के लए एक न के प म काय करता है ।
ऊपर दए गए कथन म से कौन-सा/से सह है /ह?
(a) केवल 1
(b) केवल 2
(c) दोन 1 और 2
(d) न तो 1 और न ह 2

उ र: c
या या:
कथन 1 सह है : अनु छे द 256 के तहत, क को न न ल खत मामल म रा य को उनक
कायकार शि त के योग के संबंध म नदश दे ने का अ धकार है :
• रा य वारा संचार के साधन (रा य या सै य मह व के घो षत) का
नमाण और रखरखाव ;
• रा य के भीतर रे लवे क सरु ा के लए कए जाने वाले उपाय;
• रा य म भाषाई अ पसं यक समह
ू के ब च को श ा के ाथ मक तर
पर मातभ
ृ ाषा म श ा के लए पया त सु वधाएं उपल ध कराना; तथा
• रा य म अनस
ु ू चत जनजा तय के क याण के लए न द ट योजनाओं क
परे खा तैयार करना और उनका या वयन करना।
कथन 2 सह है : डॉ . बीआर अंबेडकर ने सं वधान सभा म अनु छे द 365 को शा मल करने
का समथन कया था य क इसने अनु छे द 256 और 257 के तहत नदश को पूरा करने
म वफलता होने पर संघ को कारवाई करने क शि त दान क थी । सभा ने पाया क
अनु छे द 365 म दान कया गया उपाय अनु छे द 356 क तल
ु ना म कम कठोर है ।
अनु छे द 365 का तधारण आव यक है य क यह अनु छे द 356 के तहत कठोर
कारवाई के लए कसी भी ज दबाजी के उपाय को रोकने के लए एक न के प म
काय करता है । अनु छे द 365 को बनाए रखने और इसके योग से पहले सतक
ि टकोण के लए आयोग क सफा रश को वीकार करने का नणय लया गया था।
• अनु छे द 356 रा प त को एक उ घोषणा जार करने का अ धकार दे ता है , य द
वह संतु ट है क ऐसी ि थ त उ प न हो गई है िजसम कसी रा य क सरकार
सं वधान के ावधान के अनुसार नह ं चल सकती है । रा प त रा य के रा यपाल क
रपोट पर या रा यपाल क रपोट के बना भी काय कर सकता है ।
• अनु छे द 365 कहता है क जब भी कोई रा य क के कसी नदश का पालन
करने या उसे लागू करने म वफल रहता है , तो रा प त के लए यह मानना वैध
होगा क ऐसी ि थ त उ प न हो गई है िजसम रा य क सरकार सं वधान के
ावधान उसके अनुसार नह ं चल सकती है ।
ोत:
एम. ल मीकांत वारा भारतीय राजनी त, छठा सं करण- अ याय- 14, 16।
https://archive.pib.gov.in/archive/releases98/lyr2003/
raug2003 / 28082003/r280820037। एचट एमएल

7. न न ल खत शहर थानीय नकाय पर वचार क िजये:


1. नगर नगम
2. छावनी बोड
3. नगर य े
4. नगर प रषद
5. अ धसू चत े समत
74व सं वधान संशोधन अ ध नयम, 1993 वारा था पत कए जाने के लए उपरो त म
से कौन-सा ावधान कया गया है ?
(a) केवल 1 और 4
(b) केवल 1, 4 और 5
(c) केवल 2, 3 और 5
(d) 1, 2, 3, 4 और 5

उ र: a
या या:
74व सं वधान संशोधन अ ध नयम म येक रा य म न न ल खत तीन कार क नगर
पा लकाओं के गठन का ावधान कया गया है ।
 एक सं मणकाल न े के लए एक नगर पंचायत (चाहे कसी भी नाम से
पुकारा जाए)।
 एक छोटे शहर े के लए एक नगरपा लका प रषद।
 एक बड़े शहर े के लए एक नगर नगम।
कथन 1 सह है : द ल , मुंबई, कोलकाता, है दराबाद, बगलोर और अ य जैसे बड़े शहर
के शासन के लए नगर नगम का ावधान कया गया है । वे रा य म संबं धत रा य
वधानसभाओं क वध वारा और क शा सत दे श म भारत क संसद क वध
वारा था पत कए जाते ह। एक रा य म सभी नगर नगम के लए एक समान
अ ध नयम या येक नगर नगम के लए एक पथ
ृ क अ ध नयम हो सकता है ।
कथन 2 गलत है : छावनी े म स वल जनसं या के शासन के लए एक छावनी
प रषद था पत करने का ावधान कया गया है । यह 2006 के छावनी अ ध नयम के
ावधान के अंतगत था पत कया गया है - यह वधान क सरकार वारा न मत
कया गया है । यह क सरकार के र ा मं ालय के शास नक नयं ण के अधीन काय
करता है । अ य शहर थानीय नकाय के वपर त, जो क रा य वारा शा सत एवं
ग ठत क गयी ह, एक छावनी प रषद क सरकार वारा शा सत एवं ग ठत कया
जाता है ।
कथन 3 गलत है: नगर य े , एक कार क शहर सरकार है , िजसे बड़े सावज नक
उ यम वारा संयं के पास बनी आवासीय कॉलो नय म रहने वाले अपने कमचा रय
और मक को नाग रक सु वधाएं दान करने के लए था पत कया जाता है । यह
उप म नगर के शासन क दे खभाल के लए एक नगर शासक क नयिु त करता है।
उसे कुछ इंजी नयर और अ य तकनीक और गैर-तकनीक कमचा रय वारा सहायता
दान क जाती है । इस कार, शहर शासन के नगर य प म कोई नवा चत सद य
नह ं होते ह। वा तव म, यह उप म क नौकरशाह संरचना का व तार है ।
कथन 4 सह है : नगर पा लकाओं क थापना क ब और छोटे शहर के शासन के
लए क जाती है । नगम क तरह, वे भी रा य म संबं धत रा य वधानमंडल के
अ ध नयम वारा और क शा सत दे श म भारत क संसद के अ ध नयम वारा
ग ठत क जाती ह। उ ह कई अ य नाम से भी जाना जाता है जैस-े नगरपा लका
प रषद, नगरपा लका स म त, नगरपा लका बोड, उपनगर य नगरपा लका, शहर
नगरपा लका और अ य।
कथन 5 गलत है: दो कार के े के शासन के लए एक अ धसू चत े स म त का
गठन कया गया है - औ योगीकरण के कारण वकासशील क बा, और वह क बा, जो
अभी तक नगरपा लका के गठन के लए आव यक सभी शत को पूरा नह ं करता है ,
ले कन जो रा य सरकार वारा मह वपण
ू माना जाता है ।चंू क, यह सरकार राजप म
एक अ धसूचना वारा था पत कया जाता है , इस लए इसे अ धसू चत े स म त के
प म जाना जाता है ।

ोत: एम ल मीकांत वारा भारतीय राजनी त छठा सं करण, अ याय 39

8. कसी राजनी तक दल को रा य दल के प म मा यता दान करने के लए


न न ल खत म से कौन-सी आव यक शत पूर करनी होगी?
1. इसे लोकसभा या वधान सभा के आम चन
ु ाव म क ह ं चार या अ धक रा य म
डाले गए वैध मत का छह तशत ा त करना चा हए
2. आम चन
ु ाव म इसे लोकसभा म दो तशत सीट जीतनी चा हए और ये उ मीदवार
तीन रा य से चन
ु े जाने चा हए।
3. इसे चार रा य म एक रा य दल के प म मा यता ा त होनी चा हए।
नीचे दए गए कूट का योग कर सह उ र चु नए:
(a) केवल 1 और 2
(b) केवल 2 और 3
(c) केवल 1 और 3
(d) 1, 2 और 3

उ र: b
या या:
वक प (b) सह है : चन
ु ाव आयोग, चन
ु ाव के उ दे य से राजनी तक दल को पंजीकृत
करता है और उ ह उनके चन
ु ावी दशन के आधार पर रा य या रा य दल के प म
मा यता दान करता है ।
एक दल को रा य दल के प म मा यता द जाती है , य द न न ल खत म से कोई
भी शत परू करता है :
 य द यह लोकसभा या वधान सभा के आम चन
ु ाव म क ह ं चार या अ धक
रा य म डाले गए वैध मत का छह तशत ा त करता है ; और, इसके
अ त र त, यह कसी भी रा य या रा य से लोकसभा म चार सीट जीतता है ; या
(दोन शत एक ह समय म पूर होनी चा हए) ( ववरण 1 गलत है )
 य द वह आम चन
ु ाव म लोकसभा म दो तशत सीट जीतता है ; और ये
उ मीदवार तीन रा य से चन
ु े जाने चा हए ( ववरण 2 सह है ); या
 य द इसे चार रा य म एक रा य दल के प म मा यता ा त होनी चा हए
( ववरण 3 सह है )।

ोत: एम ल मीकांत वारा भारतीय राजनी त 6 वां सं करण, अ याय 69

9. रा प त शासन क उ घोषणा क या यक समी ा के संदभ म, न न ल खत कथन


पर वचार क िजएः
1. 38व सं वधान संशोधन अ ध नयम, 1975 ने रा प त क संतुि ट को या यक
समी ा के दायरे म ला दया।
2. अदालत साम ी क शु धता म नह ं जा सकती है ले कन यह दे ख सकती है क
यह कारवाई के लए ासं गक है या नह ं।
3. भले ह अदालत रा प त क उ घोषणा को असंवैधा नक और अमा य मानती हो,
ले कन उसके पास रा य वधान सभा को भंग करने क ि थ त म उसे पुनज वत
करने क शि त नह ं है ।
ऊपर दए गए कथन म से कौन-सा/से सह नह ं है /ह?
(a) केवल 1 और 3
(b) केवल 1 और 2
(c) केवल 2
(d) 1, 2 और 3

उ र: A
या या:
कथन 1 गलत है : 1975 के 38व सं वधान संशोधन अ ध नयम ने अनु छे द 356 को
अं तम और नणायक लागू करने म रा प त को संतु ट कया िजसे कसी भी आधार
पर कसी भी अदालत म चन
ु ौती नह ं द जा सकती थी। ले कन इस ावधान को बाद म
1978 के 44व संशोधन अ ध नयम वारा हटा दया गया था, िजसका अथ था क
रा प त क संतिु ट या यक समी ा से परे नह ं है ।
कथन 2 सह है ले कन कथन 3 गलत है : S.R. बो मई केस (1994), सु ीम कोट
वारा अनु छे द 356 के तहत एक रा य म रा प त शासन लगाने पर न न ल खत
ताव रखे गए ह:
1. रा प त शासन लागू करने वाल रा प त क घोषणा या यक समी ा के अधीन
है ।
2. रा प त क संतुि ट ासं गक साम ी पर आधा रत होनी चा हए। रा प त क
कारवाई को अदालत वारा र द कया जा सकता है य द यह अ ासं गक या
बाहर आधार पर आधा रत है या य द इसे दभ
ु ावनापण
ू या वकृत पाया गया है ।
3. रा प त शासन लागू करने को सह ठहराने के लए ासं गक साम ी मौजूद है ,
यह सा बत करना क क िज मेदार है ।
4. यायालय साम ी क शु धता या उसक पया तता म नह ं जा सकता ले कन यह
दे ख सकता है क यह कारवाई के लए ासं गक है या नह ं।
5. य द अदालत रा प त क घोषणा को असंवैधा नक और अमा य मानती है , तो
उसे बखा त रा य सरकार को बहाल करने और रा य वधानसभा को नलं बत
या भंग करने पर पुनज वत करने क शि त है।
6. संसद वारा रा पत क उ घोषणा को मंजरू मलने के बाद ह रा य
वधानसभा को भंग कया जाना चा हए। जब तक ऐसी मंजरू नह ं द जाती,
रा प त केवल वधानसभा को नलं बत कर सकते ह। य द संसद उ घोषणा को
मंजूर दे ने म वफल रहती है , तो वधानसभा फर से स य हो जाएगी।
7. धम नरपे ता सं वधान क 'बु नयाद वशेषताओं' म से एक है । इस लए,
धम नरपे वरोधी राजनी त करने वाल रा य सरकार अनु छे द 356 के तहत
कारवाई के लए उ रदायी है ।
8. रा य सरकार के वधान सभा का व वास खोने के न का नणय सदन के
पटल पर कया जाना चा हए और जब तक ऐसा नह ं कया जाता है तब तक
मं ालय को नह ं बैठना चा हए।
9. जहां कोई नया राजनी तक दल क म स ा हण करता है , उसे रा य म अ य
दल वारा ग ठत मं ालय को बखा त करने का अ धकार नह ं होगा।
10. अनु छे द 356 के तहत शि त एक असाधारण शि त है और वशेष प रि थ तय
क आव यकताओं को परू ा करने के लए कभी-कभार ह इसका उपयोग कया
जाना चा हए।

10. भारत म संघ कायका रणी के संबंध म न न ल खत कथन पर वचार कर:


1. धान मं ी के अलावा यि तगत मं ी धान मं ी क इ छानस
ु ार पद धारण
करते ह।
2. मं प रषद सामू हक प से संसद के त उ रदायी होगी।
ऊपर दए गए कथन म से कौन-सा/से सह नह ं है /ह?
(a) केवल 1
(b) केवल 2
(c) दोन 1 और 2
(d) न तो 1 और न ह 2
उ र: C
या या:
कथन 1 गलत है: अनु छे द 75(2) म यि तगत िज मेदार का स धांत शा मल है ।
इसम कहा गया है क मं प रषद रा प त क इ छानुसार पद धारण करती है , िजसका
अथ है क रा प त कसी मं ी को ऐसे समय म भी हटा सकता है जब मं प रषद को
लोकसभा का व वास ा त हो। हालाँ क, रा प त कसी मं ी को धानमं ी क सलाह
पर ह हटाता है । कसी मं ी के दशन से मतभेद या असंतोष के मामले म, धान मं ी
उसे इ तीफा दे ने के लए कह सकता है या रा प त को उसे बखा त करने क सलाह दे
सकता है । मं प रषद म मं य क तीन े णयां होती ह, अथात ् कै बनेट मं ी, रा य
मं ी और उप मं ी। धानमं ी इन सभी मं य म सबसे ऊपर होता है |
कथन 2 गलत है : अनु छे द 75(3) सरकार क संसद य णाल क आधार शला है और
यह ावधान करता है क मं प रषद सामू हक प से लोकसभा के त उ रदायी होगी
(संसद नह ं)। वे एक ट म के प म काय करते ह, और एक साथ तैरते और डूबते ह।
सामू हक िज मेदार के स धांत का ता पय है क लोकसभा अ व वास ताव पा रत
करके मं ालय (यानी, धान मं ी क अ य ता वाल मं प रषद) को पद से हटा सकती
है ।

ोत: एम ल मीकांत वारा भारतीय राजनी त छठा सं करण, अ याय-17, 20

11. भारतीय सं वधान के आपातकाल न ावधान को भौ तक प से उधार लया गया है


(a) भारत सरकार अ ध नयम, 1935
(b) जमनी का वीमर सं वधान
(c) द ण अ क सं वधान
(d) टश सं वधान

उ र: A
वक प (a) सह है : भारत के सं वधान ने अपने अ धकांश ावधान को व भ न दे श के
सं वधान के साथ-साथ 1935 के भारत सरकार अ ध नयम से उधार लया है । वा तव म,
डॉ बी.आर. अ बेडकर ने गव से शंसा क क भारत के सं वधान को 'द ु नया के सभी
ात सं वधान को तोड़कर' बनाया गया है। वा तव म, सं वधान का सबसे गहरा भाव
और भौ तक ोत भारत सरकार अ ध नयम, 1935 है । संघीय योजना, यायपा लका,
रा यपाल, आपातकाल न ावधान और शास नक ववरण, लोक सेवा आयोग और
अ धकांश शास नक ववरण इसी से लए गए ह। सं वधान के आधे से अ धक ावधान
1935 के अ ध नयम के समान ह।
जमनी के वीमर सं वधान से लए गए आपातकाल के दौरान 'मौ लक अ धकार के
नलंबन' के ोत से मत नह ं होना चा हए।

ोत: एम ल मीकांत वारा भारतीय राजनी त छठा सं करण

12. भारतीय सं वधान के अनु छे द 352 के संदभ म न न ल खत कथन पर वचार


कर:
1. यह 'रा य आपातकाल' श द को यु ध, बाहर आ मण या सश व ोह के
कारण आपातकाल के प म प रभा षत करता है ।
2. रा प त यु ध या बाहर आ मण या सश व ोह क वा त वक घटना से
पहले ह रा य आपातकाल क घोषणा कर सकते ह।
3. 44व सं वधान संशोधन ने रा प त को रा य आपातकाल के संचालन को भारत
के एक व श ट ह से तक सी मत करने म स म बनाया।
4. रा प त मं प रषद से ल खत सफा रश ा त करने के बाद ह रा य
आपातकाल क घोषणा कर सकते ह।
ऊपर दए गए कथन म से कौन-सा/से सह है /ह?
(a) केवल 2
(b) केवल 1 और 3
(c) केवल 3 और 4
(d) केवल 2 और 3

उ र: A

या या:
कथन 1 गलत है : अनु छे द 352 के तहत, सं वधान यु ध, बाहर आ मण या सश
व ोह के कारण आपातकाल क यव था करता है । इसे लोक य प से 'रा य
आपातकाल' के प म जाना जाता है । हालाँ क, सं वधान 'रा य आपातकाल'
अ भ यि त का उपयोग नह ं करता है । यह इस कार क आपात ि थ त को दशाने के
लए 'आपातकाल क उ घोषणा' अ भ यि त को नयोिजत करता है ।
कथन 2 सह है : रा प त यु ध या बाहर आ मण या सश व ोह क वा त वक
घटना से पहले ह रा य आपातकाल क घोषणा कर सकते ह, य द वे संतु ट ह क
एक आस न खतरा है ।
कथन 3 गलत है : 1976 के 42व संशोधन अ ध नयम ने रा प त को रा य
आपातकाल के संचालन को भारत के एक व श ट ह से तक सी मत करने म स म
बनाया।
कथन 4 गलत है : हालां क, रा प त, कै बनेट से ल खत सफा रश ा त करने के बाद
ह रा य आपातकाल क घोषणा कर सकते ह (मं प रषद क प रषद नह ं)। इस
ावधान को 44व सं वधान संशोधन म जोड़ा गया।

ोत: एम ल मीकांत वारा भारतीय राजनी त छठा सं करण, अ याय-16

13. भारतीय धान मं ी के संबंध म, न न ल खत कथन पर वचार कर:


1. भारत के सं वधान ने धानमं ी का कायकाल पांच साल के लए तय कया है ।
2. एक यि त जो संसद के कसी भी सदन का सद य नह ं है , उसे धान मं ी के
प म नयु त नह ं कया जा सकता है ।
3. संसद य परं परा के अनुसार, एक यि त को धान मं ी के प म नयु त होने से
पहले लोकसभा म अपना बहुमत सा बत करना होगा।
ऊपर दए गए कथन म से कौन-सा/से सह नह ं है /ह ?
(a) केवल 1
(b) केवल 2 और 3
(c) केवल 3
(d) 1, 2 और 3
उ र: d
या या:
कथन 1 गलत है : धान मं ी का कायकाल सं वधान वारा नधा रत नह ं होता है और
वह रा प त के साद पयत पद धारण करता है । हालां क, इसका मतलब यह नह ं है क
रा प त कसी भी समय धानमं ी को बखा त कर सकते ह। जब तक धान मं ी को
लोकसभा म बहुमत का समथन ा त है , तब तक उ ह रा प त वारा बखा त नह ं
कया जा सकता है । हालाँ क, य द वह लोकसभा का व वास खो दे ता है , तो उसे इ तीफा
दे ना होगा या रा प त उसे बखा त कर सकता है ।
कथन 2 गलत है : भारतीय सं वधान म ावधान है क धानमं ी संसद के कसी भी सदन
का सद य हो सकता है । इसके अलावा, 1997 म, सव च यायालय ने यह यव था द क
जो यि त संसद के कसी भी सदन का सद य नह ं है , उसे छह मह ने के लए धान
मं ी के प म नयु त कया जा सकता है , िजसके भीतर उसे संसद के कसी भी सदन
का सद य बनना चा हए ; अ यथा, वह धान मं ी पद पर नह ं बना रह सकता।
कथन 3 गलत है : 1980 म, द ल उ च यायालय ने माना क सं वधान के अनुसार यह
आव यक नह ं है क एक यि त को धान मं ी के प म नयु त होने से पहले
लोकसभा म अपना बहुमत सा बत करना होगा। रा प त पहले उसे धान मं ी नयु त
कर सकता है और फर उसे उ चत अव ध के भीतर लोकसभा म बहुमत सा बत करने के
लए कह सकता है ।
ोत: एम. ल मीकांत वारा भारतीय राजनी त, छठा सं करण- अ याय- 19।

14. न न ल खत कथन पर वचार कर:


1. रा प त केवल उ ह ं यि तय को मं ी के प म नयु त कर सकता है िजनक
सफा रश धान मं ी वारा क जाती है ।
2. जब एक मौजद
ू ा धान मं ी क म ृ यु हो जाती है , तो इसका मतलब यह नह ं है
क मं प रषद वतः भंग हो जाती है ।
3. धानमं ी कसी भी समय रा प त को लोकसभा भंग करने क सफा रश कर
सकता है ।
ऊपर दए गए कथन म से कौन-से सह ह?
(a) केवल 1 और 2
(b) केवल 1 और 3
(c) केवल 2 और 3
(d) 1, 2, और 3

उ र: b
या या:
कथन 1 सह है : धानमं ी उन यि तय क सफा रश करता है िज ह रा पत वारा
मं ी के प म नयु त कया जाना है । रा प त केवल उ ह ं यि तय को मं ी के प
म नयु त कर सकता है िजनक सफा रश धान मं ी वारा क जाती है ।
कथन 2 गलत है : धान मं ी मं प रषद के मख
ु के प म होता है , अ य मं ी तब
काय नह ं कर सकते जब धान मं ी इ तीफा दे दे ते ह या उनक म ृ यु हो जाती
ह । दस
ू रे श द म, एक मौजद
ू ा धान मं ी का इ तीफा या मृ यु वतः ह मं प रषद
को भंग कर दे ती है और इस तरह खाल पन(वै यम
ू ) क ि थ त बनती है ।
कथन 3 सह है : धानमं ी मं प रषद के मु खया के प म होता है । धान मं ी
रा प त को संसद के स को बुलाने और स ावसान करने के संबंध म सलाह दे ता
है । वह कसी भी समय रा प त को लोकसभा भंग करने क सफा रश कर सकता है ।
ोत: एम. ल मीकांत वारा भारतीय राजनी त, छठा सं करण- अ याय- 19।

15. भारत के सं वधान के संदभ म, न न ल खत म से कौन से कथन सह ह?

1. अनु छे द 358 अनु छे द 19 के तहत मौ लक अ धकार को परू तरह से नलं बत करता है


जब क अनु छे द 359 रा प त को अ य सभी मौ लक अ धकार के वतन को नलं बत
करने का अ धकार दे ता है ।
2. अनु छे द 358 का व तार परू े दे श म है जब क अनु छे द 359 का व तार परू े दे श या उसके
कसी भाग पर हो सकता है ।
3. अनु छे द 19 के तहत मौ लक अ धकार को तभी नलं बत कया जा सकता है जब यु ध
या बाहर आ मण के आधार पर रा य आपातकाल क घोषणा क जाती है।
नीचे दए गए कूट का योग कर सह उ र चु नए:
(a) केवल 1 और 2
(b) केवल 2 और 3
(c) केवल 1 और 3
(d) 1, 2 और 3

उ र: B
या या:
अनु छे द 358 और 359 मौ लक अ धकार पर रा य आपातकाल के भाव का वणन करते ह।
अनु छे द 358 अनु छे द 19 वारा गारं ट कृत मौ लक अ धकार के नलंबन से संबं धत है , जब क
अनु छे द 359 अ य मौ लक अ धकार (अनु छे द 20 और 21 वारा गारं ट कृत को छोड़कर) के
नलंबन से संबं धत है । अनु छे द 358 और 359 के बीच के अंतर को सं ेप म तुत कया जा
सकता है :
कथन 1 गलत है: जैसे ह आपातकाल घो षत कया जाता है , अनु छे द 358 अनु छे द 19 के तहत
मौ लक अ धकार को वचा लत प से नलं बत कर दे ता है । दस
ू र ओर, अनु छे द 359 कसी भी
मौ लक अ धकार को वतः नलं बत नह ं करता है । यह रा प त को अनु छे द 20 और 21 के तहत
दए गए अ धकार को छोड़कर अ य न द ट मौ लक अ धकार के वतन को नलं बत करने का
अ धकार दे ता है ।
कथन 2 सह है : अनु छे द 358 का व तार पूरे दे श म है जब क अनु छे द 359 का व तार पूरे दे श
या उसके एक ह से तक हो सकता है ।
कथन 3 सह है: 1978 के 44व सं वधान संशोधन अ ध नयम ने अनु छे द 358 के दायरे को दो तरह
से तबं धत कर दया। सबसे पहले, अनु छे द 19 के तहत छह मौ लक अ धकार को केवल तभी
नलं बत कया जा सकता है जब रा य आपातकाल यु ध या बाहर आ मण के आधार पर
घो षत कया गया हो, न क सश व ोह के आधार पर। दस
ू रे , केवल वे कानन
ू जो आपातकाल से
संबं धत ह, उ ह चन
ु ौती दे ने से बचाया जाता है न क अ य कानून को। साथ ह ऐसे कानून के तहत
क गई कायकार कारवाई भी सुर त है ।

ोत: एम ल मीकांत वारा भारतीय राजनी त छठा सं करण, अ याय-16

16. न न ल खत कथन पर वचार क िजएः


1. क और रा य के बीच वधायी शि तय का वतरण अन य है ।
2. संसद क के कसी भी कायकार काय को कसी भी रा य को स प सकती
है , यहां तक क उस रा य क सहम त के बना भी।
3. क और रा य के बीच काय का पार प रक आदान दान एक कानून के
मा यम से ह हो सकता है ।
ऊपर दए गए कथन म से कौन-सा/से सह है /ह?
(a) केवल 1
(b) केवल 1 और 3
(c) केवल 2 और 3
(d) 1, 2, और 3

उ र: a
या या:
कथन 1 सह है : क और रा य के बीच वधायी शि तय का वतरण अन य
है । नतीजतन, क अपनी वधायी शि तय को रा य को नह ं स प सकता है और एक
रा य संसद से रा य के वषय पर कानून बनाने का अनरु ोध नह ं कर सकता
है । कायपा लका े म इस तरह के कठोर वभाजन से दोन के बीच कभी-कभार टकराव
हो सकता है । इस लए, सं वधान कठोरता को कम करने और ग तरोध क ि थ त से बचने
के लए कायकार काय के अंतर-सरकार आदान दान का ावधान करता है ।
कथन 2 गलत है : रा प त [संसद नह ं], रा य सरकार क सहम त से, क के कसी भी
कायकार काय को उस सरकार को स प सकते ह। इसके वपर त, कसी रा य का
रा यपाल, क सरकार क सहम त से, क को रा य के कसी भी कायकार काय को
स प सकता है । शास नक काय का यह पार प रक आदान दान सशत या बना शत
हो सकता है। सं वधान क के कायकार काय को उस रा य क सहम त के बना कसी
रा य को स पने का भी ावधान करता है । ले कन, इस वषय म, आदान दान संसद
वारा होता है , न क रा प त वारा ।
कथन 3 गलत है : क और रा य के बीच काय का पार प रक आदान दान या तो एक
समझौते के तहत या एक कानन
ू वारा हो सकता है । जब क क दोन व धय का
उपयोग कर सकता है , एक रा य केवल पहल व ध का उपयोग कर सकता है ।
ोत: एम. ल मीकांत वारा भारतीय राजनी त, छठा सं करण- अ याय- 14।

17. न न ल खत यु म पर वचार क िजएः


सं वधान का ावधान
अनु छे द
1. अनु छे द 268 संघ वारा लगाए गए कर ले कन रा य वारा एक
और व नयोिजत।
2. अनु छे द 269 कर क वारा लगाया और एक कया जाता है
ले कन क और रा य के बीच वत रत कया जाता
है ।
3. अनु छे द 270 क वारा लगाए और एक कए गए कर ले कन
रा य को स पे गए।
4. अनु छे द 271 संसद व तु एवं सेवा कर पर अ धभार नह ं लगा
सकती है ।
ऊपर दए गए कतने जोड़े सह सुमे लत ह/ह?
(a) केवल एक जोड़ी
(b) केवल दो जोड़े
(c) केवल तीन जोड़े
(d) सभी चार जोड़े

उ र: b
या या:
जोड़ी 1 सह ढं ग से मेल खाती है : अनु छे द 268 क वारा लगाए गए कर के लए
ावधान करता है ले कन ये रा य वारा एक और व नयोिजत कये जाते है । इस
ेणी म बल ऑफ ए सचज, चेक, ॉ मसर नो स, बीमा पॉ ल सय , शेयर के ह तांतरण
और टांप पर शु क शा मल ह। कसी भी रा य के भीतर लगाए गए इन करो क
आय भारत क सं चत न ध का ह सा नह ं होती है , बि क उस रा य को स पी जाती है ।
जोड़ी 2 का गलत मलान कया गया है: अनु छे द 269 क वारा लगाए और एक कए
गए ले कन रा य को स पे गए करो के वषय म ावधान करता है । न न ल खत कर
इस ेणी के अंतगत आते ह:
(i) अंतर-रा यीय यापार या वा ण य के दौरान माल (समाचार प के अलावा) क ब
या खर द पर कर।
(ii) अंतर-रा यीय यापार या वा ण य के दौरान माल क खेप पर कर।
इन कर क शु ध आय भारत क सं चत न ध का ह सा नह ं है । उ ह संसद वारा
नधा रत स धांत के अनुसार संबं धत रा य को स पा गया है ।
जोड़ी 3 का गलत मलान कया गया है : अनु छे द 270 क वारा लगाए और एक कए
गए कर के वषय म ावधान करता है ले कन ये क और रा य के बीच वत रत
कये जाते है । इस ेणी म न न ल खत को छोड़कर संघ सूची म न द ट सभी कर
और शु क शा मल ह:
(i) अंतरा यीय यापार या वा ण य के दौरान आपू त, पर व तु एवं सेवा कर (जीएसट )
(अनु छे द 268,269 एव 269A म संद भत शु क और कर)
(ii) अनु छे द 271 म उि ल खत वा ण य कर और शु क पर अ धभार।
(iii) व श ट उ दे य के लए लगाया गया कोई उपकर।
इन कर और शु क क शु ध आय के वतरण का तर का रा पत वारा व आयोग
क सफा रश पर नधा रत कया जाता है ।

जोड़ी 4 का सह मलान कया गया है : संसद कसी भी समय अनु छे द 269 और 270
(ऊपर उि ल खत) म उि ल खत कर और शु क पर अ धभार लगा सकती है । इस तरह
के अ धभार क आय वशेष प से क के पास जाती है । दस
ू रे श द म, इन अ धभार म
रा य का कोई ह सा नह ं है । हालां क, व तु और सेवा कर (जीएसट ) को इस अ धभार
से छूट द गई है । दस
ू रे श द म, यह अ धभार GST पर नह ं लगाया जा सकता है ।
ोत: एम. ल मीकांत वारा भारतीय राजनी त, छठा सं करण- अ याय -14।

18. न न ल खत कथन पर वचार क िजएः


1. ववेकाधीन अनद
ु ान क सरकार वारा उन रा य को दया जाता है िज ह
व ीय सहायता क आव यकता होती है और भारत क सं चत न ध पर भा रत
होते ह।
2. अनु छे द 275 के तहत वैधा नक अनद
ु ान रा य को व आयोग क
सफा रश पर दया जाता है ।
3. स को और मु ा से ाि तयां क के लए गैर-कर राज व के ोत ह।
ऊपर दए गए कथन म से कौन-से सह ह?
(a) केवल 1 और 2
(b) केवल 1 और 3
(c) केवल 2 और 3
(d) 1, 2, और 3

उ र: c
या या:
कथन 1 गलत है : अनु छे द 275 संसद (क सरकार को नह ं) को उन रा य को सहायता
अनद
ु ान दे ने का अ धकार दे ता है , िज ह व ीय सहायता क आव यकता है , न क येक
रा य को। साथ ह अलग-अलग रा य के लए अलग-अलग रा श तय क जा सकती
है । ये रकम हर साल भारत क सं चत न ध से दान क जाती है । इस सामा य
ावधान के अलावा, सं वधान रा य म अनस
ु ू चत जनजा तय के क याण को बढ़ावा दे ने
के लए असम स हत अ य रा यो म अनस
ु ू चत े के शासन तर को बढ़ाने
के लए व श ट अनद
ु ान दान करता है ।
कथन 2 सह है : अनु छे द 275 (सामा य और व श ट दोन ) के तहत वैधा नक अनुदान
रा य को व आयोग क सफा रश पर दया जाता है ।
कथन 3 सह है : न न ल खत से ाि तयां क के गैर-कर राज व के मु ख ोत ह: (i)
पो ट और टे ल ाफ; (ii) रे लवे; (iii) ब कंग; (iv) सारण (v) स का और मु ा ; (vi) क य
सावज नक े के उ यम; (vii) ए चेअत एंड लै स (escheat and lapse) (viii) अ य।

ोत: एम. ल मीकांत वारा भारतीय राजनी त, छठा सं करण- अ याय- 14 I

19. न न ल खत कथन पर वचार कर:


1. अमे रक रा प त को वधा यका का एक ह सा नह ं माना जाता है ।
2. ू ाइटे ड कंगडम म,
यन ाउन संसद क सं था का एक अ भ न अंग है ।
ऊपर दए गए कथन म से कौन-सा/से सह है /ह?
(a) केवल 1
(b) केवल 2
(c) दोन 1 और 2
(d) न तो 1 और न ह 2

उ र: c
या या:
कथन 1 सह है: अमे रका क रा पत णाल के तहत, रा प त कायपा लका मुख होता
है । शासन क शि त उसी म न हत होती है । अमे रका क रा प त णाल कायपा लका और
वधा यका के अलगाव पर आधा रत है । ता क रा प त और उनके स चव कां ेस (अमे रक
वधा यका ) के सद य न हो सक, िजसम सीनेट (उ च सदन) और त न ध सभा ( न न सदन)
शा मल ह।
ू ाइटे ड कंगडम म संसद सव च वधायी
कथन 2 सह है : यन ा धकरण है । जो हाउस ऑफ
कॉम स, हाउस ऑफ लॉ स और कंग (जो यूके के वतमान वंशानुगत स ाट ह) से बनती है । हाउस
ऑफ कॉम स और हाउस ऑफ लॉ स के साथ , ाउन संसद का एक अ भ न अंग होता है ।राजा
संसद को शु और भंग करने और कानून बनने से पहले वधेयक को मंजूर दे ने म एक संवैधा नक
भू मका नभाता है । ाउन संसद को रा य के उ घाटन (संसद य वष क शु आत को चि नत
करते हुए) के मा यम से खोलता है । ाउन आम चन
ु ाव से पहले संसद को भंग कर दे ता है । ाउन
हाउस ऑफ लॉ स म संहासन से दए गए भाषण म संसद को सरकार के नी तगत वचार और नए
कानून क योजनाओं के बारे म सू चत करता है । हालां क राजा अ भभाषण दे ता है , जो सरकार
वारा तैयार कया जाता है।
इसी तरह, भारत का रा प त संसद के कसी भी सदन का सद य नह ं है और संसद क बैठक म
भाग लेने के लए नह ं बैठता है , फर भी वह संसद का एक अ भ न अंग है । ऐसा इस लए है य क
संसद के दोन सदन वारा पा रत वधेयक रा प त क सहम त के बना कानन
ू नह ं बन सकते
है ।
ोत:
एम. ल मीकांत वारा भारतीय राजनी त -6 वां सं करण-अ याय- 22
https://www। संसद.यूके /के बारे म/कैसे/भू मका/ संबंध के साथ-अ य- सं थान/संसद-
मुकुट/ #:~:पाठ=साथ%20साथ%20the% 20 हाउस%20of,Bills%20% 20before%20वे%20कानून
बन जाते ह

20. न न ल खत म से कौन सा/से कथन सह है /ह?


1. भारत का सं वधान रा प त को यह नधा रत करने का अ धकार दे ता है क प रसीमन
कस तर के से कया जाना है ।
2. वतमान म, लोकसभा और रा य वधानसभाओं म व भ न रा य को आवं टत मौजूदा
सीट क कुल सं या 1971 क जनगणना पर आधा रत है ।
3. 2003 के 87व संशोधन अ ध नयम ने सरकार को 1971 क जनगणना के जनसं या आंकड़
के आधार पर रा य म े ीय नवाचन े के पुन: समायोजन और युि तकरण करने का
अ धकार दया।
नीचे दए गए कूट का योग कर सह उ र चु नए:
(a) केवल 1
(b) केवल 1 और 3
(c) केवल 2
(d) 1, 2, और 3
उ र: c
या या:
कथन 1 गलत है: भारत के सं वधान ने संसद को प रसीमन के तर के का नधारण करने का
अ धकार दया है । तदनुसार, संसद ने इस उ दे य के लए 1952, 1962, 1972 और 2002 म
प रसीमन आयोग अ ध नयम बनाए ह।
कथन 2 सह है: 1976 के 42व संशोधन अ ध नयम ने 1971 के तर पर वष 2000 तक रा य को
लोकसभा म सीट के आवंटन और े ीय नवाचन े म येक रा य के वभाजन को रोक
दया । 2001 क जनगणना के आंकड़ के काशन तक रा य को आवं टत सीट क सं या और
नवाचन े क सीमाओं को तय करने पर रोक लगाने के लए सं वधान के अनु छे द 82 और 170
म संशोधन कया गया था।
हालां क, 2001 म, 84 व सं वधान संशोधन अ ध नयम ने नवाचन े क सं या पर और 25 वष
तक रोक लगा द - 2026 तक । दस
ू रे श द म, 1971 क जनगणना के आधार पर 'जनता के सदन'
(लोकसभा) और रा य वधानसभाओं म व भ न रा य को आवं टत मौजूदा सीट क कुल सं या
वष 2026 के बाद होने वाल पहल जनगणना तक अप रव तत रहे गी।
कथन 3 गलत है : 2001 के 84व संशोधन अ ध नयम ने सरकार को 1991 क जनगणना के
जनसं या आंकड़ के आधार पर रा य म े ीय नवाचन े के पुनसमायोजन और युि तकरण
करने का अ धकार दया । बाद म, 2003 के 87व संशोधन अ ध नयम ने 2001 क जनगणना के
आधार पर नवाचन े के प रसीमन का ावधान कया, न क 1991 क जनगणना के आधार
पर । हालां क, यह लोकसभा म येक रा य को आवं टत सीट क सं या म बदलाव कए बना
कया जा सकता है ।
' नवाचन े पर रोक' ने सम याओं के एक और सेट को ज म दया। इन वष म, सामा य
जनसं या के सापे अनस
ु ू चत जा त और अनस
ु ू चत जनजा त क जनसं या का अनप
ु ात मशः
15 तशत से बढ़कर 16.6 तशत और 7.5 से 8.6 तशत हो गया। लोक तनध व
अ ध नयम, 1961 क धारा 3 के साथ प ठत सं वधान के अनु छे द 330 के जनादे श के अनस
ु ार,
उनके लए आर त सीट क सं या को समायोिजत कया जाना था। यह वह संदभ था िजसम
87वां संशोधन 1991 क जनगणना के बजाय 2001 क जनगणना के आधार पर आर त सीट के
पन
ु नधारण का ावधान करने के लए कया गया था।
अ त र त जानकार :
2019 के ज मू और क मीर अ ध नयम वारा पुनगठन के बाद, रा य को ल दाख और ज म-ू
क मीर क शा सत दे श के प म पुनग ठत कया गया था। 2011 क जनगणना के आधार पर
ज म-ू क मीर म नवाचन े का प रसीमन करने के लए यायमू त रं जना दे साई क अ य ता
म प रसीमन आयोग क थापना क गई थी। इसके आधार पर ज मू म 43 और क मीर म
वधानसभा क 47 सीट ह गी और पहल बार नौ सीट अनुसू चत जनजा त के लए और सात
अनुसू चत जा त के लए आर त ह गी। अ य 24 सीट को पा क तान के क जे वाले क मीर
(पीओके) के नवाचन े के लए रखा गया है ।

Sources:
https://theprint.in/ राय/is- india -ready-for- delimitation -of-constuencies -time-has-
come- for-reorganisation-of-states/ 793024/
https://www.thehindu.com/news/ National /law-ministry- notify -may-20-as-date-for-
delimitation-order- काया वयन/ article65440305.ece
एम. ल मीकांत वारा राजनी त -6 वां सं करण-अ याय -22

21. व भ न आयोग क सफा रश के संबंध म, न न ल खत कथन पर वचार कर:


1. शास नक सध
ु ार आयोग ने 1966 म सं वधान के अनु छे द 263 के तहत
एक अंतर-रा य प रषद क थापना क सफा रश क ।
2. सरका रया आयोग ने सफा रश क क रा यपाल मं प रषद को तब तक
बखा त नह ं कर सकता जब तक क उसके पास वधानसभा म बहुमत न हो।
3. पुंछ आयोग ने सफा रश क क रा यपाल के प म नयु त यि त
रा य के बाहर से होना चा हए।
ऊपर दए गए कथन म से कौन-से सह ह?
(a) केवल 1 और 2
(b) केवल 1 और 3
(c) केवल 2 और 3
(d) 1, 2, और 3
उ र: d
या या:
कथन 1 सह है : क सरकार ने 1966 म मोरारजी दे साई (के हनुमंत या के बाद) क
अ य ता म छह सद यीय शास नक सुधार आयोग (एआरसी) क नयुि त क
। इसने सं वधान के अनु छे द 263 के तहत एक अंतर-रा य प रषद क थापना क
सफा रश क । एआरसी क सफा रश पर क सरकार वारा कोई कारवाई नह ं क गई।
कथन 2 सह है : 1983 म, क सरकार ने सु ीम कोट के एक सेवा नव ृ यायाधीश, आरएस
सरका रया क अ य ता म क -रा य संबंध पर तीन सद यीय आयोग नयु त
कया। आयोग को सभी े म क और रा य के बीच मौजूदा यव थाओं क जांच
और समी ा करने और उ चत बदलाव और उपाय क सफा रश करने का काम स पा
गया था।
इसने क -रा य संबंध म सुधार के लए 247 सफा रश क ं । ऐसी सफा रश म से एक
यह थी क रा यपाल मं प रषद को तब तक बखा त नह ं कर सकता जब तक क
उसके पास वधानसभा म बहुमत न हो।
कथन 3 सह है : क -रा य संबंध पर दस
ू रा आयोग भारत सरकार वारा अ ैल 2007 म
भारत के पूव मु य यायाधीश मदन मोहन पुंछ क अ य ता म था पत कया गया
था। इसने सफा रश क क रा यपाल का चयन करते समय, क सरकार को सरका रया
आयोग क रपोट म अनुशं सत न न ल खत स त दशा- नदश को अपनाना चा हए और
उसके वा त वक अथ व भाव को अपने आदे श म पालन करना चा हए:
• उसे जीवन के कसी न कसी े म ति ठत होना चा हए
• वह रा य के बाहर का यि त होना चा हए
• वह एक अलग यि त होना चा हए और रा य क थानीय राजनी त से बहुत
अ धक नकटता से नह ं जुड़ा होना चा हए।
• वह ऐसा यि त होना चा हए िजसने आम तौर पर और वशेष प से हाल के
दन म राजनी त म बहुत अ धक स य नह ं हो।
ोत: एम. ल मीकांत वारा भारतीय राजनी त, छठा सं करण- अ याय- 14।

22. न न ल खत कथन पर वचार कर:


1. अंतर-रा यीय जल ववाद अ ध नयम रा प त को अंतर-रा यीय नद
ववाद के याय नणयन के लए एक तदथ याया धकरण था पत करने का
अ धकार दे ता है ।
2. रा प त जन हत क पू त के लए कसी भी समय अंतररा यीय प रषद
क थापना कर सकते ह।
3. येक मु यमं ी एक बार म एक वष क अव ध के लए च ानु म वारा
जोनल प रषद के अ य के प म काय करता है ।
ऊपर दए गए कथन म से कौन-सा/से सह है /ह?
(a) केवल 1 और 2
(b) केवल 2
(c) केवल 2 और 3
(d) केवल 3

उ र: b
या या:
कथन 1 गलत है : अंतर-रा यीय जल ववाद अ ध नयम क सरकार को एक अंतर-रा यीय
नद या नद घाट के पानी के संबंध म दो या दो से अ धक रा य के बीच ववाद के
नणय के लए एक तदथ याया धकरण था पत करने का अ धकार दे ता
है । याया धकरण का नणय अं तम होगा और ववाद के प कार पर बा यकार
होगा। कसी भी जल ववाद के संबंध म न तो सव च यायालय और न ह कसी अ य
यायालय के पास अ धकार े है , िजसे इस अ ध नयम के तहत ऐसे याया धकरण को
संद भत कया जा सकता है । अब तक (2019) क सरकार ने नौ अंतररा यीय जल ववाद
याया धकरण का गठन कया है ।
कथन 2 सह है : सं वधान का अनु छे द 263 रा य के बीच और क और रा य के बीच
सम वय को न पा दत करने के लए एक अंतर-रा य प रषद क थापना पर वचार
करता है । रा प त ऐसी प रषद क थापना कर सकता है य द उसे कसी भी समय यह
तीत हो क उसक थापना से जन हत क पू त होगी। वह ऐसी प रषद और उसके
संगठन और याओं वारा कए जाने वाले कत य क कृ त को प रभा षत कर
सकता है ।
कथन 3 गलत है : े ीय प रषद वैधा नक ( संवैधा नक नह ं) नकाय ह। वे संसद के एक
अ ध नयम, यानी रा य पुनगठन अ ध नयम 1956 वारा था पत क गयी ह
। अ ध नयम ने दे श को पांच े (उ र , म य, पूव , पि चमी और द णी) म वभािजत
कया और येक े के लए एक े ीय प रषद दान क । येक े ीय प रषद म
न न ल खत सद य होते ह :
• क सरकार के गह
ृ मं ी।
• अंचल म सभी रा य के मु यमं ी।
• अंचल म येक रा य से दो अ य मं ी।
• अंचल म येक क शा सत दे श का शासक।
इसके अलावा, न न ल खत यि तय को सलाहकार के प म े ीय प रषद के साथ
जोड़ा जा सकता है (अथात, बैठक म मतदान के अ धकार के बना): (i) योजना आयोग
वारा ना मत यि त; (ii) े म येक रा य क सरकार के मु य स चव; और (iii) े
म येक रा य के वकास आयु त।
क सरकार के गह
ृ मं ी पांच े ीय प रषद के अ य होते ह। येक मु यमं ी बार -
बार से प रषद के उपा य के प म काय करता है , एक समय म एक वष क अव ध
के लए पद धारण करता है ।
ोत: एम. ल मीकांत वारा भारतीय राजनी त, छठा सं करण- अ याय- 14।

23. भारत के उपरा प त के संबंध म, न न ल खत कथन पर वचार क िजएः


1. संसद के दोन सदन के नवा चत और मनोनीत दोन सद य उपरा प त के
चन
ु ाव म भाग लेते ह।
2. उ ह रा य सभा वारा कुल सं या के 2/3 बहुमत से पा रत ताव वारा हटाया
जा सकता है ।
3. उपरा प त, रा यसभा के सभाप त के प म काय करते हुए, मत क समानता
के मामले म मतदान कर सकता है ।
ऊपर दए गए कथन म से कौन-सा/से सह है /ह?
(a) केवल 1 और 3
(b) केवल 1 और 2
(c) केवल 2
(d) 1, 2 और
उ र: a
या या:
कथन 1 सह है : भारत के उपरा प त का चन
ु ाव एक नवाचक मंडल के सद य वारा
कया जाता है िजसम संसद के दोन सदन के नवा चत और मनोनीत दोन सद य होते ह
। यहां यह यान दया जाना चा हए क उपरा प त के चन
ु ाव के लए नवाचक मंडल म
रा य वधानसभाओं के सद य शा मल नह ं होते ह।
कथन 2 गलत है : उपरा प त अपने कायालय म वेश करने क तार ख से पांच साल क
अव ध के लए पद धारण करता है । हालाँ क, वह कसी भी समय रा प त को याग प
संबो धत करके अपने पद से इ तीफा दे सकता है । उ ह कायकाल पूरा होने से पहले पद
से हटाया भी जा सकता है । उसे हटाने के लए औपचा रक महा भयोग क आव यकता
नह ं है । अनु छे द 67 (सी) के ावधान के अनुसार, उ ह रा यसभा के सभी त काल न
सद य के बहुमत से पा रत ताव वारा हटाया जा सकता है और लोकसभा वारा
सहम त य त क जा सकती है । इसका मतलब है क यह ताव रा यसभा म भावी
बहुमत से और लोकसभा म साधारण बहुमत से पा रत होना चा हए ।
कथन 3 सह है : अ य के वपर त, जो नचले सदन का सद य होता है , सभाप त
(उपरा प त) उ च सदन का सद य नह ं होता है । ले कन अ य क तरह, सभाप त भी
पहल बार म मतदान नह ं कर सकते। वोट क समानता के मामले म वह भी वोट डाल
सकता है ।
ोत: एम. ल मीकांत वारा भारतीय राजनी त, छठा सं करण- अ याय-18, 22.

24. भारत के उपरा प त के संबंध म न न ल खत कथन पर वचार कर:


1. उपरा प त कायवाहक रा प त के प म काय का नवहन करते हुए
रा यसभा के सभाप त के कत य का पालन कर सकता है ।
2. अमे रक उपरा प त के वपर त, भारतीय उपरा प त केवल तभी कायवाहक
रा प त के प म काय करता है जब रा प त का पद र त हो जाता है ।
ऊपर दए गए कथन म से कौन-सा/से सह है /ह?
(a) केवल 1
(b) केवल 2
(c) दोन 1 और 2
(d) न तो 1 और 2
उ र: b
या या:
कथन 1 गलत है : उपरा प त भारत के रा प त के प म काय कर सकता है जब
रा प त के कायालय म उनके इ तीफे, महा भयोग, म ृ यु या अ य कारण से रि त
होती है । वह केवल छह मह ने क अ धकतम अव ध के लए रा प त के प म काय
कर सकता है िजसके भीतर एक नया रा प त चन
ु ना होता है । इसके अलावा, जब
रा पत अनुपि थ त, बीमार या कसी अ य कारण से अपने काय का नवहन करने
म असमथ होता है , तब उपरा प त रा प त के काय का नवहन तब तक करता है जब
तक क रा प त अपना पद फर से शु नह ं कर दे ता।
रा प त के प म काय करते हुए या रा प त के काय का नवहन करते हुए,
उपरा प त रा य सभा के सभाप त के कायालय के कत य का पालन नह ं करता है । इस
अव ध के दौरान, उन कत य का पालन रा य सभा के उपसभाप त वारा कया जाता है ।
कथन 2 सह है : भारतीय उपरा प त का कायालय अमे रक उपरा प त क तज पर
बनाया गया है , इसम अंतर है । अमे रक उपरा प त रा पत र त होने पर रा पत
पद पर आसीन होता है, और अपने पूववत क असमा त अव ध के लए रा प त बना
रहता है । दस
ू र ओर, भारतीय उपरा प त, रा प त का पद हण नह ं करता है , जब वह
असमा त अव ध के लए खाल हो जाता है । वह केवल एक कायवाहक रा प त के प
म काय करता है जब तक क नया रा प त कायभार हण नह ं करता।
ोत: एम. ल मीकांत वारा भारतीय राजनी त, छठा सं करण- अ याय-18।

25. न न ल खत कथन पर वचार कर:


1. संसद य परं परा के अनुसार, रा प त मं प रषद वारा द गई सलाह के
अनुसार काय करता है ।
2. 91व सं वधान संशोधन अ ध नयम ने मं प रषद क यूनतम सं या लोकसभा
क कुल सं या का 15% नधा रत क है ।
3. संसद के कसी भी सदन का एक सद य जो दलबदल के आधार पर अयो य
हो जाता है , वह मं ी के प म नयु त होने के लए अयो य हो जाता है ।
ऊपर दए गए कथन म से कौन सा सह नह ं है ?
(a) केवल 1 और 2
(b) केवल 1 और 3
(c) केवल 2 और 3
(d) 1, 2, और 3
उ र: a
या या:
कथन 1 गलत है : अनु छे द 74 रा प त को अपने काय के अ यास म सहायता और
सलाह दे ने के लए धान मं ी क अ य ता म एक मं प रषद का ावधान करता
है । इसके अलावा, 1976 के 42व सं वधान संशोधन अ ध नयम म यह ावधान था क
रा प त अपने काय के अ यास म, मं प रषद वारा द गई सलाह के अनुसार काय
करे गा। 1978 के 44व सं वधान संशोधन अ ध नयम ने इस अनु छे द म एक परं तुक जोड़ा
क रा पत ऐसी सलाह पर पन
ु वचार करने के लए मं प रषद को कह सकते है ।और
रा प त ऐसे पुन वचार के बाद द गई सलाह के अनुसार काय करे गा। इसके अलावा,
1971 म, सु ीम कोट ने कहा क 'लोकसभा के वघटन के बाद भी, मं प रषद का पद
समा त नह ं होता है । अनु छे द 74 अ नवाय है और इस लए रा प त मं प रषद क
सहायता और सलाह के बना कायकार शि त का योग नह ं कर सकता है । सहायता
और सलाह के बना कायकार शि त का कोई भी योग असंवैधा नक होगा य क यह
अनु छे द 74 का उ लंघन है ।
कथन 2 गलत है : मं प रषद म धान मं ी स हत मं य क कुल सं या लोकसभा क
कुल सं या के 15% से अ धक नह ं होनी चा हए । यह ावधान 2003 के 91व सं वधान
संशोधन अ ध नयम वारा जोड़ा गया था। इसके वारा मं प रषद क अ धकतम सं या
तय क गई है , यूनतम नह ं।
कथन 3 सह है : 2003 के 91व सं वधान संशोधन अ ध नयम ने यह भी ावधान कया है
क, कसी भी राजनी तक दल से संबं धत संसद के कसी भी सदन का सद य जो
दलबदल के आधार पर अयो य है , वह भी मं ी के प म नयु त होने के लए अयो य
होगा ।
ोत: एम. ल मीकांत वारा भारतीय राजनी त -6 वां सं करण-अ याय -20।

26. कै बनेट स म तय के संबंध म, न न ल खत कथन पर वचार कर:


1. धान मं ी संसद क सभी कै बनेट स म तय के अ य के प म काय करता
है ।
2. कै बनेट स म तय म गैर-कै बनेट मं ी भी शा मल हो सकते ह, ज र नह ं क
केवल कै बनेट मं ी ह ह ।
ऊपर दए गए कथन म से कौन-सा/से सह है /ह?
(a) केवल 1
(b) केवल 2
(c) दोन 1 और 2
(d) न तो 1 और न ह 2

उ र: b
या या:
कथन 1 गलत है : कै बनेट स म तयां अपने आप म गैर-संवैधा नक ह । दस
ू रे श द म,
उनका सं वधान म उ लेख नह ं है । हालाँ क, काय नयम उनक थापना के लए ावधान
करते ह। ये दो कार क होती ह- थायी और तदथ। पव
ू थायी कृ त क ह जब क
बाद वाले अ थायी कृ त क ह। वशेष सम याओं को सल
ु झाने के लए समय-समय
पर तदथ स म तय का गठन कया जाता है । काय परू ा होने के बाद उ ह भंग कर दया
जाता है ।
न न ल खत चार अ धक मह वपूण कै बनेट स म तयां ह:
राजनी तक मामल क स म त घरे लू और वदे शी मामल से संबं धत सभी नी तगत
मामल को दे खती है ।
आ थक मामल क स म त आ थक े म सरकार ग त व धय का नदशन और
सम वय करती है ।
 नयुि त स म त क य स चवालय, सावज नक उ यम , बक और व ीय
सं थान म सभी उ च तर य नयुि तय का फैसला करती है ।
 संसद य काय स म त संसद म सरकार कामकाज क ग त को दे खती है ।
राजनी तक मामल , आ थक मामल और नयुि त स म तय क अ य ता धान मं ी
करते ह, जब क संसद य मामल क स म त क अ य ता गह
ृ मं ी करते ह। कसी भी
वषय म, य द धान मं ी कसी स म त का सद य है , तो वह हमेशा उसक अ य ता
करता है ।
कथन 2 सह है : समय क आव यकता और ि थ त क आव यकताओं के अनुसार धान
मं ी वारा कै बनेट स म तय का गठन कया जाता है । इस लए, उनक सं या, नामकरण
और संरचना समय-समय पर बदलती रहती है । उनक सद यता तीन से आठ तक होती
है । इनम आमतौर पर केवल कै बनेट मं ी शा मल होते ह। हालाँ क, गैर-कै बनेट मं य को
उनक सद यता से वं चत नह ं कया जाता है ।
ोत: एम. ल मीकांत वारा राजनी त, छठा सं करण
-अ याय-21।

27. भारत क संसद के संदभ म, न न ल खत म से कौन सी संसद य स म त जांच करती है और


सदन को रपोट करती है क या नयम, नयम, उप- नयम, उप- नयम, आ द बनाने क शि तयां
सं वधान वारा द ह या संसद वारा यायोिजत ह। ऐसे त न धमंडल के दायरे म
कायपा लका वारा उ चत प से योग कया जा रहा है ?
)A) सरकार आ वासन पर स म त
)B) अधीन थ वधान पर स म त
)C) नयम स म त
)D) यापार सलाहकार स म त

उ र: B

या या:
मोटे तौर पर संसद य स म तयां दो कार क होती ह- थायी स म तयां और तदथ स म तयां।
पहला थायी है (हर साल या समय-समय पर ग ठत) और नरं तर आधार पर काम करता है , जब क
बाद वाला अ थायी होता है और उ ह स पे गए काय के पूरा होने पर अि त व म रहता है ।
वक प (b) सह है : यह स म त जांच करती है और सदन को सू चत करती है क या कायपा लका
सं वधान वारा दए गए अ धकार का सह उपयोग कर रह है या संसद वारा व नयम , नयम ,
उप- नयम और उप- नयम को अ ध नय मत करने के लए यायोिजत है । स म त के दोन
सदन म 15 सद य ह।
यह स म त उन गारं टय , वाद और उप म क समी ा करती है जो मं ी कभी-कभी सदन के
पटल पर करते ह और एक रपोट दान करते ह क उनका कतनी अ छ तरह पालन कया गया
है । रा यसभा म इसके कुल 10 सद य और लोकसभा म 15 सद य ह। यह स म त या के
नयम क जांच करती है और सदन म यवसाय कैसे संचा लत होता है और कसी भी आव यक
प रवतन या प रवधन के लए सफा रश करता है । यह स म त सदन के एजडे और काय म को
नयं त करती है । यह सरकार को सदन के सम लाए जाने वाले वधायी और अ य मामल के
संचालन के लए समय आवं टत करता है ।

ोत: यूपीएससी सीएसई पीवाई यू 2018

28. भारतीय यायपा लका के संदभ म, न न ल खत कथन पर वचार क िजएः


1. भारत के सव च यायालय के कसी भी सेवा नव ृ यायाधीश को भारत के रा पतक
पव
ू अनम
ु त से भारत के मु य यायाधीश वारा बैठने और सव च यायालय के
यायाधीश के प म काय करने के लए वापस बल
ु ाया जा सकता है ।
2. भारत म एक उ च यायालय को अपने वयं के नणय क समी ा करने क शि त है
जैसा क सव च यायालय करता है ।
ऊपर दए गए कथन म से कौन-सा/से सह है /ह?
)A) केवल 1
)B) केवल 2
)C) दोन 1 और 2
)D) न तो 1 और न ह 2

उ र: A

या या:
कथन 1 सह है : भारत का मु य यायाधीश अनुरोध कर सकता है क एक सेवा नव ृ सव च
यायालय के यायाधीश या एक सेवा नव ृ उ च यायालय के यायाधीश (सव च यायालय के
यायाधीश के प म नयुि त के लए व धवत प से यो य) एक सी मत समय के लए सव च
यायालय के यायाधीश के प म काय कर। वह ऐसा केवल रा प त और नयु त कए जाने
वाले यि त दोन क पूवानुम त से ह कर सकता है । ऐसा यायाधीश रा प त वारा उ चत
समझे जाने वाले सभी भ का हकदार है । उसके पास सव च यायालय के यायधीश के सभी
अ धकार े , शि तयां और वशेषा धकार भी ह गे।
कथन 2 गलत है: एक उ च यायालय, रकॉड क अदालत के प म, अपने वयं के नणय, आदे श
या नणय को सह करने का अ धकार रखता है , इस त य के बावजद
ू क सं वधान वारा इसे
समी ा क कोई व श ट शि त दान नह ं क गई है ।

ोत: यूपीएससी सीएसई पीवाई यू 2021

29. भारतीय रा प त क वीटो शि तय के संबंध म न न ल खत कथन पर वचार कर:


1. रा प त धन वधेयक के मामले म नलि बत वीटो का योग नह ं कर सकता।
2. सं वधान संशोधन वधेयक के मामले म रा प त के पास वीटो पावर नह ं है ।
3. अमे रक रा प त के वपर त, िज ह 10 दन के भीतर बल को फर से भेजना होता है ,
भारतीय रा प त के पास ऐसा कोई समय- नयम नह ं है ।
इनम से कौन सा/से कथन सह नह ं है/ह?
(A) केवल 1
(B) केवल 2
(C) केवल 1 और 3
(D) उपरो त म से कोई नह ं
उ र: D
या या:
कथन 1 सह है: रा प त धन वधेयक के मामले म नलि बत वीटो का योग नह ं कर सकते।
वह या तो अपनी सहम त रोक सकता है या उसक पिु ट कर सकता है । ले कन वह इसे पन
ु वचार के
लए वापस नह ं भेज सकता।
कथन 2 सह है: सं वधान संशोधन वधेयक के मामले म, रा प त के पास अपनी सहम त दे ने के
अलावा कोई वक प नह ं है । सं वधान संशोधन वधेयक के संबंध म रा प त के पास कोई वीटो
शि त नह ं है । 1971 के 24व सं वधान संशोधन अ ध नयम ने रा प त के लए एक संवैधा नक
संशोधन वधेयक पर अपनी सहम त दे ना अ नवाय कर दया।
कथन 3 सह है: भारतीय रा प त क पॉकेट वीटो पावर के बारे म त य इस कार ह।
1. वधेयक को रा प त वारा अ नि चत काल के लए लं बत रखा जा सकता है , जब वह
अपनी जेब से वीटो का योग करता है ।
2. वह न तो बल को अ वीकार करता है और न ह पुन वचार के लए बल लौटाता है ।
3. सं वधान रा प त के लए कोई समय-सीमा नधा रत नह ं करता है िजसके भीतर उसे
वधेयक पर कारवाई करनी होती है ।
इस लए, रा प त अपने पॉकेट वीटो का उपयोग करता है जहां उसे बल पर कारवाई करने क
आव यकता नह ं होती है । अमे रक रा प त के वपर त, िज ह 10 दन के भीतर बल को फर से
भेजना होता है , भारतीय रा प त के पास ऐसा कोई समय-सीमा नह ं है ।

ोत: एम ल मीकांत वारा भारतीय राजनी त छठा सं करण, अ याय-17

30. भारत के रा प त के संबंध म न न ल खत कथन पर वचार कर:


1. य द मं प रषद अपना बहुमत खो दे ती है तो उसके पास लोकसभा को भंग करने क
ववेकाधीन शि तयां ह।
2. मूल सं वधान ने रा प त को मं प रषद क सलाह से बा य नह ं कया बि क 44व
सं वधान संशोधन के मा यम से कया गया।
ऊपर दए गए कथन म से कौन-सा/से सह है /ह?
(A) केवल 1
(B) केवल 2
(C) दोन 1 और 2
(D) न तो 1 और न ह 2

उ र: A
या या:
कथन 1 सह है : हालां क रा प त के पास कोई संवैधा नक ववेका धकार नह ं है , ले कन
उनके पास कुछ ि थ तज य ववेका धकार ह। दस
ू रे श द म, रा पत न न ल खत
प रि थ तय म अपने ववेक से (अथात मं य क सलाह के बना) काय कर सकता है :
1. धान मं ी क नयिु त जब लोकसभा म कसी भी दल के पास प ट बहुमत
नह ं होता है या जब धान मं ी क अचानक मृ यु हो जाती है और कोई प ट
उ रा धकार नह ं होता है ।
2. मं प रषद क बखा तगी जब वह लोकसभा के व वास को सा बत नह ं कर
सकती।
3. लोकसभा का वघटन य द मं प रषद ने अपना बहुमत खो दया है ।
कथन 2 गलत है : 1976 के 42व सं वधान संशोधन अ ध नयम (इं दरा गांधी सरकार
वारा अ ध नय मत) ने रा प त को धान मं ी क अ य ता वाल मं प रषद क
सलाह से बा य कया। 1978 के 44व सं वधान संशोधन अ ध नयम (मोरारजी दे साई के
नेत ृ व वाल जनता पाट सरकार वारा अ ध नय मत) ने रा प त को इस तरह क
सलाह पर आम तौर पर या अ यथा पुन वचार करने के लए मं प रषद क आव यकता
के लए अ धकृत कया। हालां क, वह इस तरह के पुन वचार के बाद द गई सलाह के
अनुसार काय करे गा। दस
ू रे श द म, रा प त अपने मं य के पुन वचार के लए एक
बार मामले को वापस कर सकता है , ले कन पुन वचार क सलाह बा यकार होगी।

ोत: एम ल मीकांत वारा भारतीय राजनी त छठा सं करण, अ याय-17

31. चन
ु ाव क फ ट पा ट द पो ट (FPTP) णाल और आनुपा तक त न ध व (PR)
णाल के बीच न न ल खत अंतर पर वचार कर:

1. FPTP णाल म, येक नवाचन े एक त न ध का चन


ु ाव करता है जब क PR
णाल म, एक नवाचन े से एक से अ धक त न ध चन
ु े जा सकते ह।
2. FPTP णाल म, एक मतदाता एक उ मीदवार को वोट दे ता है जब क PR स टम म,
एक मतदाता पाट को वोट दे ता है ।
3. FPTP णाल म, एक पाट को वधा यका म वोट के अनुपात से अ धक सीट मल
सकती ह, जब क PR णाल म येक पाट को मले वोट के तशत के अनुपात म
वधा यका म सीट मलती ह।
ऊपर दए गए कथन म से कौन-से सह ह?
(A) केवल 1 और 2
(B) केवल 2 और 3
(C) केवल 1 और 3
(D) 1, 2 और 3

उ र: D
या या:
वक प (D) सह है : फ ट पा ट द पो ट (एफपीट पी) और आनुपा तक त न ध व के
बीच बु नयाद अंतर ह:
तुलना के आधार फ ट पा ट द पो ट आनुपा तक तनध व

अथ फ ट पा ट द पो ट एक वो टंग आनुपा तक तनध व


स टम है , िजसम लोग अपनी एक चन
ु ावी उपकरण है
पसंद के उ मीदवार को वोट दे ते ह िजसम राजनी तक दल
और सबसे यादा वोट पाने वाले को उनके लए डाले गए
क जीत होती है । मत क सं या के आधार
पर सीट आवं टत क
जाती ह।

चन
ु ाव े पूरे दे श को व भ न भौगो लक बड़े भौगो लक े को
इकाइय , यानी नवाचन े म नवाचन े कहा जाता
वभािजत कया गया है । है ।

तनध येक नवाचन े से एक एक नवाचन े से एक


त न ध चन
ु ा जाता है । या अ धक त न ध चन
ु े
जा सकते ह।

मतदान उ मीदवार के लए वोट डाले जाते पाट के लए वोट डाले


ह। जाते ह।

सीट वोट ा त सीट के बराबर हो भी एक पाट को सीट मलती


सकते ह और नह ं भी। ह, वोट के अनुपात के
अनुसार उसे मलता है ।
बहुलता जीतने वाले उ मीदवार को बहुमत जीतने वाले उ मीदवार को
नह ं मल सकता है । बहुमत मलता है ।

जवाबदे ह मौजूद अि त व म नह ं है

बल नह ं होता बल हो सकता है

वचार का
टकराव

ोत: एनसीईआरट क ा XI - काम पर भारतीय सं वधान - अ याय 3

32. न न ल खत म से कौन सा संसद सद य क अयो यता के लए प ट आधार ह


जैसा क भारत के सं वधान के अनु छे द 102 के तहत दान कया गया है ?
1. य द वह संसद वारा बनाए गए कसी कानून के तहत अयो य हो जाता है तो
उसे नवा चत होने के लए अयो य घो षत कर दया जाता है ।
2. य द वह अनु मो चत दवा लया है , तो उसे नवा चत होने के लए अयो य घो षत
कर दया जाता है ।
3. य द वह समय के भीतर अपने चन
ु ाव खच का लेखा-जोखा तुत करने म वफल
रहता है , तो उसे नवा चत होने के लए अयो य घो षत कर दया जाता है ।
4. य द व भ न समूह के बीच श त
ु ा को बढ़ावा दे ने के लए दोषी ठहराया जाता है
तो उसे चन
ु े जाने के लए अयो य घो षत कया जाता है ।
नीचे दए गए कूट का योग कर सह उ र चु नए:
(A) केवल 1 और 2
(B) केवल 1, 2 और 3
(C) केवल 2 और 3
(D) केवल 3 और 4
उ र: A
या या:
कथन 1 और 2 सह ह: भारतीय सं वधान का अनु छे द 102 संसद के कसी भी सदन
के सद य के अयो य होने क शत था पत करता है । य द सद य बताए गए नयम का
पालन करने म वफल रहता है , तो उसे अयो य घो षत कर दया जाएगा।
1. वह भारत सरकार या कसी रा य सरकार (एक मं ी या संसद वारा छूट ा त
कसी अ य कायालय को छोड़कर) के भीतर एक लाभ कमाने वाला कायालय
रखता है ।
2. स म यायालय वारा उसे वकृत च पाया गया है ,
3. उसे अनु मो चत दवा लया घो षत कया गया है ,
4. वह एक भारतीय नाग रक नह ं है , या य द उसने जानबूझकर कसी अ य दे श क
नाग रकता ा त क है , या य द उसने कसी अ य दे श के त न ठा या पालन
क त ा क है ;
5. संसद वारा पा रत कोई भी कानून उसे अयो य घो षत करता है ।
कथन 3 गलत है : चन
ु ाव म अपने खच का लेखा-जोखा तुत करने म वफल रहने के
कारण अयो यता मानदं ड का उ लेख सं वधान (अनु छे द 102) म नह ं है , बि क लोक
त न ध व अ ध नयम (1951) म दया गया है ।
कथन 4 गलत है : य द कसी यि त को व भ न समूह के बीच श त
ु ा को बढ़ावा दे ने
और र वतखोर के अपराध म दोषी ठहराया जाता है , तो उसे लोक तनध व
अ ध नयम (1951) के अनस
ु ार अयो य घो षत कया जाना चा हए, न क भारतीय
सं वधान के अनु छे द 102 के तहत।

ोत: एम ल मीकांत वारा भारतीय राजनी त 6 वां सं करण, अ याय -22, 72

33. सं वधान क दसवीं अनुसूची के तहत, न न ल खत म से कौन आम तौर पर


अयो यता क ओर नह ं ले जाएगा?
(A) य द कसी सदन का कोई सद य वे छा से अपने राजनी तक दल क सद यता
छोड़ दे ता है
(B) य द सदन का कोई सद य अपने राजनी तक दल के नदश के वपर त मतदान
करता है ।
(c) य द कोई नदल य उ मीदवार चन
ु ाव के बाद कसी राजनी तक दल म शा मल हो
जाता है ।
(D) य द कोई मनोनीत सद य सदन का सद य बनने के छह मह ने के भीतर पाट म
शा मल हो जाता है ।

उ र: D
या या:
वक प (d) सह है : भारतीय सं वधान के दलबदल वरोधी कानून (दसवीं अनुसूची) के
तहत दान क गई अयो यता के ावधान इस कार ह -
य द कसी राजनी तक दल से संबं धत सदन का सद य:
1. वे छा से अपने राजनी तक दल क सद यता छोड़ दे ता है , या
2. अपने राजनी तक दल के नदश के वपर त, वधा यका म वोट दे ता है या नह ं
दे ता है । हालां क, य द सद य ने पूव अनुम त ल है , या ऐसे मतदान या
अनुपि थत रहने से 15 दन के भीतर पाट वारा माफ कर दया जाता है , तो
सद य को अयो य घो षत नह ं कया जाएगा।
3. य द कोई नदल य उ मीदवार चन
ु ाव के बाद कसी राजनी तक दल म शा मल हो
जाता है ।
य द कोई मनोनीत सद य वधा यका का सद य बनने के छह मह ने बाद (अंदर नह ं)
पाट म शा मल होता है ।

ोत: एम ल मीकांत वारा भारतीय राजनी त छठा सं करण, अ याय-76

34. न न ल खत कथन पर वचार कर:


1. अंतर-रा य प रषद क थापना संसद के एक अ ध नयम वारा संबं धत रा य के अनरु ोध पर क
जाती है ।
2. रा प त अंतर-रा य प रषद और उसके संगठन और उसक या वारा कए जाने वाले कत य
क कृ त को प रभा षत कर सकते ह।
3. संसद अंतरा यीय प रषद को कत य स पती है और दशा- नदश दान करती है िजसका उसे
अपने कत य का पालन करते समय पालन करना होता है ।
4. रा प त एक अंतर-रा य प रषद क थापना कर सकते ह य द कसी भी समय उ ह यह तीत
होता है क सावज नक हत क सेवा क जाएगी।
ऊपर दए गए कथन म से कौन सा सह नह ं है ?
(A) 1 और 3
(B) केवल 2 और 4
(C) केवल 3 और 4
(D) 2, 3 और 4

उ र: A
या या:
कथन 1 गलत है: भारतीय सं वधान के अनु छे द 263 के ावधान के अनस
ु ार, अंतर-रा य प रषद
क थापना संसद या संसद के एक अ ध नयम वारा नह ं क जाती है , ले कन रा पत वारा,
कसी भी समय उसे ऐसा तीत होता है क इससे सावज नक हत होगा तो ग ठत करता है।
कथन 2 सह है : भारतीय सं वधान के अनु छे द 263 के ावधान के अनस
ु ार, रा प त को अपने
संगठन और याओं के साथ अंतरा यीय प रषद वारा कए जाने वाले कत य क कृ त को
प रभा षत करने का अ धकार है।
कथन 3 गलत है : संसद अंतर-रा य प रषद के कामकाज के लए कत य का नधारण या
दशा नदश दान नह ं कर सकती है ।
कथन 4 सह है : रा प त अंतर-रा यीय ववाद क जांच और सलाह दे ने के लए एक अंतर-रा य
प रषद क थापना कर सकते ह, अगर उ ह कसी भी समय ऐसा लगता है क ऐसी प रषद क
थापना से सावज नक हत क सेवा होगी।

ोत: एम ल मीकांत वारा भारतीय राजनी त छठा सं करण, अ याय -14;


डीडी बसु वारा भारत के सं वधान का प रचय, 23वां सं करण

35. भारत म रा य आपातकाल के संबध


ं म न न ल खत कथन पर वचार कर:
1. 1976 के 42व सं वधान संशोधन अ ध नयम ने रा य आपातकाल क घोषणा को या यक
समी ा से मु त कर दया।
2. मनवा म स मामले म सव च यायालय ने माना क रा य आपातकाल क उ घोषणा को
दभ
ु ावना के आधार पर अदालत म चन
ु ौती द जा सकती है ।
3. आपातकाल क घोषणा को संसद के दोन सदन वारा इसके जार होने क तार ख से दो मह ने
के भीतर अनम
ु ो दत कया जाना चा हए।
ऊपर दए गए कथन म से कौन-सा/से सह है/ह?
(A) केवल 1
(B) केवल 2
(C) केवल 3
(D) केवल 1 और 2
उ र: B
या या:
कथन 1 गलत है: 1975 के 38व संवध
ै ा नक संशोधन अ ध नयम ने रा य आपातकाल क घोषणा
को या यक समी ा से मु त कर दया। ले कन, बाद म इस ावधान को 44व सं वधान संशोधन
अ ध नयम 1978 वारा हटा दया गया।
कथन 2 सह है : मनवा म स मामले, 1980 म, सव च यायालय ने माना क रा य आपातकाल
क उ घोषणा को दभ
ु ावना के आधार पर अदालत म चुनौती द जा सकती है या यह क घोषणा परू
तरह से असंगत और अ ासं गक त य पर आधा रत थी या बेतक
ु ा है या वकृत।
कथन 3 गलत है: आपातकाल क उ घोषणा को संसद के दोन सदन वारा इसके जार होने क
तार ख से एक मह ने के भीतर अनम
ु ो दत कया जाना चा हए। मल
ू प से, संसद वारा अनम
ु ोदन
क अनम
ु त क अव ध दो मह ने थी, ले कन 44 व सं वधान संशोधन अ ध नयम 1978 वारा कम
कर द गई थी। हालां क, य द आपातकाल क घोषणा ऐसे समय म जार क जाती है जब लोकसभा
भंग हो गई हो या संसद का वघटन हो गया हो। लोकसभा एक मह ने क अव ध के दौरान
उ घोषणा को मंजरू दए बना होती है , फर उ घोषणा लोकसभा क पहल बैठक से उसके पन
ु गठन
के बाद 30 दन तक जी वत रहती है , बशत रा य सभा ने इस बीच इसे मंजूर दे द हो।

ोत: एम ल मीकांत वारा भारतीय राजनी त छठा सं करण, अ याय-16

36. क -रा य संबध


ं पर रा य आपातकाल के भाव के संबध
ं म न न ल खत कथन पर वचार
कर:
1. रा य आपातकाल के दौरान, संसद को रा य सच
ू ी म उि ल खत कसी भी वषय पर कानन

बनाने का अ धकार मल जाता है ।
2. रा य आपातकाल के दौरान रा य के वषय पर संसद वारा बनाए गए कानन
ू आपातकाल
समा त होने के छह मह ने बाद नि य हो जाते ह।
3. संसद क वधायी शि तयाँ न केवल उस रा य तक फैल हुई ह जहाँ आपातकाल लागू है बि क
कसी अ य रा य म भी।
ऊपर दए गए कथन म से कौन-से सह ह?
(A) केवल 1 और 2
(B) केवल 2 और 3
(C) केवल 1 और 3
(D) 1, 2 और 3
उ र: D
या या:
कथन 1 सह है : रा य आपातकाल के दौरान, संसद को रा य सच
ू ी म उि ल खत कसी भी वषय
पर कानन
ू बनाने का अ धकार मल जाता है । य य प रा य वधानमंडल क वधायी शि त नलं बत
नह ं है , यह संसद क अ धभावी शि त के अधीन हो जाती है । इस कार, क और रा य के बीच
वधायी शि तय का सामा य वतरण नलं बत है , हालां क रा य वधानमंडल नलं बत नह ं ह।
सं ेप म, सं वधान संघीय के बजाय एका मक हो जाता है।
कथन 2 सह है : रा य आपातकाल के दौरान रा य के वषय पर संसद वारा बनाए गए कानन

आपातकाल के समा त होने के छह मह ने बाद नि य हो जाते ह। वशेष प से, जब क रा य
आपातकाल क उ घोषणा चल रह है , रा प त रा य के वषय पर भी अ यादे श जार कर सकते ह,
य द संसद स म नह ं है ।
कथन 3 सह है : 1976 के 42व संवध
ै ा नक संशोधन अ ध नयम म ावधान है क वधायी प रणाम
और भाव न केवल उस रा य पर लागू होते ह जहां आपातकाल लागू है बि क कसी अ य रा य म
भी लागू होता है ।

ोत 1: एम ल मीकांत वारा भारतीय राजनी त छठा सं करण, अ याय-16


ोत 2: डीडी बसु वारा भारत के सं वधान का प रचय 23वां सं करण

37. भारत म रा प त शासन के संबध


ं म न न ल खत कथन पर वचार कर:
1. अनु छे द 365 रा प त को एक उ घोषणा जार करने का अ धकार दे ता है , य द वह संतु ट है
क ऐसी ि थ त उ प न हो गई है िजसम कसी रा य क सरकार सं वधान के ावधान के अनस
ु ार
नह ं चल सकती है।
2. अनु छे द 356 म यह ावधान है क जब भी कोई रा य क के कसी नदश का पालन करने या
उसे लागू करने म वफल रहता है , तो रा प त के लए यह मानना वैध होगा क ऐसी ि थ त
उ प न हो गई है िजसम रा य क सरकार सं वधान के ावधान के अनस
ु ार नह चल सकती ।
3. रा प त शासन क घोषणा या इसे जार रखने का अनम
ु ोदन करने वाला येक ताव संसद के
कसी भी सदन वारा साधारण बहुमत से पा रत कया जा सकता है।
ऊपर दए गए कथन म से कौन-सा/से सह नह ं है/ह?
(A) केवल 1
(B) केवल 1 और 2
(C) केवल 2 और 3
(D) 1, 2 और 3

उ र: B
या या:
कसी रा य म रा प त शासन क घोषणा दो आधार पर क जा सकती है - एक अनु छे द 356 म
ह व णत है और दस
ू रा अनु छे द 365 म:
कथन 1 गलत है : अनु छे द 356 रा प त को एक उ घोषणा जार करने का अ धकार दे ता है , य द
वह संतु ट है क ऐसी ि थ त उ प न हो गई है िजसम कसी रा य क सरकार सं वधान के
ावधान के अनस
ु ार नह ं चल सकती है। वशेष प से, रा प त या तो रा य के रा यपाल क
रपोट पर काय कर सकता है या अ यथा भी (अथात रा यपाल क रपोट के बना भी)।
कथन 2 गलत है : अनु छे द 365 कहता है क जब भी कोई रा य क के कसी नदश का पालन
करने या उसे लागू करने म वफल रहता है , तो रा प त के लए यह मानना वैध होगा क ऐसी
ि थ त उ प न हो गई है िजसम रा य क सरकार नह ं हो सकती है । सं वधान के ावधान के
अनस
ु ार कया जाता है ।
कथन 3 सह है : रा प त शासन क उ घोषणा या उसके जार रहने का अनुमोदन करने वाला
येक ताव संसद के कसी भी सदन वारा केवल साधारण बहुमत से पा रत कया जा सकता है ,
अथात उस सदन के उपि थत और मतदान करने वाले सद य के बहुमत से।

ोत: एम ल मीकांत वारा भारतीय राजनी त छठा सं करण, अ याय-16

38. रा प त शासन के दौरान रा पत वारा ा त शि तय के संदभ म, न न ल खत म से कौन


सा कथन स य है ?
1. वह मु यमं ी क अ य ता वाल रा य मं प रषद को बखा त करता है।
2. वह रा य वधान सभा को भंग करता है ।
3. वह घोषणा कर सकता है क रा य वधानमंडल क शि तय का योग संसद वारा कया जाना
है ।
नीचे दए गए कूट का योग कर सह उ र चु नए:
(A) केवल 1
(B) केवल 1 और 3
(C) केवल 2 और 3
(D) 1, 2 और 3

उ र: B
या या:
अनु छे द 355 यह सु नि चत करने के लए क पर एक कत य लगाता है क येक रा य क
सरकार सं वधान के ावधान के अनस
ु ार चलती है । यह वह कत य है िजसके दशन म क रा य
म संवध
ै ा नक तं के वफल होने क ि थ त म अनु छे द 356 के तहत कसी रा य क सरकार को
अपने हाथ म लेता है । इसे लोक य प से 'रा प त शासन' के प म जाना जाता है । इसे 'रा य
आपातकाल' या 'संवध
ै ा नक आपातकाल' के प म भी जाना जाता है।
कथन 1 सह है : जब कसी रा य म रा प त शासन लगाया जाता है , तो रा प त मु यमं ी क
अ य ता वाल रा य मं प रषद को बखा त कर दे ता है। रा य का रा यपाल, रा प त क ओर से,
रा य के मु य स चव या रा पत वारा नयु त सलाहकार क सहायता से रा य शासन का
संचालन करता है । यह कारण है क अनु छे द 356 के तहत एक उ घोषणा को रा य म 'रा पत
शासन' लागू करने के प म जाना जाता है।
कथन 2 गलत है: इसके अलावा, रा प त रा य वधान सभा को या तो नलं बत या भंग कर दे ता
है । संसद रा य के वधायी वधेयक और रा य के बजट को पा रत करती है । वधान सभा भंग हो
भी सकती है और नह ं भी।
कथन 3 सह है : वह घोषणा कर सकता है क रा य वधानमंडल क शि तय का योग संसद
वारा कया जाना है । वह रा य म कसी भी नकाय या ा धकरण से संबं धत संवध
ै ा नक ावधान
के नलंबन स हत अ य सभी आव यक कदम उठा सकता है ।

ोत: एम ल मीकांत वारा भारतीय राजनी त छठा सं करण, अ याय-16

39. भारत के सं वधान के अनु छे द 360 के तहत व ीय आपातकाल के संबध


ं म न न ल खत म
से कौन सा/से कथन सह है /ह?
1. व ीय आपातकाल क उ घोषणा एक मह ने क समाि त पर काम करना बंद कर दे गी, य द तब
तक संसद के दोन सदन के ताव वारा अनम
ु ो दत नह ं कया जाता है।
2. एक बार संसद वारा अनुमो दत व ीय आपातकाल अ नि चत काल तक जार रहता है जब तक
क इसे र द नह ं कया जाता है ।
3. व ीय आपातकाल के दौरान, रा प त सभी सरकार कमचा रय के वेतन और भ म कटौती के
लए नदश जार कर सकते ह, ले कन सव च यायालय और उ च यायालय के यायाधीश को
छोड़कर।
नीचे दए गए कूट का योग कर सह उ र चु नए:
(A) केवल 1 और 2
(B) केवल 2
(C) केवल 2 और 3
(D) 1, 2 और 3

उ र: B
या या:
अनु छे द 360 रा प त को व ीय आपातकाल क घोषणा करने का अ धकार दे ता है य द वह संतु ट
है क ऐसी ि थ त उ प न हुई है िजसके कारण भारत या उसके े के कसी भी ह से क व ीय
ि थरता या े डट को खतरा है ।
कथन 1 गलत है: व ीय आपातकाल क घोषणा करने वाल उ घोषणा को संसद के दोन सदन
वारा इसके जार होने क तार ख से दो मह ने के भीतर अनम
ु ो दत कया जाना चा हए। हालाँ क,
य द व ीय आपातकाल क उ घोषणा ऐसे समय म जार क जाती है जब लोकसभा भंग हो गई हो
या लोकसभा का वघटन उ घोषणा को मंजूर दए बना दो मह ने क अव ध के दौरान होता है , तो
उ घोषणा पहले से 30 दन तक जी वत रहती है । इसके पन
ु गठन के बाद लोकसभा क बैठक, बशत
रा य सभा ने इस बीच इसे मंजूर दे द हो।
कथन 2 सह है: संसद के दोन सदन वारा अनम
ु ो दत होने के बाद, व ीय आपातकाल अ नि चत
काल तक जार रहता है जब तक क इसे र द नह ं कया जाता है। इसका ता पय दो बात से है:
1. इसके संचालन के लए कोई अ धकतम अव ध नधा रत नह ं है ; तथा
2. इसे जार रखने के लए बार-बार संसद य अनम
ु ोदन क आव यकता नह ं है।
कथन 3 गलत है: रा प त (A) संघ क सेवा करने वाले सभी या कसी भी वग के यि तय के
वेतन और भ म कमी के लए नदश जार कर सकते ह; और (B) सव च यायालय और उ च
यायालय के यायाधीश। इस कार, एक व ीय आपातकाल के संचालन के दौरान, क व ीय
मामल म रा य पर पण
ू नयं ण ा त करता है।

ोत:
एम ल मीकांत वारा भारतीय राजनी त, छठा सं करण, अ याय-16
40. न न ल खत म से कौन भारत क संघ कायका रणी का ह सा ह?
1. भारत के रा पत
2. भारत के धानमं ी
3. भारत के उपरा पत
4. भारत के महा यायवाद
5. भारत के कै बनेट स चव
नीचे दए गए कूट का योग कर सह उ र चु नए:
(A) केवल 1 और 3
(B) केवल 2, 4 और 5
(C) केवल 1, 2, 3 और 4
(D) 1, 2, 3, 4 और 5

उ र: C
या या:
वक प (C) सह है: कायपा लका वह अंग है जो वधा यका वारा बनाए गए कानन
ू को लागू करती
है और रा य क इ छा को लागू करती है । भारत क संघ कायका रणी म शा मल ह
1. भारत के रा प त,
2. भारत के उपरा प त,
3. भारत के धान मं ी,
4. मं प रषद और
5. भारत के महा यायवाद ।
भारत का कै बनेट स चव क य कायका रणी का ह सा नह ं है।

ोत: एम ल मीकांत वारा भारतीय राजनी त छठा सं करण, अ याय-17, 20

41. भारत म रा प त के चुनाव के संबध


ं म न न ल खत कथन पर वचार कर:
1. यह एकल सं मणीय मत के मा यम से आनप
ु ा तक तनध व णाल के अनस
ु ार आयोिजत
कया जाता है ।
2. सभी वधायक के वोट का कुल मू य सभी सांसद के वोट के कुल मू य के बराबर है।
3. क शा सत दे श द ल और पड
ु ु चेर क वधानसभाओं के नवा चत सद य भी नवाचक मंडल
का ह सा बनते ह।
ऊपर दए गए कथन म से कौन-से सह ह?
(A) केवल 1 और 2
(B) केवल 2 और 3
(C) केवल 1 और 3
(D) 1, 2 और 3

उ र: D
या या:
कथन 1 सह है : रा प त का चुनाव आनप
ु ा तक तनध व णाल के अनस
ु ार एकल सं मणीय
मत के मा यम से होता है और मतदान गु त मतदान वारा होता है । यह णाल सु नि चत करती
है क सफल उ मीदवार को पण
ू बहुमत से लौटाया जाए।
कथन 2 सह है : सं वधान दान करता है क रा प त के चुनाव म व भ न रा य के तनध व
के पैमाने म एक पता के साथ-साथ रा य और संघ के बीच समानता होगी। इस कार, इस
स धांत के अनु प होने के लए, सभी वधायक के वोट का मू य सभी सांसद के वोट के मू य
के समान है । दो अलग-अलग रा य के वधायक के वोट का मू य भ न हो सकता है य क एक
वधायक के वोट का मू य उसके रा य क जनसं या के सीधे आनप
ु ा तक होता है और उसक
वधानसभा म सीट क सं या के यु मानप
ु ाती होता है ।
कथन 3 सह है : रा प त का चुनाव अ य प से भारत के लोग वारा नवाचक मंडल के
सद य वारा कया जाता है , िजसम शा मल ह:
1. संसद के दोन सदन के नवा चत सद य;
2. रा य क वधानसभाओं के नवा चत सद य; तथा
3. क शा सत दे श द ल और पड
ु ु चेर क वधानसभाओं के नवा चत सद य।

ोत: एम ल मीकांत वारा भारतीय राजनी त छठा सं करण, अ याय-17

42. न न ल खत कथन पर वचार क िजएः


1. भारत के महा यायवाद और भारत के सॉ ल सटर जनरल सरकार के एकमा अ धकार ह िज ह
भारत क संसद क बैठक म भाग लेने क अनम
ु त है।
2. भारत के सं वधान के अनस
ु ार, भारत का महा यायवाद अपना इ तीफा तब दे ता है जब उसे
नयु त करने वाल सरकार इ तीफा दे दे ती है।
ऊपर दए गए कथन म से कौन-सा/से सह है/ह?
(A) केवल 1
(B) केवल 2
(C) दोन 1 और 2
(D) न तो 1 और न ह 2

उ र: D

या या:
कथन 1 गलत है: सं वधान के अनु छे द 76 के अनस
ु ार, केवल महा यायवाद को संसद य कायवाह
म भाग लेने का अ धकार है । यह अ धकार भारत के सॉ ल सटर जनरल तक व ता रत नह ं है।
कथन 2 गलत है: सरकार वारा अटॉन जनरल क नयिु त नह ं क जाती है । इसके बजाय, उ ह
रा पत वारा सरकार के परामश से नयु त कया जाता है। हालां क, परं परागत प से, सरकार के
इ तीफा दे ने के बाद अटॉन जनरल कायालय से इ तीफा दे दे ता है , य क उसे उसक सलाह पर
नयु त कया जाता है ।

ोत: यप
ू ीएससी सीएसई पीवाई यू 2022

43. न न ल खत कथन पर वचार क िजएः


1. क य शास नक याया धकरण (CAT) क थापना लाल बहादरु शा ी के धानमं व काल म
क गई थी।
2. कैट के सद य या यक और शास नक दोन धाराओं से लए गए ह।
ऊपर दए गए कथन म से कौन-सा/से सह है/ह?
(A) केवल 1
(B) केवल 2
(C) दोन 1 और 2
(D) न तो 1 और न ह 2

उ र: B
या या:
कथन 1 सह नह ं है: क य शास नक याया धकरण क थापना सं वधान के अनु छे द 323-A के
तहत, लोक सेवाओं और पद पर नयु त यि तय क भत और सेवा क शत के संबध
ं म ववाद
और शकायत के याय नणयन के लए क गई थी। सरकार के नयं ण म संघ या अ य
ा धकरण। इसक थापना राजीव गांधी के धानमं व काल म हुई थी।
कथन 2 सह है : क य शास नक याया धकरण को एक वशेष नकाय के प म था पत कया
गया है िजसम शास नक सद य और या यक सद य शा मल ह जो अपने वशेष ान के आधार
पर व रत और भावी याय दे ने के लए बेहतर ढं ग से सस
ु ि जत ह।

ोत: यप
ू ीएससी सीएसई पीवाई यू 2009

44. न न ल खत कथन पर वचार क िजएः


1. रा प त भारत सरकार के काय के अ धक सु वधाजनक लेन-दे न के लए और उ त काय के
मं य के बीच आवंटन के लए नयम बनाएंगे।
2. भारत सरकार के सभी कायकार काय को धान मं ी के नाम पर कया जाना य त कया
जाएगा।
ऊपर दए गए कथन म से कौन-सा/से सह है/ह? (2014)
(A) केवल 1
(B) केवल 2
(C) दोन 1 और 2
(D) न तो 1 और न ह 2

उ र: A
या या:
कथन 1 सह है: अनु छे द 77(3) के अनस
ु ार, रा प त भारत सरकार के यवसाय के अ धक
सु वधाजनक लेन-दे न के लए और उ त यवसाय के मं य के बीच आवंटन के लए नयम
बनाएगा।
कथन 2 गलत है: भारत म, रा प त रा य का कानन
ू ी मख
ु होता है । भारत सरकार क सभी
कायकार कारवाइयां रा प त के नाम पर क जाएंगी, न क धानमं ी के नाम पर। धानमं ी
मं प रषद और रा प त के बीच मु य संचारक होता है । वह रा प त को सलाह दे ता है और
मं प रषद क सलाह रा प त पर बा यकार होती है।

ोत: यप
ू ीएससी सीएसई पीवाई यू 2014

45. पंचायती राज चुनाव के संबध


ं म न न ल खत म से कौन सा कथन सह नह ं है ?
(a) म हलाओं के लए कुल सीट क सं या का कम से कम एक तहाई और अ य क कुल
सं या का एक तहाई आर ण अ नवाय है ।
(b) म यवत और िजला तर पर पंचायत के अ य अ य प से चन
ु े जाते ह।
(c) पंचायत के गठन के लए नए चुनाव उसके वघटन क तार ख से छह मह ने क अव ध क
समाि त से पहले परू े कए जाएंगे।
(d) रा य चुनाव आयोग, पंचायत के चुनाव से संबं धत मामल के संबध
ं म ावधान करता है ।

उ र: d
या या:
पंचायती राज चुनाव के संबध
ं म 73व सं वधान संशोधन अ ध नयम, 1992 म न न ल खत
ावधान कये गए ह:
वक प (a) सह है : अ ध नयम म म हलाओं के लए सीट क कुल सं या के कम से कम एक
तहाई आर ण का ावधान कया गया है (अनस
ु ू चत जा त और अनस
ु ू चत जनजा त से संबं धत
म हलाओं के लए आर त सीट क सं या स हत)। इसके अ त र त, येक तर पर पंचायत म
अ य क कुल सं या के एक तहाई से कम पद म हलाओं के लए आर त नह ं ह गे।
वक प (b) सह है : गांव, म यवत और िजला तर पर पंचायत के सभी सद य य प से
लोग वारा चुने जाएंगे। इसके अ त र त, म यवत और िजला तर पर पंचायत के अ य का
चन
ु ाव अ य प से उनके नवा चत सद य वारा और उनम से कया जाएगा। तथा प, ाम
तर पर पंचायत के अ य का चुनाव रा य वधानमंडल वारा नधा रत तर के से कया जाएगा।
वक प (c) सह है : अ ध नयम येक तर पर पंचायत को पांच साल के कायकाल का ावधान
करता है । हालां क, इसका कायकाल परू ा होने से पहले भी इसे भंग कया जा सकता है। इसके
अ त र त, पंचायत के गठन के लए नए सरे से चुनाव परू ा कया जाएगा-
 पांच साल क अव ध क समाि त से पहले; या
 वघटन के मामले म, इसके वघटन क तार ख से छह मह ने क अव ध क समाि त से
पहले।
वक प (d) गलत है : मतदाता सच
ू ी तैयार करने और पंचायत के सभी चुनाव के संचालन का
अधी ण, नदशन और नयं ण रा य चुनाव आयोग म न हत होगा। पर त,ु पंचायत के चुनाव से
संबं धत सभी मामल के संबध
ं म ावधान रा य वधानमंडल वारा कए जाते ह।

ोत: एम ल मीकांत वारा भारतीय राजनी त छठा सं करण, अ याय 38

46. पेसा (पंचायत के ावधान का अनुसू चत े तक व तार) अ ध नयम, 1996 अ ध नयम के


तहत ाम सभा क शि त के संदभ म न न ल खत कथन पर वचार क िजये:
1. येक ाम सभा लोग क परं पराओं और र त- रवाज , उनक सां कृ तक पहचान, सामद
ु ा यक
संसाधन और ववाद समाधान के पारं प रक तर के क सरु ा और संर ण के लए स म होगी ।
2. अनस
ु ू चत े म मख
ु ख नज के लए पव
ू ण अनु ि त प (लाइसस) या खनन प टा दान
करने के लए उपयु त तर पर ाम सभा या पंचायत क सफा रश अ नवाय ह गी।
3. येक ाम सभा गर बी उ मल
ू न और अ य काय म के तहत लाभा थय क पहचान के लए
उ रदायी होगी।
ऊपर दए गए कथन म से कौन-सा/से सह है/ह?
(a) केवल 1
(b) केवल 1 और 3
(c) केवल 2 और 3
(d) 1, 2 और 3

उ र: b
या या:
वक प (b) सह है : पेसा (पंचायत के ावधान का अनस
ु ू चत े तक व तार) अ ध नयम, क
वशेषताएं (या ावधान) इस कार ह:
 अनस
ु ू चत े म पंचायत पर रा य का वधान, थागत कानन
ू , सामािजक और धा मक
थाओं और सामद
ु ा यक संसाधन के पारं प रक बंधन थाओं के अनु प होगा।
 एक गांव म सामा यतः एक समद
ु ाय का वास थल या वास थल का समह
ू या एक टोला
या टोल का एक समह
ू होता है , िजसम एक समद
ु ाय अपने परं पराओं और र त- रवाज के
अनस
ु ार जीवन यापन करता है ।
 येक गांव म एक ाम सभा होगी, िजसम ऐसे यि त ह गे, िजनके नाम ाम तर पर
पंचायत के लए नवाचक सच
ू ी म दज ह ।
 येक ाम सभा लोग क परं पराओं और र त- रवाज , उनक सां कृ तक पहचान,
सामद
ु ा यक संसाधन और ववाद नवारण के पारं प रक तर के क सरु ा और संर ण के लए
स म होगी।
 येक ाम सभा
o सामािजक और आ थक वकास के योजनाओं, काय म और प रयोजनाओं को
वीकृ त दे गी, इससे पहले क ाम तर पर पंचायत वारा लागू करने के लए हाथ
म लए जाएँ तथा
o गर बी उ मल
ू न और अ य काय म के तहत लाभा थय क पहचान के लए
िज मेदार होगी।
 ाम तर पर येक पंचायत को उपरो त योजनाओं, काय म और प रयोजनाओं के लए
न धय के उपयोग का माण प ाम सभा से ा त करे गी।
 येक पंचायत म अनस
ु ू चत े म सीट का आर ण उन समद
ु ाय क जनसं या के
अनप
ु ात म होगा, िजनके लए सं वधान के भाग IX के तहत आर ण क यव था क गयी
है ।
 तथा प, अनस
ु ू चत जनजा तय के लए आर ण, कुल सीट क सं या के आधे से कम नह ं
होगा। इसके अ त र त, पंचायत के सभी तर पर अ य क सभी सीट अनुसू चत
जनजा तय के लए आर त ह गी।
 अनस
ु ू चत े म उपख नज ( मख
ु नह ं) के लए पव
ू ण अनु ि त प या खनन प टा
दान करने के लए उपयु त तर पर ाम सभा या पंचायत क सफा रश अ नवाय ह गी।

ोत: एम ल मीकांत वारा भारतीय राजनी त छठा सं करण, अ याय 38

47. भारत के रा प त के संदभ म, न न ल खत म से कौन सा/से कथन सह नह ं है/ह?


1. रा प त के लए शपथ सं वधान क अनस
ु च
ू ी III म मं प रषद क शपथ के साथ नधा रत है ।
2. रा प त अपने कायकाल के दौरान द वानी और आपरा धक कायवाह से परू तरह से मु त है ।
3. सं वधान म प रभा षत सं वधान के उ लंघन के आधार पर रा प त पर महा भयोग चलाया जा
सकता है ।
नीचे दए गए कूट का योग कर उ र चु नए।
(A) केवल 1
(B) केवल 1 और 2
(C) केवल 2 और 3
(D) 1, 2 और 3

उ र: D
या या:
कथन 1 गलत है: मं प रषद क शपथ सं वधान क अनस
ु च
ू ी III म मौजद
ू है। मं प रषद कायालय
और गोपनीयता क शपथ लेती है । हालाँ क, भारतीय सं वधान के अनु छे द 60 म रा पत क
'सं वधान क र ा, र ा और बचाव' क शपथ दान क गई है ।
कथन 2 गलत है: अपने कायकाल के दौरान, वह कसी भी आपरा धक कायवाह से मु त है , यहां
तक क अपने यि तगत कृ य के संबध
ं म भी। उसे गर तार या कैद नह ं कया जा सकता है।
हालाँ क, दो मह ने का नो टस दे ने के बाद, उनके यि तगत काय के संबध
ं म उनके कायकाल के
दौरान उनके खलाफ स वल कायवाह शु क जा सकती है।
कथन 3 गलत है: रा प त को अनु छे द 61 के अनस
ु ार 'सं वधान के उ लंघन' के लए महा भयोग
क या वारा पद से हटाया जा सकता है । हालां क, सं वधान 'सं वधान के उ लंघन' वा यांश के
अथ को प रभा षत नह ं करता है ।

ोत: एम ल मीकांत वारा भारतीय राजनी त छठा सं करण, अ याय-17

48. रा प त के महा भयोग के संदभ म, नीचे दया गया कौन सा कथन सह नह ं है ?


(A) इसे लोकसभा के साथ-साथ रा यसभा म भी शु कया जा सकता है।
(B) दोन सदन के मनोनीत सद य इस या म भाग लेते ह, भले ह वे रा प त का चन
ु ाव करने
के लए नवाचक मंडल का ह सा न ह ।
(C) रा प त को 'कदाचार या अ मता सा बत' के लए इस या वारा पद से हटाया जा सकता
है ।
(D) रा य क वधानसभाओं के नवा चत सद य इसम भाग नह ं लेते ह, भले ह वे रा प त का
चुनाव करने के लए नवाचक मंडल का ह सा ह ।

उ र: C
या या:
वक प (A) सह है: महा भयोग क कायवाह संसद के कसी भी सदन वारा शु क जा सकती है ।
इन आरोप पर सदन के एक चौथाई सद य वारा ह ता र कए जाने चा हए िज ह ने महा भयोग
क कायवाह शु क थी। उदाहरण के लए, लोकसभा म इसे 543 सद य म से एक-चौथाई वारा
ह ता रत कया जाना चा हए।
वक प (B) और (D) सह ह: संसद म महा भयोग एक अध- या यक या है । इस संदभ म दो
बात का यान रखना चा हए:
1. संसद के कसी भी सदन के मनोनीत सद य रा प त के महा भयोग म भाग ले सकते ह,
हालां क वे उसके चन
ु ाव म भाग नह ं लेते ह;
2. रा य और क शा सत दे श द ल और पड
ु ु चेर क वधानसभाओं के नवा चत सद य रा पत
के महा भयोग म भाग नह ं लेते ह, हालां क वे उनके चुनाव म भाग लेते ह।
वक प (C) गलत है: रा प त को अनु छे द 61 के अनस
ु ार 'सं वधान के उ लंघन' के लए
महा भयोग क या वारा पद से हटाया जा सकता है । हालां क, सं वधान 'सं वधान के उ लंघन'
वा यांश के अथ को प रभा षत नह ं करता है ।

ोत: एम ल मीकांत वारा भारतीय राजनी त छठा सं करण, अ याय-17


49. वभागीय थायी स म तय के संदभ म, न न ल खत कथन पर वचार क िजएः

1. येक थायी स म त म लोकसभा और रा य सभा दोन से समान सं या म सद य होते ह।


2. कोई भी मं ी कसी भी थायी स म त के सद य के प म मनोनीत होने के यो य नह ं है ।
3. ये स म तयां संबं धत मं ालय के दन- त दन के शासन को यान म रखते हुए संसद के त
मं प रषद क जवाबदे ह सु नि चत करती ह।
ऊपर दए गए कथन म से कौन-सा/से सह है/ह?
(A) केवल 1 और 2
(B) केवल 2
(C) केवल 3
(D) केवल 2 और 3

उ र: B
या या:
लोकसभा क नयम स म त क सफा रश पर, 1993 म संसद म 17 वभागीय-संबं धत थायी
स म तय का गठन कया गया था। 2004 म, ऐसी सात और स म तयाँ था पत क ग , इस कार
उनक सं या 17 से बढ़कर 24 हो गई।
कथन 1 गलत है: 24 वभागीय थायी स म तयाँ अपने अ धकार े म क सरकार के सभी
मं ालय / वभाग को कवर करती ह। येक थायी स म त म 31 सद य होते ह (21 लोकसभा से
और 10 रा य सभा से)। लोकसभा के सद य को अ य वारा अपने सद य म से ना मत कया
जाता है , जैसे रा यसभा के सद य को सभाप त वारा अपने सद य म से ना मत कया जाता है।
कथन 2 सह है : एक मं ी कसी भी वभागीय थायी स म त के सद य के प म मनोनीत होने के
यो य नह ं है । य द कोई सद य कसी वभागीय थायी स म त म नामांकन के बाद मं ी बना दया
जाता है , तो वह स म त का सद य नह ं रह जाता है।
कथन 3 गलत है: इन वभागीय थायी स म तय का मु य उ दे य संसद के त कायपा लका
(अथात मं प रषद) क अ धक जवाबदे ह , वशेष प से व ीय जवाबदे ह को सरु त करना है।
हालां क, इन थायी स म तय के कामकाज पर इस अथ म सीमाएं लगाई गई ह क उ ह संबं धत
मं ालय / वभाग के दन- त दन के शासन के मामल पर वचार नह ं करना चा हए।
ोत: एम. ल मीकांत वारा राजनी त- छठा सं करण-अ याय- 23।

50. यायाधीश क नयिु त के संदभ म, न न ल खत कथन पर वचार क िजएः


1. भारत के सं वधान म कह ं भी 'कॉलेिजयम' श द का उ लेख नह ं है।
2. थम यायाधीश के मामले ने फैसला सन
ु ाया क परामश क या के दौरान सीजेआई वारा
द गई सलाह सव च यायालय के यायाधीश क नयिु त के मामले म रा प त पर बा यकार
है ।
3. तीसरे यायाधीश के मामले म यह माना गया क भारत के मु य यायाधीश क एकमा राय
सव च यायालय के यायाधीश क नयिु त के मामल म परामश या का गठन नह ं करती है ।
ऊपर दए गए कथन म से कौन-से सह ह?
(A) केवल 1 और 2
(B) केवल 1 और 3
(C) केवल 2 और 3
(D) 1, 2 और 3
उ र: B
या या:
कथन 1 सह है : जज का कॉलेिजयम सु ीम कोट का आ व कार है । यह सं वधान म नह ं है।
अनु छे द 124 म केवल यह ावधान है क सव च यायालय और उ च यायालय के यायाधीश
को रा पत वारा नयु त कया जाना है और परामश क या क बात करता है।
कथन 2 गलत है: थम यायाधीश के मामले (1982) म, सव च यायालय ने माना क परामश
का अथ सहम त नह ं है और इसका अथ केवल वचार का आदान- दान है। ले कन, दस
ू रे
यायाधीश के मामले (1993) म, सव च यायालय ने अपने पहले के फैसले को उलट दया और
परामश श द का अथ सहम त से बदल दया। इस लए, इसने फैसला सन
ु ाया क भारत के मु य
यायाधीश वारा द गई सलाह सव च यायालय के यायाधीश क नयिु त के मामल म
रा प त पर बा यकार है।
कथन 3 सह है : तीसरे यायाधीश के मामले (1998) म, सव च यायालय ने कहा क सव च
यायालय के यायाधीश क नयिु त के मामल म भारत के मु य यायाधीश वारा अपनाई जाने
वाल परामश या के लए ' यायाधीश क बहुलता के परामश क आव यकता है । '। भारत के
मु य यायाधीश क एकमा राय सव च यायालय के यायाधीश क नयिु त के मामल म
परामश या का गठन नह ं करती है । सु ीम कोट ने इस मामले म कॉलेिजयम का व तार पांच
सद यीय नकाय म कर दया, िजसम सीजेआई और उनके चार व र ठतम सहयोगी शा मल थे।
सु ीम कोट ने माना क भारत के मु य यायाधीश वारा परामश या के मानदं ड और
आव यकताओं का पालन कए बना क गई सफा रश सरकार पर बा यकार नह ं है। और अगर दो
यायाधीश तकूल राय दे ते ह, तो भी उ ह सरकार को सफा रश नह ं भेजनी चा हए।
ोत: एम. ल मीकांत वारा राजनी त- छठा सं करण-अ याय- 26।
51. न न ल खत पंचायती राज स म तय पर वचार क िजये:

1. बलवंत राय मेहता स म त

2. एलएम संघवी स म त

3. अशोक मेहता स म त

4. जीवीके राव स म त

न न ल खत म से कौन सा ऊपर द गई स म तय के गठन का सह कालानु मक म है ?

(a) 1 - 3 - 2 - 4

(b) 1 - 3– 4 - 2

(c) 3 - 1 - 2 - 4

(d) 3 - 1 - 4 - 2

उ र: b

या या:

वक प (b) सह है: जनवर 1957 म, भारत सरकार ने सामद


ु ा यक वकास काय म (1952) और
रा य व तार सेवा (1953) के कामकाज क जांच करने तथा उनके बेहतर संचालन के उपाय का
सझ
ु ाव दे ने के लए एक स म त का गठन कया। इस स म त के अ य बलवंत राय मेहता थे।
स म त ने नवंबर 1957 म अपना तवेदन तत
ु कया और 'लोकतां क वक करण' क योजना
क थापना संबध
ं ी सफा रश क , िजसे अंततः पंचायती राज के प म जाना जाने लगा।

दस बर 1977 म जनता सरकार ने अशोक मेहता क अ य ता म पंचायती राज सं थाओं के संबध



म एक स म त का गठन कया। इसने अग त 1978 म अपना तवेदन तुत कया और दे श म
गरती पंचायती राज यव था को पन
ु ज वत एवं मजबत
ू करने के लए 132 सफा रश तत
ु क ं।

ामीण वकास और गर बी उ मल
ू न काय म के लए मौजूदा शास नक यव था क समी ा
करने के लए जीवीके राव क अ य ता म एक स म त का गठन कया गया। राव को 1985 म
योजना आयोग वारा नयु त कया गया था। स म त इस न कष पर पहुंची क वकास या
धीरे -धीरे नौकरशाह के समान बन गई और पंचायती राज से अलग हो गई। इसने वकास शासन म
पंचायती राज सं थाओं को नौकरशाह करण वारा कमजोर होने के कारण, इसे मजबत
ू करने हे तु कुछ
मह वपण
ू सफा रश तुत क ं, िज ह उपयु त प से 'जड़ के बना घास' कहा गया।
1986 म, राजीव गांधी सरकार ने एलएम संघवी क अ य ता म 'लोकतं और वकास के लए
पंचायती राज सं थान के पन
ु रो धार' पर एक अवधारणा प तैयार करने के लए एक स म त का
गठन कया।

सहायक अनुमान: कालानु मक म म स म तय को याद रखने का उपाय - बै स (बलवंत - अशोक


- जी.वी.के.राव - संघवी)। जी को गाड गल स म त के साथ मत न कर।

ोत: एम ल मीकांत वारा भारतीय राजनी त छठा सं करण, अ याय 38

52. 73व सं वधान संशोधन अ ध नयम, 1992 क वशेषताओं के संबध


ं म न न ल खत कथन पर
वचार क िजये:

1. इसने भारत के सं वधान म एक नया भाग-XI जोड़ा।

2. इसने भारत के सं वधान म एक नई नौवीं अनस


ु च
ू ी जोड़ी।

3. इसने रा य के नी त नदे शक त व के अनु छे द 40 को यावहा रक प दया।

4. इसने पहल बार पंचायती राज सं थाओं को संवध


ै ा नक दजा दया।

ऊपर दए गए कथन म से कौन-सा/से सह है/ह?

(a) केवल 4

(b) केवल 1 और 3

(c) केवल 3 और 4

(d) केवल 2, 3 और 4

उ र: c

या या:

कथन 1 और 2 गलत ह: 73व सं वधान संशोधन अ ध नयम वारा भारत के सं वधान म एक नया
भाग-IX जोड़ा गया। यह भाग 'पंचायत ' के प म व णत है और इसम अनु छे द 243 से 243 O
तक ावधान शा मल ह। इसके अ त र त, अ ध नयम वारा सं वधान म एक नई यारहवीं अनस
ु च
ू ी
भी जोड़ी गयी। इस अनुसच
ू ी म पंचायत क 29 काया मक मद शा मल ह। यह अनु छे द 243-G से
संबं धत है ।

कथन 3 सह है : अ ध नयम ने सं वधान के अनु छे द 40 को एक यावहा रक प भी दया है जो


कहता है , िजसके अनस
ु ार, “रा य ाम पंचायत को संग ठत करने के लए कदम उठाएगा और उ ह
ऐसी शि तयाँ एवं अ धकार दान करे गा, जो उ ह व-शासन संबध
ं ी इकाइय के प म” काय करने
म स म बनाने के लए आव यक हो सकते ह। यह अनु छे द रा य के नी त नदे शक त व का एक
ह सा है ।

कथन 4 सह है : अ ध नयम वारा पहल बार पंचायती राज सं थाओं को संवध


ै ा नक दजा दया।
इसने उ ह सं वधान के यायो चत भाग के दायरे म शा मल कर दया। दस
ू रे श द म, रा य सरकार
अ ध नयम के ावधान के अनस
ु ार, नई पंचायती राज णाल को अपनाने के लए संवध
ै ा नक
दा य व के अधीन ह। प रणाम व प, न तो पंचायत का गठन और न ह नय मत अंतराल पर
चन
ु ाव कराना रा य सरकार क इ छा पर नभर करता है।

ोत: एम ल मीकांत वारा भारतीय राजनी त छठा सं करण, अ याय 38

53. नीचे दए गए कथन म से कौन सा सह नह ं है?


(A) सदन को अ नि चत काल के लए थ गत कर दए जाने के बाद ह सदन का स ावसान कया
जा सकता है ।
(B) रा प त संघ क कायका रणी के साथ-साथ संसद का भी एक ह सा है।
(C) सं वधान अब एं लो-इं डयन समद
ु ाय के सद य के लोकसभा म नामांकन को अ नवाय नह ं
करता है ।
(D) जब कसी रा प त पर महा भयोग चलाया जाता है , तो उपरा प त नए रा प त के शपथ लेने
तक रा प त के प म काय करता है।

उ र: A
या या:
वक प (A) गलत है: पीठासीन अ धकार वारा सदन को अ नि चत काल के लए थ गत करने के
कुछ दन बाद, रा पत स के स ावसान के लए एक अ धसच
ू ना जार करता है । हालाँ क,
रा प त सदन के स के दौरान भी सदन का स ावसान कर सकता है।
वक प (B) सह है: भारत का रा प त कसी भी सदन का सद य नह ं है और संसद क बैठक म
भाग लेने के लए नह ं बैठता है ले कन वह संसद का अ भ न अंग है । वह संघ कायका रणी का भी
ह सा है और दे श म सव च औपचा रक ा धकरण है।
वक प (C) सह है: हाल ह म पा रत, 104 व सं वधान संशोधन ने अनस
ु ू चत जा तय और
अनस
ु ू चत जनजा तय के सद य के लए लोकसभा और रा य वधानसभाओं म सीट के आर ण
को समा त करने क समय सीमा 10 साल बढ़ा द है । हालाँ क, संशोधन दो लोकसभा सीट और
रा य वधान सभाओं म सीट के आर ण क अव ध को एं लो-इं डयन समद
ु ाय के सद य के लए
आर त नह ं करता है । इस कार, भारत के रा प त (रा य म रा यपाल) वारा भारत के धान
मं ी (रा य म मु यमं ी) क सफा रश पर एं लो-इं डयन समद
ु ाय (रा य म एक) के दो सद य को
ना मत करने क था को भावी ढं ग से समा त कर दया गया था।
वक प (D) सह है: जब रा प त पर महा भयोग चलाया जाता है , तो 6 मह ने के भीतर रि त को
भरना होगा। उपरा प त रा प त के प म तब तक काय करता है जब तक क नया रा प त पद
के लए शपथ नह ं लेता।

ोत: एम ल मीकांत वारा भारतीय राजनी त छठा सं करण, अ याय-17

54. भारतीय सं वधान के न न ल खत ावधान पर वचार कर:


1. संवध
ै ा नक संशोधन क मांग करने वाला वधेयक एक नजी सद य वारा पेश नह ं कया जा
सकता है ।
2. धन वधेयक पेश करने से पहले रा प त क पव
ू ानम
ु त आव यक है।
3. साधारण वधेयक और धन वधेयक के मामले म संयु त बैठक का ावधान मौजूद है ले कन
संवध
ै ा नक संशोधन वधेयक के मामले म नह ं।
ऊपर दए गए कथन म से कौन-सा/से सह नह ं है/ह?
(A) केवल 1
(B) केवल 2
(C) केवल 1 और 3
(D) उपरो त म से कोई नह ं

उ र: C
या या:
कथन 1 गलत है : एक संवध
ै ा नक संशोधन वधेयक एक मं ी के साथ-साथ एक नजी सद य (सदन
का सद य जो मं ी नह ं है) दोन वारा पेश कया जा सकता है और इसके लए रा प त क पव

अनम
ु त क भी आव यकता नह ं होती है।
कथन 2 सह है : सं वधान संसद म धन वधेयक को पा रत करने के लए एक वशेष या
नधा रत करता है । धन वधेयक केवल लोकसभा म पेश कया जा सकता है और वह भी रा पत
क सफा रश पर। ऐसे येक वधेयक को एक सरकार वधेयक माना जाता है और इसे केवल एक
मं ी ह पेश कर सकता है ।
कथन 3 गलत है: भारतीय सं वधान का अनु छे द 108 दोन सदन के बीच ग तरोध क ि थ त म
संयु त बैठक का ावधान करता है । रा प त वधेयक पर वचार- वमश करने और मतदान करने के
उ दे य से दोन सदन को संयु त बैठक म बल
ु ा सकता है । यहां यह यान दया जाना चा हए क
संयु त बैठक का ावधान केवल साधारण वधेयक या व ीय वधेयक पर लागू होता है न क धन
वधेयक या सं वधान संशोधन वधेयक पर। धन वधेयक के मामले म, लोकसभा के पास अ धभावी
शि तयां होती ह, जब क सं वधान संशोधन वधेयक को येक सदन वारा अलग से पा रत कया
जाना चा हए।

ोत: एम ल मीकांत वारा भारतीय राजनी त छठा सं करण, अ याय-22

55. रा यपाल क तुलना म रा पत क मादान शि तय के संबध


ं म न न ल खत कथन पर
वचार कर:
1. य द दं ड या सजा कोट माशल वारा है तो रा प त अपनी मादान शि तय का उपयोग कर
सकते ह जब क रा यपाल ऐसा नह ं कर सकते ह।
2. अनु छे द 72 के तहत रा पत क मादान शि त का दायरा अनु छे द 161 के तहत रा यपाल
क मादान शि त से अ धक यापक है।
3. रा पत क मादान शि त या यक समी ा के दायरे से बाहर है और इसे कसी भी यायालय
म चन
ु ौती नह ं द जा सकती है ।
ऊपर दए गए कथन म से कौन-सा/से सह है/ह?
(A) केवल 1 और 2
(B) केवल 2 और 3
(C) केवल 1 और 3
(D) 1, 2 और 3

उ र: A
या या:
भारत का सं वधान मशः अनु छे द 72 और 161 के मा यम से रा य के रा प त और रा यपाल
को मादान क शि त दान करता है । ये शि तयां कसी भी संभा वत या यक ु टय के सध
ु ार क
अनम
ु त दे ती ह और उन लोग को राहत दान करती ह िज ह वे अनु चत प से कठोर दं ड के प
म मानते ह।
कथन 1 सह है : सं वधान के अनु छे द 72(1) के तहत, रा प त को कसी भी अपराध के लए दोषी
ठहराए गए कसी भी यि त क सजा को माफ करने, राहत दे ने, राहत दे ने या छूट दे न,े या
नलं बत करने, कम करने या सजा को कम करने का अ धकार है ।
जहां:
1. सजा या सजा कोट माशल वारा है ;
2. सजा या सजा उस मामले से संबं धत कसी भी कानन
ू के खलाफ अपराध के लए है, िजस पर
संघ क कायकार शि त का व तार होता है ;
3. सजा मौत क सजा है ।
कथन 2 सह है : इसी तरह, अनु छे द 161 के तहत, रा य के रा यपाल को दं ड से संबं धत कसी
भी कानन
ू के खलाफ कसी भी अपराध के लए दोषी ठहराए गए कसी भी यि त क सजा को
माफ करने, राहत दे ने, राहत दे ने या छूट दे ने का अ धकार है। वह वषय िजस पर रा य क
कायपा लका शि त का व तार होता है।
इस कार, अनु छे द 72 के तहत रा पत क मादान शि त का दायरा अनु छे द 161 के तहत
रा यपाल क मा शि त से अ धक यापक है ।
कथन 3 गलत है: रा प त और रा यपाल क मादान शि तयां उनके संबं धत मं प रषद क
सलाह से बा य होती ह। सु ीम कोट ने माना है क दभ
ु ावना, अ ासं गक, तकह न या भेदभावपण

मा के मामले म मादान शि त या यक समी ा के अधीन है ।

ोत: एम ल मीकांत वारा भारतीय राजनी त छठा सं करण, अ याय-17

56. दलबदल वरोधी कानून के संबध


ं म न न ल खत कथन पर वचार कर:
1. 52व सं वधान संशोधन अ ध नयम म दल-बदल के आधार पर संसद सद य क अयो यता का
ावधान कया गया।
2. य द वधायक दल के एक तहाई सद य दल से अलग हो जाते ह, तो यह दलबदल वरोधी नह ं
है ।
3. अयो यता के संबध
ं म नणय या यक समी ा के अधीन नह ं हो सकता।
ऊपर दए गए कथन म से कौन-सा/से सह है/ह?
(A) केवल 1 और 2
(B) केवल 2
(C) केवल 1
(D) केवल 2 और 3

उ र: C
या या:
कथन 1 सह है : दलबदल वरोधी कानून - दसवीं अनस
ु च
ू ी को 1985 म 52व संशोधन अ ध नयम
वारा सं वधान म शा मल कया गया था। यह उस या को नधा रत करता है िजसके वारा
वधा यका के पीठासीन अ धकार वारा सदन के कसी अ य सद य वारा या चका के आधार पर
वधायक को दलबदल के आधार पर अयो य ठहराया जा सकता है।
कथन 2 गलत है : अयो यता ावधान –
 य द कसी राजनी तक दल से संबं धत सदन का सद य:
o वे छा से अपने राजनी तक दल क सद यता छोड़ दे ता है , या
o अपने राजनी तक दल के नदश के वपर त, वधा यका म वोट दे ता है या नह ं दे ता
है । हालां क, य द सद य ने पव
ू अनम
ु त ल है , या ऐसे मतदान या अनप
ु ि थत रहने
से 15 दन के भीतर पाट वारा माफ कर दया जाता है , तो सद य को अयो य
घो षत नह ं कया जाएगा।
 य द कोई नदल य उ मीदवार चुनाव के बाद कसी राजनी तक दल म शा मल हो जाता है ।
 य द कोई मनोनीत सद य वधा यका का सद य बनने के छह मह ने बाद कसी पाट म
शा मल होता है ।
कानन
ू के तहत अपवाद:
 वधायक कुछ प रि थ तय म अयो यता के जो खम के बना अपनी पाट बदल सकते ह।
कानन
ू कसी पाट को कसी अ य पाट के साथ या कसी अ य पाट म वलय करने क
अनम
ु त दे ता है , बशत क उसके कम से कम दो- तहाई वधायक वलय के प म ह।
 ऐसे म न तो वलय का फैसला करने वाले सद य और न ह मल
ू पाट के साथ रहने वाले
सद य को अयो यता का सामना करना पड़ेगा।
 वधायक दल के एक तहाई सद य वारा वभाजन के मामले म अयो यता से छूट से
संबं धत दसवीं अनस
ु च
ू ी के ावधान को 91व संशोधन अ ध नयम 2003 वारा हटा दया
गया है । बंटवारा
कथन 3 गलत है : कानन
ू ने शु म कहा था क पीठासीन अ धकार का नणय या यक समी ा के
अधीन नह ं है। 1992 म सव च यायालय वारा इस शत को र द कर दया गया, िजससे उ च
यायालय और सव च यायालय म पीठासीन अ धकार के नणय के खलाफ अपील क अनम
ु त
मल गई। हालां क, सु ीम कोट ने माना है क जब तक पीठासीन अ धकार अपना आदे श नह ं दे ता
तब तक कोई या यक ह त ेप नह ं हो सकता है।

ोत: एम ल मीकांत वारा भारतीय राजनी त छठा सं करण, अ याय-76

57. भारत म संसद य कायवाह के उपकरण के संदभ म न न ल खत कथन पर वचार कर:


1. येक संसद य बैठक का पहला घंटा शू यकाल के लए आर त होता है।
2. नकाल के वपर त, या के नयम म शू यकाल का उ लेख नह ं है।
ऊपर दए गए कथन म से कौन-सा/से सह है/ह?
(A) केवल 1
(B) केवल 2
(C) दोन 1 और 2
(D) न तो 1 और न ह 2

उ र: B
या या:
कथन 1 गलत है: नकाल - इसके लए येक संसद य बैठक का पहला घंटा नधा रत कया जाता
है । इस दौरान सद य सवाल पछ
ू ते ह और मं ी आमतौर पर जवाब दे ते ह। न तीन कार के होते
ह, तारां कत, अतारां कत और सं त सच
ू ना।
1. एक तारां कत न (तारांकन वारा व श ट) के लए एक मौ खक उ र क आव यकता होती है
और इस लए परू क न हो सकते ह।
2. दस
ू र ओर, एक अतारां कत न के लए ल खत उ र क आव यकता होती है और इस लए,
परू क न नह ं हो सकते।
3. शॉट नो टस न वह होता है िजसे दस दन से कम समय का नो टस दे कर पछ
ू ा जाता है ।
इसका उ र मौ खक प से दया जाता है ।
मं य के अलावा नजी सद य से भी सवाल पछ
ू े जा सकते ह। इस कार, एक न एक गैर-
सरकार सद य को संबो धत कया जा सकता है य द न क वषय व तु कसी वधेयक, संक प
या सदन के काय से संबं धत अ य मामले से संबं धत है िजसके लए वह सद य िज मेदार है । इस
तरह के न के संबध
ं म या वह है जो कसी मं ी को संबो धत न के मामले म अपनाई
जाती है ।
कथन 2 सह है : नकाल के वपर त, या के नयम म शू यकाल का उ लेख नह ं है। शू यकाल
संसद सद य के लए बना कसी पव
ू सच
ू ना के मामल को उठाने के लए उपल ध एक
अनौपचा रक उपकरण है। शू यकाल नकाल के तरु ं त बाद शु होता है और तब तक रहता है जब
तक क दन क कायसच
ू ी (अथात ् सदन का नय मत काय) शु नह ं हो जाती। दस
ू रे श द म,
नकाल और कायसच
ू ी के बीच के समय के अंतराल को शू य काल के प म जाना जाता है। यह
संसद य याओं के े म एक भारतीय नवाचार है और 1962 से अि त व म है।

ोत: एम ल मीकांत वारा भारतीय राजनी त छठा सं करण, अ याय-22

58. न न ल खत यु म पर वचार क िजएः


ताव उपयोग

1. समापन रा पत वारा उ घाटन भाषण


2. थगन कसी भी मामले पर बहस को कम करने के लए
3. अ व वास मं ालय को पद से हटाने के लए लोकसभा म का इ तेमाल
कया जा सकता है
4. ध यवाद ताव रा पत वारा सावज नक मह व के कसी मामले क ओर
यान आक षत करने के लए
ऊपर दए गए यु म म से कतने सह सम
ु े लत ह/ह?
(A) केवल एक जोड़ी
(B) केवल दो जोड़े
(C) केवल तीन जोड़े
(D) सभी चार जोड़े

उ र:A
या या:
पीठासीन अ धकार क सहम त से कए गए ताव के अलावा आम सावज नक मह व के मामले पर
कोई चचा नह ं हो सकती है। सदन व भ न मु द पर या तो मं य या गैर-सरकार सद य वारा
पेश कए गए ताव को वीकार या अ वीकार करके अपने नणय या राय य त करता है।
जोड़ी 1 का गलत मलान कया गया है: समापन ताव - यह सदन के सम कसी मामले पर
बहस को कम करने के लए एक सद य वारा पेश कया गया ताव है। य द ताव को सदन
वारा अनम
ु ो दत कया जाता है , तो बहस को तरु ं त रोक दया जाता है और मामले पर मतदान कया
जाता है ।
जोड़ी 2 का गलत मलान कया गया है: थगन ताव - इसे संसद म त काल सावज नक मह व
के एक नि चत मामले पर सदन का यान आक षत करने के लए पेश कया जाता है , और इसे 50
सद य के समथन क आव यकता होती है । चूं क यह सदन के सामा य काय को बा धत करता है ,
इस लए इसे एक असाधारण यिु त माना जाता है। इसम सरकार के खलाफ नंदा का एक त व
शा मल है और इस लए रा यसभा को इस उपकरण का उपयोग करने क अनम
ु त नह ं है। थगन
ताव पर चचा कम से कम दो घंटे तीस मनट तक चलनी चा हए।
जोड़ी 3 सह सम
ु े लत है : अ व वास ताव - सं वधान का अनु छे द 75 कहता है क मं प रषद
सामू हक प से लोकसभा के त उ रदायी होगी। इसका अथ है क मं ालय तब तक अपने पद पर
बना रहता है जब तक उसे लोकसभा के अ धकांश सद य का व वास ा त होता है । दस
ू रे श द म,
लोकसभा अ व वास ताव पा रत करके मं ालय को पद से हटा सकती है। ताव को वीकार
करने के लए 50 सद य के समथन क आव यकता है।
जोड़ी 4 का गलत मलान कया गया है: ध यवाद ताव - येक आम चुनाव के बाद पहला स
और येक व ीय वष का पहला स रा पत वारा संबो धत कया जाता है । इस संबोधन म
रा प त पव
ू वत वष और आगामी वष म सरकार क नी तय और काय म क परे खा तैयार करते
ह। रा प त के इस अ भभाषण पर, जो ' टे न म संहासन से भाषण' से मेल खाता है , संसद के
दोन सदन म 'ध यवाद ताव' नामक ताव पर चचा क जाती है । चचा के अंत म, ताव को
मतदान के लए रखा जाता है । यह ताव सदन म पा रत होना चा हए। नह ं तो यह सरकार क
हार है । रा प त का यह उ घाटन भाषण संसद सद य के लए सरकार और शासन क खा मय
और वफलताओं क जांच और आलोचना करने के लए चचा और बहस करने का अवसर है ।

ोत: एम ल मीकांत वारा भारतीय राजनी त छठा सं करण, अ याय-22

59. लोकसभा के चन
ु ाव के लए नामांकन प कसके वारा दा खल कया जा सकता है। (2017)
(A) भारत म रहने वाला कोई भी यि त।
(B) उस नवाचन े का नवासी जहां से चुनाव लड़ा जाना है ।
(C) भारत का कोई भी नाग रक िजसका नाम कसी नवाचन े क मतदाता सच
ू ी म आता है ।
(D) भारत का कोई भी नाग रक।

उ र: C
या या:
वक प (C) सह है: नामांकन प 25 वष या उससे अ धक उ के कसी भी यि त वारा दा खल
कया जा सकता है और लोकसभा सीट के लए चुनाव लड़ने के यो य है ।
उ मीदवार को एक नवाचन े का पंजीकृत मतदाता और भारत का नाग रक होना चा हए। हालाँ क,
य द वह कसी वशेष रा य म पंजीकृत मतदाता है , तो वह कसी भी रा य क कसी भी सीट से
चुनाव लड़ सकता है । उ मीदवार को ना मत नह ं कया जा सकता है य द उ ह कसी अपराध के
लए दोषी ठहराया गया है और वे जेल म ह, या य द वह जमानत पर बाहर ह, तो उनक अपील का
नपटारा लं बत है । चुनाव आयोग के दशा- नदश के अनस
ु ार, ऐसे यि त को चन
ु ाव लड़ने से अयो य
घो षत कया जाता है। एक यि त, िजसने भारत सरकार के अधीन या कसी रा य क सरकार के
अधीन एक पद धारण कया है , उसे बखा त कर दया गया है , चन
ु ाव लड़ना चाहता है , उसे अपने
नामांकन प के साथ, धारा के अनस
ु ार चुनाव आयोग वारा जार माण प तुत करना आव यक
है । आरपी अ ध नयम, 1951 क 9(2)।

ोत: यप
ू ीएससी सीएसई पीवाई यू 2017

60. सीमा बंधन वभाग न न ल खत क य मं ालय म से कस एक का वभाग है ?


(A) र ा मं ालय
(B) गहृ मं ालय
(C) श पंग, सड़क प रवहन और राजमाग मं ालय
(D) पयावरण और वन मं ालय
उ र: B
या या:
वक प (B) सह है: सीमा बंधन वभाग गह
ृ मं ालय के अधीन है । गहृ मं ालय (एमएचए) व वध
िज मेदा रय का नवहन करता है , उनम से मह वपण
ू ह- आंत रक सरु ा, सीमा बंधन, क -रा य
संबध
ं ,क शा सत दे श का शासन, क य सश पु लस बल का बंधन, आपदा बंधन, आ द।

ोत: यप
ू ीएससी सीएसई 2008

61. सं वधान म नधा रत ावधान के संदभ म, न न ल खत म से कौन सा कथन सह नह ं है ?


(A) कोई वधेयक धन वधेयक है या नह ं, इस संबध
ं म लोकसभा अ य का नणय अं तम होता
है ।
(B) अ य का पद बहुमत दल के सद य के पास जाता है जब क उपा य का पद वप के लए
होता है ।
(C) लोक सभा का स ावसान करने क शि त रा प त के पास होती है जब क इसे थ गत करने
क शि त लोकसभा अ य के पास होती है ।
(D) जब भी लोकसभा भंग हो जाती है , अ य अपना पद खाल नह ं करता है और नव- नवा चत
लोकसभा क बैठक तक जार रहता है।

उ र: B
या या:
वक प (A) सह है: अनु छे द 110 (3) के अनस
ु ार, य द कोई वधेयक धन वधेयक है या नह ं, इस
पर कोई न उठता है तो लोकसभा अ य का नणय अं तम होता है । इस संबध
ं म उनके नणय
पर कसी भी यायालय म या संसद के कसी भी सदन म या यहां तक क रा पत वारा भी
सवाल नह ं उठाया जा सकता है । अ य कसी वधेयक को धन वधेयक के प म समथन दे ता है
जब ऐसा वधेयक रा य सभा को े षत कया जाता है और जब इसे रा प त क सहम त के लए
तुत कया जाता है ।
वक प (B) गलत है : ऐसा कोई संवध
ै ा नक ावधान नह ं है जो यह दान करता हो क अ य का
पद बहुमत दल को और उपा य का पद वप के पास जाता है। अ य और उपा य का चन
ु ाव
संसद के सद य वारा कया जाता है । वे कसी भी राजनी तक दल के हो सकते ह। 11वीं लोकसभा
के बाद से यह चलन था क अ य स ा प से आता है जब क उपा य मु य वप ी दल से
आता है ।
वक प (C) सह है: संसद क बैठक थगन या अ नि चत काल के लए थगन या स ावसान या
वघटन (लोकसभा के मामले म) वारा समा त क जा सकती है । एक थगन एक नि चत समय के
लए बैठक म काम को नलं बत कर दे ता है , जो घंटे, दन या स ताह हो सकता है । अ नि चत काल
के लए थगन का अथ है अ नि चत काल के लए संसद क बैठक को समा त करना। यह रा पत
है िजसके पास लोक सभा को भंग करने के साथ-साथ उसका स ावसान करने क शि त है । पीकर
केवल नचले सदन के स को थ गत कर सकता है।
वक प (D) सह है: अ य अपने चन
ु ाव क तार ख से अगल लोकसभा क पहल बैठक (5 साल
के लए) के ठ क पहले तक पद धारण करता है । एक बार नवा चत अ य पन
ु : नवाचन के लए
पा होता है । जब भी लोकसभा भंग होती है , अ य अपना पद खाल नह ं करता है और नव-
नवा चत लोकसभा क बैठक तक जार रहता है ।

ोत: एम ल मीकांत वारा भारतीय राजनी त छठा सं करण, अ याय-22

62. न न ल खत कथन पर वचार कर:


1. लोकसभा के लए गणपू त अ य स हत कुल सद य क सं या का दसवां ह सा है।
2. लोक लेखा स म त का पदे न अ य लोक सभा का अ य होता है ।
3. अ य को केवल लोकसभा वारा भावी बहुमत (यानी सदन के सभी त काल न सद य के
बहुमत) वारा पा रत ताव वारा हटाया जा सकता है।
ऊपर दए गए कथन म से कौन-सा/से सह है/ह?
(A) केवल 1
(B) केवल 1 और 3
(C) केवल 2 और 3
(D) 1, 2 और 3

उ र: B
या या:
कथन 1 सह है : कोरम कसी भी काय को करने से पहले सदन म उपि थत होने के लए आव यक
यूनतम सद य क सं या है । यह अनु छे द 100(3) के अनस
ु ार पीठासीन अ धकार स हत येक
सदन म सद य क कुल सं या का दसवां ह सा है। इसका अथ है क य द कोई काय करना है तो
लोकसभा म कम से कम 55 सद य और रा य सभा म 25 सद य उपि थत होने चा हए। य द
सदन क बैठक के दौरान गणपू त नह ं होती है , तो पीठासीन अ धकार का यह कत य है क वह या
तो सदन को थ गत करे या बैठक को गणपू त होने तक थ गत करे ।
कथन 2 गलत है : लोक लेखा स म त सबसे परु ानी संसद य स म त है और इसे पहल बार 1921 म
ग ठत कया गया था। इसका गठन भारत सरकार अ ध नयम 1919 के तहत कया गया था। स म त
म 22 सद य होते ह, 15 सद य लोकसभा वारा चुने जाते ह और 7 सद य होते ह। रा यसभा।
अ य को अपने सद य म से स म त के अ य को नयु त करने का अ धकार है। परं परा के
अनस
ु ार इस स म त के अ य क नयिु त वप ी दल वारा क जाती है ।
कथन 3 सह है : अ य को कायकाल क सरु ा दान क जाती है । उसे केवल अनु छे द 94 (C) के
अनस
ु ार लोकसभा वारा भावी बहुमत (अथात सदन के सभी त काल न सद य के बहुमत) वारा
पा रत एक ताव वारा हटाया जा सकता है , न क सामा य बहुमत (यानी, बहुमत से) सदन म
उपि थत और मतदान करने वाले सद य)। हटाने के इस ताव पर तभी वचार और चचा हो सकती
है जब इसे कम से कम 50 सद य का समथन ा त हो।

ोत:
भारत का सं वधान [अनु छे द 100(3), 94(सी)]
एम ल मीकांत वारा भारतीय राजनी त का छठा सं करण, अ याय-23
63. लोक सभा के उपा य के संदभ म, न न ल खत कथन पर वचार क िजएः
1. संवध
ै ा नक ावधान के अनस
ु ार, अ य का पद बहुमत दल को और उपा य का पद वप के
पास जाता है ।
2. जब वह अ य क अनप
ु ि थ त म सदन का पीठासीन अ धकार होता है , तो उसके पास वह
शि तयाँ होती ह जो अ य क होती ह।
3. य द वह कसी संसद य स म त का सद य है तो वह वतः ह उस स म त का अ य बन जाता
है ।
4. जब क अ य पद के लए शपथ लेता है , उपा य पद के लए कोई शपथ नह ं लेता है ।
ऊपर दए गए कथन म से कतने/ कतने सह ह/ह?
(A) केवल एक
(B) केवल दो
(C) केवल तीन
(D) सभी चार

उ र: B
या या:
पीकर वारा तय कए गए दन पर पीकर के बाद सदन वारा ड ट पीकर का चुनाव कया
जाता है । उनके पद क शत और हटाने का आधार और हटाने क या, वेतन और भ का
ावधान अ य के समान ह है । उ ह ने अ य को प लखकर इ तीफा दे सकते है ।
कथन 1 गलत है: भारत के सं वधान के अनस
ु ार, लोकसभा म अ य और उपा य इसके सद य
के बीच चन
ु े जाते ह। 10वीं लोकसभा तक, अ य और उपा य दोन आमतौर पर स ाधार दल से
होते थे। 11वीं लोकसभा के बाद से यह आम सहम त रह है क अ य स ाधार दल (या स ा ढ़
गठबंधन) से आता है और उपा य का पद मु य वप ी दल को जाता है। हालां क, ऐसा कोई
संवध
ै ा नक आदे श नह ं है।
कथन 2 सह है : जब वह अ य क अनुपि थ त म सदन का पीठासीन अ धकार होता है , तो उसके
पास वह शि तयाँ होती ह जो अ य क होती ह। वह अ य के अधीन थ नह ं होता बि क सदन
के त सीधे तौर पर िज मेदार होता है ।
कथन 3 सह है : जब भी अ य उपि थत होता है , उपा य सदन का सामा य सद य बन जाता
है । हालाँ क, य द वह कसी संसद य स म त का सद य है तो वह वतः ह उस स म त का अ य
बन जाता है ।
कथन 4 गलत है: अ य और उपा य दोन पद क शपथ नह ं लेते ह।
ोत: एम ल मीकांत वारा भारतीय राजनी त छठा सं करण, अ याय-22

64. रा यसभा के सभाप त के प म उपरा प त क भू मका के संबध


ं म न न ल खत कथन पर
वचार कर:
1. सभाप त संसद का सद य नह ं होता है ।
2. अ य पहल बार म अ य क तरह मतदान नह ं कर सकते, ले कन अ य के वपर त वोट
क समानता के मामले म उनके पास नणायक वोट नह ं होता है
3. अ य अपने वेतन और भ को उपरा प त के प म लेता है।
ऊपर दए गए कथन म से कौन-सा/से सह है/ह?
(A) केवल 1
(B) केवल 2 और 3
(C) केवल 1 और 3
(D) 1, 2 और 3

उ र: A
या या:
कथन 1 सह है और 2 गलत है : अ य (जो सदन का सद य है ) के वपर त, सभाप त सदन का
सद य नह ं होता है । ले कन अ य क तरह, सभाप त भी पहल बार म मतदान नह ं कर सकते।
वोट क समानता के मामले म वह भी वोट डाल सकता है । उपरा प त रा यसभा के सभाप त के
प म कसी बैठक क अ य ता नह ं कर सकता है जब उसे हटाने का ताव वचाराधीन हो।
हालाँ क, वह सदन म उपि थत हो सकता है और बोल सकता है और बना मतदान के, उसक
कायवाह म भाग ले सकता है , यहाँ तक क ऐसे समय म भी (जब क अ य पहल बार मतदान
कर सकता है जब उसे हटाने का ताव लोकसभा के वचाराधीन हो) )
कथन 3 गलत है: सं वधान ने उपरा प त के लए उस मता म कोई प रलि धयां तय नह ं क ह।
वह रा यसभा के पदे न सभाप त के प म अपना नय मत वेतन ा त करते ह। कसी भी अव ध के
दौरान जब उपरा प त रा प त के प म काय करता है या रा प त के काय का नवहन करता
है , वह रा य सभा के सभाप त को दे य कसी वेतन या भ े का हकदार नह ं होता है । ले कन उ ह
ऐसे समय म रा प त के वेतन और भ े का भग
ु तान कया जाता है।

ोत: एम ल मीकांत वारा भारतीय राजनी त छठा सं करण, अ याय -18, 22


65. पंचायती राज सं थाओं के संबध
ं म न न ल खत कथन पर वचार क िजये:

1. ाम सभा, पंचायती राज यव था क नींव के प म काय करती है ।

2. येक रा य म पंचायती राज क तर य यव था- ाम, िजला और नवाचन े के तर पर


पंचायत है ।

ऊपर दए गए कथन म से कौन सा सह नह ं है ?

(a) केवल 1

(b) केवल 2

(c) दोन 1 और 2

(d) न तो 1 और न ह 2

उ र: b

या या:

कथन 1 सह है : 73वां सं वधान संशोधन अ ध नयम पंचायती राज यव था क नींव के प म एक


ाम सभा का ावधान करता है । यह ाम तर पर पंचायत के े के अंतगत एक गांव क
मतदाता सच
ू ी म पंजीकृत यि तय से न मत सं था होती है। इस कार, एक पंचायत के े म
सभी पंजीकृत मतदाताओं से मलकर एक ाम सभा का नमाण होता है। यह ऐसी शि तय का
योग कर सकती है और ाम तर पर ऐसे काय कर सकता है , जैसा क रा य क वधा यका
वारा नधा रत कया गया हो।

कथन 2 गलत है : अ ध नयम, येक रा य म पंचायती राज क - तर य णाल का भी ावधान


करता है , अथात ाम, म यवत और िजला तर पर पंचायत। इस कार, अ ध नयम परू े दे श म
पंचायती राज क संरचना म एक पता लाता है । हालाँ क, 20 लाख से अ धक क आबाद वाला रा य
म यवत तर पर पंचायत का गठन नह ं कर सकता है।

ोत: एम ल मीकांत वारा भारतीय राजनी त छठा सं करण, अ याय 38


66. न न ल खत कथन पर वचार क िजये:

1. ाम, म यवत और िजला तर पर पंचायत क सभी सीट के लए य चुनाव।

2. पंचायत के सभी तर पर पांच वष का कायकाल नि चत करना।

3. कसी भी तर पर पछड़े वग के सद य के लए सीट का आर ण दान करना।

4. रा य चुनाव आयोग क थापना

5. पंचायत को शि तय और उ रदा य व का ह तांतरण।

उपरो त म से कतने 73व सं वधान संशोधन अ ध नयम के अंतगत अ नवाय ावधान नह ं ह/ह?

(a) केवल एक कथन

(b) केवल दो कथन

(c) केवल तीन कथन

(d) केवल चार कथन

उ र: c

या या:

वक प (c) सह है : भारत म पंचायती राज सं थाओं को संवध


ै ा नक दजा दान करने के लए 73वां
सं वधान संशोधन अ ध नयम 1992 लागू हुआ। इसम रा य वधा यका वारा तैयार कए जाने वाले
अ नवाय और साथ ह वैि छक ावधान कये गए थे। अ ध नयम के अ नवाय ावधान ह:

 ाम सभा का संगठन
 िजला, खंड ( लॉक) और ाम तर पर तर य पंचायती राज संरचना का गठन
 ाम, म यवत और िजला तर पर पंचायत क सभी सीट के लए य चुनाव
 पंचायत के सभी थान, पंचायत े के ादे शक नवाचन े से य चन
ु ाव वारा चन
ु े
गए यि तय वारा भरे जाएंगे।
 पंचायत का चुनाव लड़ने के लए यूनतम आयु 21 वष है
 पंचायत म म हलाओं के लए 33 तशत तक आर ण
 पंचायत म अनस
ु ू चत जा त/अनस
ु ू चत जनजा त के लए उनक जनसं या के अनप
ु ात म
सीट का आर ण
 चुनाव कराने के लए रा य चुनाव आयोग का गठन
 सभी तर पर पंचायत का नि चत 5 वष का कायकाल
 येक रा य को पंचायत क व ीय ि थ त क समी ा के लए येक पांच वष म एक
रा य व आयोग का गठन करना होता है
पछड़े वग के सद य के लए सीट के आर ण के साथ-साथ पंचायत को शि तय और
उ रदा य व का ह तांतरण 73व संशोधन अ ध नयम के वैि छक ावधान के अंतगत आता है।

ोत: एम ल मीकांत वारा भारतीय राजनी त छठा सं करण, अ याय 38

67. न न ल खत यु म पर वचार क िजये :

चुनावी उपकरण शु आती वष

1. इले ॉ नक वो टंग मशीन 1998-99

2. नोटा (उपरो त म से कोई नह ं) 2013-14

3. मतदाता न र ण पेपर ऑ डट ायल 2013-14

4. आदश आचार सं हता 1991

उपरो त म से कतने यु म सह सम
ु े लत ह/ह?

(a) केवल एक यु म

(b) केवल दो यु म

(c) केवल तीन यु म

(d) सभी चार यु म

उ र: d

या या:
यु म 1 सह सम
ु े लत है : इले ॉ नक वो टंग मशीन (ईवीएम) का योग पहल बार 1998 म
राज थान, म य दे श और द ल क वधानसभाओं के चुनाव म चय नत नवाचन े म
योगा मक आधार पर कया गया था। 1999 म गोवा वधानसभा के आम चन
ु ाव (परू े रा य) म
पहल बार इले ॉ नक वो टंग मशीन का उपयोग कया गया था।

यु म 2 सह सम
ु े लत है: 2013 म छ ीसगढ़, म य दे श, मजोरम, रा य राजधानी े द ल
और राज थान क वधानसभाओं के आम चुनाव के बाद से नोटा का ावधान कया गया और 2014
म, सोलहवीं लोकसभा (2014) के आम चन
ु ाव के साथ, आं दे श, अ णाचल दे श, ओ डशा और
सि कम म क रा य वधानसभाओं के आम चुनाव म इसका योग जार रहा।।

यु म 3 सह सम
ु े लत है : मतदाता न र ण पेपर ऑ डट ायल (वीवीपीएट ) का योग पहल बार
2013 म नागालड के नोकसेन वधानसभा े के उप-चुनाव म कया गया था। इसके बाद, रा य के
वधान सभाओं के येक आम चुनाव के दौरान चय नत नवाचन े म मतदाता न र ण पेपर
ऑ डट ायल का उपयोग कया गया। 2014 के लोकसभा चन
ु ाव म दे श के आठ चु नंदा संसद य
े म मतदाता न र ण पेपर ऑ डट ायल का इ तेमाल कया गया था। इले ॉ नक वो टंग
मशीन, के साथ मतदाता न र ण पेपर ऑ डट ायल का योग मतदान णाल क सट कता और
पारद शता सु नि चत करती है।

यु म 4 सह सम
ु े लत है : 1968 म सभी राजनी तक दल वारा आदश आचार सं हता पर सहम त
य त क गई थी। चन
ु ाव आयोग ने न प चन
ु ाव और एक समान तर पर त पधा सु नि चत
करने के लए पहल बार वष 1991 म आदश आचार सं हता का भावी ढं ग से उपयोग कया।

ोत: एम ल मीकांत वारा भारतीय राजनी त छठा सं करण, अ याय 73

68. न न ल खत म से कौन सा चन
ु ाव सध
ु ार सह नह ं है ?

(a) चुनाव के अं तम चरण समा त होने के बाद ह नगम मतनम


ु ान (एि जट पोल) के प रणाम
सा रत कए जा सकते ह।

(b) एक उ मीदवार को कसी वशेष लोकसभा चुनाव म एक से अ धक नवाचन े से चुनाव लड़ने


क अनम
ु त नह ं है।

(c) एक यि त कसी भी चन
ु ाव म मतदान करने के यो य नह ं है , य द वह जेल म बंद है ।
(d) चुनाव लड़ने के लए, एक यि त को संबं धत नवाचन े म मतदाता के प म पंजीकृत होना
चा हए।

उ र: b

या या:

कथन 1 सह है: नगम मतनम


ु ान (एि जट पोल) पर तबंध - चन
ु ाव आयोग ने 2019 के लोकसभा
चुनाव से पहले एक बयान जार कर कहा क नगम मतनम
ु ान (एि जट पोल) के नतीजे अं तम चरण
के चुनाव समा त होने के बाद ह सा रत कए जा सकते ह। यह संभा वत मतदाताओं को कसी भी
तरह से गम
ु राह या पव
ू ा ह से सत होने से बचाने के लए कया गया था।

कथन 2 गलत है : जन त न ध व अ ध नयम, 1951 क धारा 33 (7) के अनस


ु ार, कोई यि त
लोकसभा/ वधानसभा चुनाव के लए दो से अ धक नवाचन े से चुनाव नह ं लड़ सकता है ।

कथन 3 सह है: जन त न ध व अ ध नयम, 1951 क धारा 62(5) के अनस


ु ार, कोई भी यि त
कसी भी चुनाव म मतदान नह ं करे गा, चाहे वह जेल म बंद है , चाहे वह कारावास क सजा के
अधीन हो या दे श नकला हो या पु लस क कानूनी हरासत म हो।

कथन 4 सह है : एक उ मीदवार के प म चुनाव लड़ने के लए, एक यि त को मतदाता के प म


पंजीकृत होना चा हए। जन त न ध व अ ध नयम, 1951 क धारा 4 (d) कसी यि त के तब तक
चन
ु ाव लड़ने पर तबंध लगाती है , जब तक क वह कसी संसद य नवाचन े म नवाचक न
हो। जन त न ध व अ ध नयम, 1951 क धारा 5 (c) म वधानसभा े के लए समान ावधान
कया गया है।

सहायक अनुमान: 2014 के चुनाव म धानमं ी नर मोद ने वयं 2 अलग-अलग नवाचन े से


चुनाव लड़ा था।

ोत: एम ल मीकांत वारा भारतीय राजनी त छठा सं करण, अ याय 73

69. सं वधान के अनस


ु ार, एक धन वधेयक म न न ल खत म से कौन सा ावधान हो सकता है ?

1. जुमाना या अ य आ थक दं ड लगाना।
2. क सरकार वारा पैसे उधार लेने का व नयमन।

3. थानीय ा धकरण वारा थानीय उ दे य के लए कसी भी कर का अ धरोपण।

4. भारत क सं चत न ध म से धन का व नयोग।

5. भारत क आकि मकता न ध से धन क नकासी।

नीचे दए गए कूट का योग कर सह उ र चु नए:

(A) केवल 1, 2 और 3

(B) केवल 2 और 4

(C) केवल 2, 4, और 5

(D) केवल 1, 4, और 5

उ र: C

या या:

वक प (C) सह है: सं वधान के अनु छे द 110 के अनस


ु ार, एक वधेयक को धन वधेयक माना
जाता है य द इसम न न ल खत सभी या कसी भी मामले से संबं धत 'केवल' ावधान शा मल ह:

• कसी भी कर का अ धरोपण, उ मल
ू न, छूट, प रवतन या व नयमन।

• क सरकार वारा पैसे उधार लेने का व नयमन।

• भारत क सं चत न ध या भारत क आकि मकता न ध क अ भर ा, ऐसी कसी न ध म धन


का भग
ु तान या उससे धन क नकासी।

• भारत क सं चत न ध से धन का व नयोग।

• भारत क सं चत न ध पर भा रत कसी यय क घोषणा या ऐसे कसी यय क रा श म व ृ ध


करना।

• भारत क सं चत न ध या भारत के सावज नक खाते से धन क ाि त या ऐसे धन क अ भर ा


या जार करना, या संघ या रा य के खात क लेखा पर ा।

• ऊपर व न द ट मामल म से कोई भी आकि मक मामला।


हालाँ क, कसी वधेयक को केवल इस कारण से धन वधेयक नह ं माना जाता है क वह
न न ल खत के लए ावधान करता है :

• जुमाना या अ य आ थक दं ड लगाना, या

• दान क गई सेवाओं के लए लाइसस या शु क के लए शु क क मांग या भग


ु तान; या

• थानीय उ दे य के लए कसी थानीय ा धकरण या नकाय वारा कसी भी कर का


अ धरोपण, उ मल
ू न, छूट, प रवतन या व नयमन।

ोत: एम. ल मीकांत वारा भारतीय राजनी त- छठा सं करण-अ याय- 22।

70. न न ल खत कथन पर वचार कर:

1. लोकसभा के नव नवा चत सद य को शपथ दलाने वाले ोटे म अ य का चुनाव नव नवा चत


लोकसभा क पहल बैठक म कया जाता है।

2. 'ि हप' का कायालय संसद य स मेलन पर आधा रत होता है जब क संसद य सं व ध( नयम ) म


वप के नेता के कायालय का उ लेख कया गया है।

ऊपर दए गए कथन म से कौन-सा/से सह है/ह?

(A) केवल 1

(B) केवल 2

(C) दोन 1 और 2

(D) न तो 1 और न ह 2

उ र: B

या या:

कथन 1 गलत है : ोटे म अ य क नयिु त रा पत वारा क जाती है, जो आमतौर पर


लोकसभा का सबसे व र ठ सद य होता है । ोटे म अ य को रा पत वयं शपथ दलाते ह। ोटे म
पीकर के पास पीकर के सभी अ धकार होते ह। वह नव नवा चत लोकसभा क पहल बैठक क
अ य ता करता है । उनका मु य कत य नए सद य को शपथ दलाना है। वह सदन को नए अ य
का चन
ु ाव करने म भी स म बनाता है। जब सदन वारा नए अ य का चन
ु ाव कया जाता है, तो
अ य ोटे म के कायालय का अि त व समा त हो जाता है। इस लए, यह कायालय एक अ थायी
कायालय है , जो कुछ दन के लए व यमान है।

कथन 2 सह है: हालां क भारत के सं वधान म सदन के नेता और वप के नेता के कायालय का


उ लेख नह ं कया गया है, उनका उ लेख मशः सदन और संसद य क़ानन
ू के नयम म कया गया
है । दस
ू र ओर, 'ि हप' के पद का उ लेख न तो भारत के सं वधान म और न ह सदन के नयम म
और न ह कसी संसद य क़ानून म कया गया है । यह संसद य सरकार के स मेलन पर आधा रत
है ।

ोत: एम. ल मीकांत वारा भारतीय राजनी त- छठा सं करण-अ याय- 22।

71. संसद म बजट के पा रत होने के चरण को सह म म यवि थत कर:

1. वभागीय स म तय वारा जांच।

2. व वधेयक का पा रत होना।

3. अनद
ु ान क मांग पर मतदान।

4. व नयोग वधेयक का पा रत होना।

नीचे दए गए कूट का योग कर सह उ र चु नए:

(A) 1 - 2 - 3 - 4

(B) 1 - 3 - 4 - 2

(C) 2 - 1 - 3 - 4

(D) 1 - 4 - 3 - 2

उ र: B

या या:

वक प (B) सह है: बजट संसद म न न ल खत छह चरण से गुजरता है :


• बजट क तु त: परं परागत प से, बजट फरवर के अं तम काय दवस पर व मं ी वारा
लोकसभा म तुत कया जाता है । 2017 से, बजट क तु त को 1 फरवर तक बढ़ा दया गया
है । व मं ी बजट भाषण के साथ बजट पेश करते ह िजसे 'बजट भाषण' कहा जाता है। इसके
अलावा, िजस दन बजट को सदन म पेश कया जाता है उस दन उस पर कोई चचा नह ं होगी।

• बजट पर सामा य चचा: यह पेश होने के कुछ दन बाद शु होता है । यह संसद के दोन सदन
म होता है और आमतौर पर तीन से चार दन तक चलता है । इस चरण के दौरान, लोकसभा बजट
पर सम प से या उसम शा मल स धांत के कसी भी न पर चचा कर सकती है ले कन कोई
कटौती ताव पेश नह ं कया जा सकता है और न ह बजट सदन के वोट के लए तुत कया जा
सकता है ।

• वभागीय स म तय वारा जांच: बजट पर आम चचा समा त होने के बाद, संसद के दोन सदन
लगभग तीन से चार स ताह के लए थ गत हो जाते ह। इस अंतराल अव ध के दौरान, संसद क
24 वभागीय थायी स म तयां संबं धत मं य क अनद
ु ान मांग क जांच और चचा करती ह और
उन पर रपोट तैयार करती ह। ये रपोट संसद के दोन सदन म वचार के लए तत
ु क जाती ह।

• अनद
ु ान मांग पर मतदान: वभागीय थायी स म तय क रपोट के आलोक म लोकसभा मांग पर
मतदान करती है। मतदान बजट के मतदान यो य भाग तक ह सी मत है , भारत क सं चत न ध
पर भा रत यय को मत के लए तुत नह ं कया जाता है (इस पर केवल चचा क जा सकती है )।
मांग को मं ालयवार तत
ु कया जाता है। येक मांग पर लोकसभा वारा अलग से मतदान
कया जाता है। इस चरण के दौरान, संसद सद य बजट के ववरण पर चचा कर सकते ह। वे
अनद
ु ान क कसी भी मांग को कम करने के लए ताव भी ला सकते ह। ऐसे ताव को 'कट
मोशन' कहा जाता है । व धवत मतदान के बाद एक मांग अनद
ु ान बन जाती है । रा यसभा, जो केवल
इस पर चचा कर सकती है और अनद
ु ान क मांग पर मतदान करने क कोई शि त नह ं है।

• व नयोग वधेयक का पा रत होना: व नयोग के लए भारत क सं चत न ध म से, लोक सभा


वारा दए गए अनद
ु ान को परू ा करने के लए आव यक सभी धन और भारत क सं चत न ध पर
भा रत यय क पू त करने के लए एक व नयोग वधेयक पेश कया जाता है। व नयोग वधेयक
रा पत क वीकृ त के बाद व नयोग अ ध नयम बन जाता है। यह अ ध नयम भारत क सं चत
न ध से भग
ु तान को अ धकृत (या वैध) करता है ।

• व वधेयक का पा रत होना: व वधेयक को अगले वष के लए भारत सरकार के व ीय


ताव को भावी बनाने के लए पेश कया जाता है । यह धन वधेयक पर लागू सभी शत के
अधीन है । व नयोग वधेयक के वपर त, व वधेयक के मामले म संशोधन (कर को अ वीकार या
कम करने क मांग) को थानांत रत कया जा सकता है। 1931 के अनं तम कर सं ह अ ध नयम
के अनस
ु ार, व वधेयक को 75 दन के भीतर अ ध नय मत कया जाना चा हए (अथात, संसद
वारा पा रत और रा पत वारा अनम
ु ो दत)। व अ ध नयम बजट के आय प को वैध बनाता है
और बजट के अ ध नयमन क या को परू ा करता है ।

ोत: एम. ल मीकांत वारा राजनी त- छठा सं करण-अ याय- 22।

72. क सरकार के संदभ म, न न ल खत कथन पर वचार क िजएः

1. एन. गोपाल वामी अयंगर स म त ने सझ


ु ाव दया क एक मं ी और एक स चव को केवल
शास नक सध
ु ार के वषय को आगे बढ़ाने और इसे बढ़ावा दे ने के लए ना मत कया जाना चा हए।

2. 1970 म, शास नक सध
ु ार आयोग, 1966 क सफा रश पर का मक वभाग का गठन कया गया
था और इसे धान मं ी के भार म रखा गया था।

ऊपर दए गए कथन म से कौन-सा/से सह है/ह?

(A) केवल 1

(B) केवल 2

(C) दोन 1 और 2

(D) न तो 1 और न ह 2

उ र: B

या या:

कथन 1 गलत है: ी। एन. गोपाल वामी अ यंगार ने 1950 म 'क सरकार क मशीनर का
पन
ु गठन' शीषक वाल अपनी रपोट म मं ालय के समह
ू करण और क मय क मताओं म सध
ु ार
क सफा रश क । इसम एक मं ी और एक स चव को केवल पीछा करने के उ दे य से ना मत कया
गया है और शास नक सध
ु ार को बढ़ावा दे ना।

कथन 2 सह है : शास नक सध
ु ार आयोग क सफा रश के आधार पर 1970 म कै बनेट स चवालय
म का मक वभाग क थापना क गई थी। इसे धान मं ी कायालय को स पा गया था।
ोत: यप
ू ीएससी सीएसई 2021

73. क सरकार के संदभ म, न न ल खत कथन पर वचार क िजएः

1. भारत के सं वधान म ावधान है क सभी कै बनेट मं ी अ नवाय प से केवल लोकसभा के


मौजद
ू ा सद य ह गे।

2. क य कै बनेट स चवालय संसद य काय मं ालय के नदशन म काय करता है ।

ऊपर दए गए कथन म से कौन-सा/से सह है/ह?

(A) केवल 1

(B) केवल 2

(C) दोन 1 और 2

(D) न तो 1 और न ह 2

उ र: D

या या:

कथन 1 गलत है: भारत के सं वधान के मल


ू पाठ म कै बनेट श द का उ लेख नह ं है। इसे 44व
सं वधान संशोधन अ ध नयम वारा 1978 म सं वधान के अनु छे द 352 म सि म लत कया गया
था। इस लए, सं वधान सभी कै बनेट मं य के अ नवाय प से केवल लोकसभा के मौजद
ू ा सद य
होने के संबध
ं म कुछ भी दान नह ं करता है ।

कथन 2 गलत है: क य कै बनेट स चवालय धान मं ी (संसद य काय मं ालय नह ं) के नदशन म
काय करता है ।

कै बनेट स चवालय के बारे म:


• यह भारत सरकार (कारोबार का लेन-दे न) नयम, 1961 और भारत सरकार ( यवसाय का आवंटन)
नयम 1961 के शासन के लए िज मेदार है , िजससे सरकार के मं ालय / वभाग म यापार के
सच
ु ा संचालन क सु वधा मलती है।

• यह स चवालय मं मंडल और उसक स म तय को स चवीय सहायता दान करता है

• यह अंतर-मं ालयी सम वय सु नि चत करके, मं ालय / वभाग के बीच मतभेद को दरू करके और


स चव क थायी/तदथ स म तय के मा यम से आम सहम त वक सत करके सरकार म नणय
लेने म सहायता करता है ।

ोत: यप
ू ीएससी सीएसई पीवाई यू 2009

74. न न ल खत कथन पर वचार कर:

1. ऋण के पन
ु भगतान
ु म सरकार वारा ा त सम त धन भारत क सं चत न ध का भाग होता
है ।

2. भारत के सावज नक खाते से कए गए भग


ु तान यादातर ब कं ग लेनदे न क कृ त म होते ह।

3. भारत क आकि मकता न ध, जो कायकार कारवाई वारा संचा लत होती है , रा प त क ओर


से व स चव के पास होती है।

ऊपर दए गए कथन म से कौन-सा/से सह है/ह?

(A) केवल 1 और 3

(B) केवल 2

(C) केवल 2 और 3

(D) 1, 2 और 3

उ र: D

या या:
कथन 1 सह है : अनु छे द 266(1) भारत क एक सं चत न ध के गठन का ावधान करता है । यह
एक ऐसा कोष है िजसम सभी ाि तय को जमा कया जाता है और सभी भग
ु तान को डे बट कया
जाता है । उसमे समा व ट ह:

 भारत सरकार वारा ा त सभी राज व।


 सरकार वारा े जर बल, ऋण या अ म के तर के और साधन जार करके उठाए गए सभी
ऋण।
 सरकार वारा ऋण क अदायगी म ा त सभी धन
 भारत सरकार क ओर से सभी कानन
ू ी प से अ धकृत भग
ु तान इस फंड से कए जाते ह।
इस न ध म से कोई भी धन संसद य कानन
ू के अनस
ु ार व नयोिजत (जार या आह रत)
नह ं कया जा सकता है ।
कथन 2 सह है : भारत सरकार वारा या उसक ओर से ा त अ य सभी सावज नक धन (उनके
अलावा जो भारत क सं चत न ध म जमा कए गए ह) को अनु छे द 266 के ावधान के अनस
ु ार
भारत के लोक खाते म जमा कया जाएगा। 2))। इसम भ व य न ध जमा, या यक जमा, बचत
बक जमा, वभागीय जमा और ेषण शा मल ह। यह खाता कायकार कारवाई वारा संचा लत होता
है , अथात इस खाते से भग
ु तान संसद य व नयोग के बना कया जा सकता है । इस तरह के भग
ु तान
यादातर ब कं ग लेनदे न क कृ त म होते ह।

कथन 3 सह है : अनु छे द 267 के तहत सं वधान ने संसद को 'भारत क आकि मकता न ध'
था पत करने के लए अ धकृत कया, िजसम समय-समय पर कानन
ू वारा नधा रत रा श का
भग
ु तान कया जाता है । त नुसार, संसद ने 1950 म भारतीय आकि मकता न ध अ ध नयम
बनाया। यह न ध रा प त के अ धकार म है , और वह संसद वारा इसक अनम
ु त मलने तक
अ या शत यय को परू ा करने के लए इसम से अ म कर सकता है । यह कोष रा प त क ओर
से व स चव के पास होता है । भारत के लोक लेखा क भाँ त यह भी कायपा लका या वारा
संचा लत होता है।

ोत: एम. ल मीकांत वारा राजनी त- छठा सं करण-अ याय- 22

75. न न ल खत म से कौन सा वधेयक लोकसभा के भंग होने पर यपगत नह ं होता है ?

1. लोकसभा वारा पा रत वधेयक ले कन रा यसभा म लं बत

2. एक वधेयक रा यसभा म लं बत है ले कन लोकसभा वारा पा रत नह ं कया गया है

3. एक वधेयक दोन सदन वारा पा रत ले कन रा प त क सहम त के लए लं बत


नीचे दए गए कूट का योग कर सह उ र चु नए:

(A) केवल 1

(B) केवल 1 और 2

(C) केवल 2

() केवल 2 और 3

उ र: D

या या:

वक प (D) सह है : जब लोकसभा भंग हो जाती है , तो उसके या उसक स म तय के सम लं बत


बल, ताव, संक प, नो टस, या चका आ द स हत सभी काय समा त हो जाते ह। उ ह ( आगे
बढ़ाने के लए) नवग ठत लोकसभा म फर से पेश कया जाना चा हए। हालां क, कुछ लं बत बल
और सभी लं बत आ वासन िजनक जांच सरकार आ वासन पर स म त वारा क जानी है ,
लोकसभा के वघटन पर समा त नह ं होते ह। बल के यपगत होने के संबध
ं म ि थ त इस कार
है :

 लोकसभा म लं बत कोई वधेयक यपगत हो जाता है (चाहे वह लोकसभा म उ प न हुआ हो


या रा य सभा वारा उसे े षत कया गया हो)।
 लोकसभा वारा पा रत ले कन रा यसभा म लं बत वधेयक यपगत हो जाता है।
 असहम त के कारण दोन सदन वारा पा रत नह ं कया गया वधेयक और य द रा प त ने
लोकसभा के वघटन से पहले संयु त बैठक आयोिजत करने क अ धसच
ू ना द है , तो वह
यपगत नह ं होता है ।
 रा यसभा म लं बत ले कन लोकसभा वारा पा रत नह ं कया गया एक वधेयक यपगत
नह ं होता है ।
 दोन सदन वारा पा रत वधेयक ले कन रा पत क वीकृ त के लए लं बत नह ं है।
 दोन सदन वारा पा रत ले कन सदन के पन
ु वचार के लए रा पत वारा लौटाया गया
वधेयक यपगत नह ं होता है ।
ोत: एम. ल मीकांत वारा राजनी त- छठा सं करण-अ याय- 22
76. संसद क व ीय स म तय के संबध
ं म न न ल खत कथन पर वचार कर:

1. भारत सरकार अ ध नयम, 1919 के ावधान के तहत सबसे पहले 1921 म लोक लेखा स म त
और ा कलन स म त दोन का गठन कया गया था।

2. कसी मं ी को लोक लेखा स म त या ा कलन स म त के सद य के प म नह ं चुना जा सकता


है ।

3. सावज नक उप म स म त के अ य क नयिु त संसद के कसी भी सदन से क जा सकती है।

4. लोक लेखा स म त के वपर त, ा कलन स म त और सावज नक उप म स म त क सफा रश


मं ालय के लए बा यकार नह ं ह।

ऊपर दए गए कतने कथन सह ह/ह?

(A) केवल एक बयान

(B) केवल दो बयान

(C) केवल तीन बयान

(D) सभी चार बयान

उ र: A

या या:

कथन 1 गलत है : लोक लेखा स म त क थापना 1921 म भारत सरकार अ ध नयम 1919 के
ावधान के तहत क गई थी और तब से यह अि त व म है। वतमान म, इसम 22 सद य
(लोकसभा से 15 और रा य सभा से 7) शा मल ह। जब क, इस ा कलन स म त क उ प का
पता 1921 म था पत थायी व ीय स म त से लगाया जा सकता है। वतं ता के बाद के युग म
पहल ा कलन स म त का गठन 1950 म त काल न व मं ी जॉन मथाई क सफा रश पर कया
गया था। मल
ू प से, इसम 25 सद य थे ले कन 1956 म इसक सद यता बढ़ाकर 30 कर द गई
थी। सभी तीस सद य केवल लोकसभा से ह। इस स म त म रा यसभा का कोई त न ध व नह ं है।

कथन 2 सह है : लोक लेखा स म त के सद य (लोकसभा से 15 और रा य सभा से 7) हर साल


संसद के सद य वारा आनप
ु ा तक त न ध व के स धांत के अनस
ु ार एकल सं मणीय मत के
मा यम से अपने सद य म से चुने जाते ह। ा कलन स म त के सद य केवल लोकसभा से आते
ह। इन 30 सद य का चुनाव येक वष लोकसभा के सद य वारा अपने सद य म से
आनप
ु ा तक त न ध व के स धांत के अनस
ु ार एकल सं मणीय मत के मा यम से कया जाता है ।
एक मं ी को लोक लेखा स म त या ा कलन स म त के सद य के प म नह ं चुना जा सकता है ।

लोक लेखा स म त के अ य क नयिु त अ य वारा अपने सद य म से क जाती है । 1966 -


67 तक, स म त के अ य स ाधार दल के थे। हालाँ क, 1967 के बाद से एक परं परा वक सत हुई
है िजसके तहत स म त के अ य को हमेशा वप से चुना जाता है । जब क, ा कलन स म त के
अ य क नयिु त अ य वारा अपने सद य म से क जाती है और वह नरपवाद प से
स ाधार दल से होता है।

कथन 3 गलत है: सावज नक उप म क स म त के अ य क नयिु त अ य वारा अपने


सद य म से क जाती है , जो केवल लोकसभा से चन
ु े जाते ह। इस कार, स म त के सद य जो
रा यसभा से ह, उ ह अ य के प म नयु त नह ं कया जा सकता है।

कथन 4 गलत है : तीन व ीय स म तय , अथात लोक लेखा स म त, ा कलन स म त और


सावज नक उप म स म त क सफा रश मं ालय पर बा यकार नह ं ह।

ोत: एम. ल मीकांत वारा राजनी त- छठा सं करण-अ याय- 23।

77. 'मेर पॉ लसी मेरे हाथ' अ भयान न न ल खत म से कस योजना से संबं धत है ?

(a) धान मं ी फसल बीमा योजना (पीएमएफबीवाई)

(b) धान मं ी कौशल वकास योजना 3.0 (पीएमकेवीवाई 3.0)

(c) धान मं ी कसान स मान न ध (पीएम- कसान)

(d) धान मं ी सरु ा बीमा योजना (पीएमएसबीवाई)

उ र: a

या या:

वक प (a) सह है: "मेर पॉ लसी, मेरे हाथ" धानमं ी फसल बीमा योजना (फसल बीमा पॉ लसी)
का एक ह सा है। इस पहल के अंतगत, धानमं ी फसल बीमा योजना के तहत दे श भर के बीमा
लेने वले सभी कसान को पॉ लसी के द तावेज़ उनके दरवाजे पर मलग। अ भयान का उ दे य,
सभी कसान को धानमं ी फसल बीमा योजना(पीएमएफबीवाई) के तहत अपनी नी तय , भू म
अ भलेख , दावे क या और शकायत नवारण आ द के संबध
ं म सभी जानकार से अ छ तरह
से अवगत कराना सु नि चत करना है।

वक प (b) गलत है : कौशल वकास और उ य मता मं ालय (एमएसडीई) ने जनवर 2021 म


अपनी मख
ु योजना धान मं ी कौशल वकास योजना 3.0 (पीएमकेवीवाई 3.0) का तीसरा चरण
शु कया है । यह परू े दे श म उ योग क आव यकताओं को परू ा करने, बाजार क मांग को परू ा
करने, सेवाओं म कौशल दान करने और वशेष प से महामार के बाद के नए यग
ु क नौकर क
भू मकाओं म, कौशल वकास को ो सा हत और बढ़ावा दे गा। ।

वक प (c) गलत है : धान मं ी कसान स मान न ध (पीएम- कसान) 24 फरवर , 2019 को भू म


धारक कसान क व ीय ज रत को परू ा करने के लए शु क गई एक क य े क योजना है।
य लाभ अंतरण (डीबीट ) ा प के मा यम से दे श भर के कसान प रवार के बक खात म हर
चार मह ने म तीन समान क त म 6000/- पये त वष का व ीय लाभ ह तांत रत कया जाता
है । यह योजना शु म छोटे और सीमांत कसान (एसएमएफ), िजनके पास 2 हे टे यर तक क भू म
थी, के लए थी, ले कन योजना का दायरा 01.06.2019 से सभी भू मधारक कसान को इसम
शा मल कर लया गया था।

वक प (d) गलत है : धान मं ी सरु ा बीमा योजना (पीएमएसबीवाई) एक साल क दघ


ु टना बीमा
योजना है , जो साल-दर-साल नवीकरणीय है और दघ
ु टना के कारण मृ यु या वकलांगता को इसम
शा मल करती है । बचत बक या डाकघर म खाता रखने वाले 18-70 वष के आयु वग के यि त इस
योजना के तहत नामांकन के पा ह। योजना के तहत जो खम कवरे ज दघ
ु टना म ृ यु और पण

वकलांगता के लए 2 लाख पये और आं शक वकलांगता के लए 1 लाख पये है । ।

ोत:https://pib.gov.in/PressReleaseIframePage.aspx?PRID=1799297

https://pib.gov.in/PressReleasePage.aspx?PRID=1788919

https://pib.gov.in/PressReleasePage.aspx?PRID=1800851

https://pib.gov.in/PressReleaseIframePage.aspx?PRID=1823754

78. न न ल खत कथन पर वचार क िजये:

1. ह रत ऋणप और जलवायु ऋणप दोन नि चत या नय मत आय वाले व ीय साधन ह।


2. ह रत ऋणप के वपर त, जो सरकार और नजी दोन नगम वारा जार कया जा सकता है ,
जलवायु ऋणप (बांड) केवल सरकार या बहुप ीय सं थाओं वारा जार कए जा सकते ह।

3. नीले ऋणप ( लू बॉ ड), ह रत ऋणप का एक उपसमह


ू है , जो वशेष प से समु संर ण से
संबं धत प रयोजनाओं के व पोषण के लए उपयोग कया जाता है ।

ऊपर दए गए कथन म से कौन-सा/से सह है/ह?

(a) केवल 1 और 2

(b) केवल 2

(c) केवल 1 और 3

(d) केवल 2 और 3

उ र: c

या या:

कथन 1 सह है: जलवायु ऋणप , नि चत आय वाले व ीय साधन ह, जो जलवायु प रवतन


समाधान से संबं धत होते ह। वे जलवायु प रवतन समाधान के लए व जुटाने के लए जार कए
जाते ह, उदाहरण के लए शमन या अनक
ु ू लन संबध
ं ी प रयोजनाएं। ये व छ ऊजा से लेकर ऊजा
द ता, या जलवायु प रवतन अनक
ु ू लन प रयोजनाओं तक ीनहाउस गैस उ सजन म कमी करने
वाल प रयोजनाएं हो सकती ह। अ धकांश जलवायु ऋणप , ा त आय/ व के उपयोग से संबं धत
ऋणप होते ह, जहां जार कता नवेशक से वादा करता है क इनके मा यम से ा त धन केवल
न द ट जलवाय-ु संबं धत काय म या प रसंप य , जैसे अ य ऊजा संयं या जलवायु शमन न ध
काय म म ह यय कया जायेगा।

दस
ू र ओर, व व बक के अनस
ु ार, ह रत ऋणप व ीय साधन ह, जो ह रत प रयोजनाओं को
व पो षत करते ह और नवेशक को नय मत या नि चत आय भग
ु तान दान करते ह। सकारा मक
पयावरणीय और जलवायु लाभ वाल प रयोजनाओं को न ध दे ने के लए ह रत ऋणप क शु आत
क गयी थी। पछले 14 वष म, जलवायु प रवतन और संबं धत चुनौ तय के भाव को दरू करने
के लए ह रत ऋणप एक मह वपण
ू व ीय साधन बन गया है। पहला ह रत ऋणप 2007 म
बहुप ीय सं थान , यूरोपीय नवेश बक (ईआईबी) और व व बक वारा जार कया गया था।
कथन 2 गलत है: ह रत ऋणप , बहुप ीय सं थाओं जैसे व व बक, नगम , सरकार सं थाओं और
नगर पा लकाओं वारा जार कए जाते ह। 2007 और 2012 के बीच क अव ध म, यूरोपीय नवेश
बक और व व बक जैसे अंतरा य संगठन के साथ-साथ सरकार ने भी अ धकांश ह रत ऋणप
जार कए। तब से, नगम के हत म तेजी से व ृ ध हुई है । 2014 म, ऊजा और उपयो गताओं,
उपभो ता व तओ
ु ं और रयल ए टे ट े म नगम वारा जार कए गए ऋणप का बाजार का
एक तहाई ह सा था। इसके अ त र त, सामा य ऋणप क तरह, जलवायु ऋणप सरकार , बहु-
रा य बक या नजी नगम वारा जार कए जा सकते ह।

कथन 3 सह है : नीले ऋणप ( लू बॉ ड), ह रत ऋणप का एक उपसमह


ू है , जो वशेष प से
समु संर ण से संबं धत प रयोजनाओं को व पो षत करने के लए उपयोग कया जाता है । इसम
लाि टक कचरे का बंधन शा मल है, ले कन टकाऊ, व छ और पा रि थ तक प से अनक
ु ूल
वकास सु नि चत करके समु जैव व वधता को बढ़ावा दे ना भी शा मल है। हाल ह म व ीय
बाजार नयामक भारतीय तभू त और व नमय बोड ने थायी व के एक मोड के प म नीले
ऋणप ( लू बॉ ड)क अवधारणा का ताव दया है , िजसम कहा गया है क ऐसी तभू तय का
उपयोग व भ न नील अथ यव था से संबं धत ग त व धय के लए कया जा सकता है , िजसम
समु संसाधन खनन और टकाऊ मछल पकड़ना शा मल है ।

ोत:https://bondvigilantes.com/blog/2021/02/green-bonds-blue-bonds-esg-bonds-galore-a-
beginners-guide-for-fixed-income-
investors/#:~:text=Blue%20bonds%20are%20a%20subset,clean%20and%20ecologically%2Dfriendly
%20developments.

https://economictimes.indiatimes.com/markets/stocks/news/sebi-proposes-blue-bonds-concept-
for-sustainable-financing-activities/articleshow/93433121.cms?from=mdr

https://www.climatebonds.net/resources/understanding

https://www.thehindu.com/business/what-are-green-bonds/article7070840.ece

https://www.worldbank.org/en/news/feature/2021/12/08/what-you-need-to-know-about-ifc-s-
green-bonds

79. आभासी डिजटल स प य के संबध


ं म, न न ल खत कथन पर वचार क िजये:

1. बजट 2022–2023 ने आभासी डिजटल संप के खर दार के लए व े ता को भग


ु तान क गई
रा श का 1 तशत ोत पर कर क कटौती (ट डीएस) को अ नवाय कर दया।
2. आयकर अ ध नयम 1961 के अनस
ु ार, "आभासी डिजटल संप " का अथ लॉकचेन तकनीक के
मा यम से उ प न कोई भी सांके तक धन है ।

3. बजट 2022–2023 म कसी भी आभासी डिजटल संप के ह तांतरण से होने वाल आय पर


30% कर का वधान कया गया है ।

ऊपर दए गए कथन म से कौन-से सह ह?

(a) केवल 1 और 2

(b) केवल 1 और 3

(c) केवल 2 और 3

(d) 1, 2 और 3

उ र: b

या या:

कथन 1 और 3 सह ह: व मं ी ने 2022 और 2023 के बजट के दौरान घोषणा क क कसी भी


आभासी डिजटल संप के ह तांतरण से होने वाल कसी भी आय पर 30 तशत क दर से कर
लगाया जाएगा। इसके अ त र त, आभासी संप पर 30% कर का उ लेख कसी अ य आय के
त थापन के प म नह ं कया जा सकता है (िजसका अथ है क इसे कटौती के लए दावा नह ं
कया जा सकता है )। इसके अ त र त, लेन-दे न पर नज़र रखने के लए डिजटल संप का उपयोग
करके कए गए भग
ु तान पर 1% ोत पर कर क कटौती (ट डीएस) क जाएगी।

कथन 2 गलत है : वतमान म भारत म टोकरसी और अपरू णीय टोकन (एनएफट ) गैर- व नमय
डिजटल स प ह। आयकर अ ध नयम 1961 के अनस
ु ार, 'आभासी डिजटल स प ' का अथ है
कोई भी सच
ू ना या सं हता या नंबर या टोकन (भारतीय मु ा या वदे शी मु ा नह ं), टो ा फ़क
मा यम से या अ यथा, कसी भी ोत से, को द शत करती है जो मू य का डिजटल तनध व
दान करता है । कसी भी व ीय लेनदे न या नवेश म इसके उपयोग स हत, अंत न हत मू य के
वादे या दशन के साथ, या मू य के भंडार या खाते क एक इकाई के प म काय करता है , ले कन
नवेश योजना तक सी मत नह ं है ; और इले ॉ नक प से थानांत रत, सं ह त या यापार कया
जा सकता है । आयकर अ ध नयम, आभासी डिजटल स प क प रभाषा म लॉकचेन या डीएलट
का उ लेख नह ं करता है ।
ोत:https://economictimes.indiatimes.com/wealth/tax/budget-2022-levies-30-tax-and-tds-on-
crypto-
assets/articleshow/89267756.cms?utm_source=contentofinterest&utm_medium=text&utm_campai
gn=cppst

https://economictimes.indiatimes.com/wealth/tax/budget-2022-levies-30-tax-and-tds-on-crypto-
assets/articleshow/89267756.cms

https://www.forbes.com/advisor/in/investing/cryptocurrency/cryptocurrency-tax-in-india/

80. भारत म केसर क खेती के संदभ म न न ल खत कथन पर वचार क िजएः

1. केसर क खेती लंबे समय से ज म-ू क मीर के कुछ िजल तक ह सी मत रह है।

2. उ र- पव
ू ो यौ गक अनु योग एवं सार क (नॉथ ई ट सटर फॉर टे नोलॉजी ए ल केशन एंड
र च), उ र पव
ू रा य म केसर बाउल प रयोजना के काया वयन के लए उ रदायी है।

3. पव
ू र भारत म पहल बार सि कम रा य म केसर क सफलतापव
ू क खेती क गई।

4. हाल ह म, असम और मेघालय के कुछ िजल को केसर क खेती के लए उपयु त थान के प


म चि हत कया गया है ।

ऊपर दए गए कथन म से कौन-से सह ह?

(a) केवल 1 और 3

(b) केवल 2 और 4

(c) केवल 1, 2 और 3

(d) केवल 2, 3 और 4

उ र: c

या या:

कथन 1 सह है : माना जाता है क केसर क खेती क मीर म पहल शता द ईसा पव


ू के आसपास
म य ए शयाई वा सय वारा शु क गई थी। ाचीन सं कृत सा ह य म केसर को 'बहुकम' कहा
गया है । केसर क खेती लंबे समय से ज म-ू क मीर म सी मत भौगो लक े तक सी मत रह है ,
मु य प से पंपोर, इसके बाद बडगाम, ीनगर और क तवाड़ िजले इसके अंतगत शा मल ह।
क मीर म तीन कार का केसर पाया जाता है - 'ल छा केसर', िजसम व तका केवल फूल से अलग
हो जाते ह और बना कसी सं करण के सख
ू जाते ह; 'म गरा केसर', िजसम फूल से व तका को
अलग कया जाता है , धूप म सख
ु ाया जाता है और पारं प रक प से संसा धत कया जाता है ; और
'गु छ केसर', जो ल छा केसर के समान है , सवाय इसके क बाद के सख
ू े व तका हवा-बंद कंटे नर
म श थल प म पैक कए जाते ह, जब क पव
ू के व तका एक कपड़े के धागे से बंधे बंडल म एक
साथ जड़
ु जाते ह। भारत म सालाना लगभग 6 से 7 टन केसर क खेती होती है, ले कन 100 टन
क मांग को परू ा करने के लए अ धकतर केसर ईरान से आयात कया जा रहा है।

कथन 2 सह है : केसर का कटोरा, जो अब तक क मीर तक ह सी मत था, उसी गण


ु व ा के साथ
े म केसर उगाने क यवहायता का पता लगाने के लए केसर बाउल प रयोजना के मा यम से
पव
ू र म व ता रत कया जा रहा है। उ र- पव
ू ो यौ गक अनु योग एवं सार क , उ र पव

रा य म केसर बाउल प रयोजना के काया वयन के लए उ रदायी है ।

कथन 3 सह है ले कन कथन 4 गलत है : पव


ू र ने द ण सि कम के यांगग
ं गाँव म पहल बार
केसर क सफलतापव
ू क खेती क । हाल ह म, केसर बाउल प रयोजना के तहत उ र- पव
ू ो यौ गक
अनु योग एवं सार क ने केसर क खेती के लए अ णाचल दे श और मेघालय म कुछ थान को
चि हत कया है ।

ोत:https://indianexpress.com/article/india/an-experiment-to-grow-saffron-in-sikkim-with-help-
from-jk-7034311/

https://www.thehindu.com/news/national/other-states/kashmir-saffron-gets-gi-
tag/article31484569.ece

https://dst.gov.in/nectar-brings-saffron-bowl-northeast-boosts-technology-sustainable-solutions-
regions-challenges

81. सरायघाट के यु ध के संदभ म न न ल खत कथन पर वचार क िजएः

1. ल चत बोरफुकन ने सरायघाट क लड़ाई म मग


ु ल के खलाफ अहोम सा य का नेत ृ व कया।

2. सरायघाट का यु ध, अहोम और मग
ु ल यो धाओं के म य एक नणायक नौसै नक यु ध था।

3. सरायघाट क लड़ाई ने मग
ु ल को असम को वापस लेने से रोक दया।
ऊपर दए गए कथन म से कौन-सा/से सह है/ह?

(a) केवल 1 और 2

(b) केवल 1 और 3

(c) केवल 3

(d) 1, 2 और 3

उ र: d

या या:

कथन 1 सह है : अहोम राजाओं ने 13वीं और 19वीं शता द के बीच लगभग 600 वष तक, जो अब
असम के नाम से जाना जाता है और जो आज पड़ोसी रा य ह, के बड़े ह से पर शासन कया।
ल चत बोरफुकन, अहोम सा ा य म एक सेनाप त था, िजसने 1671 म सरायघाट के यु ध म नेत ृ व
कया था, िजसने मग
ु ल यो धाओं वारा कए गए एक यास को वफल कर दया था।

कथन 2 और 3 सह ह: सरायघाट के यु ध को मग
ु ल पर जीत हा सल करने के लए नद पर सबसे
बड़े नौसै नक यु ध म से एक माना जाता था। अलाबोई के वपर त, जहां उसे नौसै नक यु ध के
बजाय जमीन पर लड़ने के लए मजबरू कया गया था, सरायघाट म ल चत ने मग
ु ल को एक
नौसै नक यु ध म ललचाया। इ तहासकार एच के बरपज
ु ार के अनस
ु ार, अहोम बल ने एक आगे से
हमला और पीछे से एक अचानक प से हमला कया। उ ह ने आगे से कुछ जहाज के साथ हमले
का बहाना करके मग
ु ल बेड़े को आगे बढ़ने का लालच दया। मग
ु ल ने अपने पीछे पानी खाल कर
दया, जहां से मु य अहोम बेड़े ने हमला कया और एक नणायक जीत हा सल क । यु ध ने मग
ु ल
को असम को वापस लेने से रोक दया। भारतीय वतं ता आंदोलन शु होने से लगभग दो शता द
पहले अ ल
ै 1672 म ल चत बोरफुकन क म ृ यु हो गई। आज, ल चतबरफुकन क जीत को रा य
र ा अकादमी (एनडीए) से नातक होने वाले सव े ठ कैडेट को हर साल दए जाने वाले वण पदक
से स मा नत कया जाता है ।

ोत:https://www.thehindu.com/news/national/other-states/pm-modi-slammed-for-calling-17th-
century-ahom-general-a-freedom-fighter/article34066126.ece

https://indianexpress.com/article/explained/explained-ahom-warrior-lachit-borphukan-and-the-
battles-of-alaboi-saraighat-7791371/

82. न न ल खत कथन पर वचार क िजये:


1. दे श म तटरे खा प रवतन और समु के तर म व ृ ध के भाव का अ ययन करने के लए
संयु त रा पयारण काय म (यए
ू नईपी) के सहयोग से पयावरण मं ालय वारा एक कृत तट य े
बंधन प रयोजना शु क गई थी।

2. हाल ह म, भारतीय रा य महासागर सच


ू ना सेवा क वारा तट य भे यता सच
ू कांक जार कया
गया।

3. तट य सभ
ु े यता सच
ू कांक, सापे जो खम का उपयोग करता है क भौ तक प रवतन समु के
तर म व ृ ध के प म ह गे, जो क वार क सीमा, लहर क ऊंचाई, तट य ढलान आ द जैसे
मापदं ड के आधार पर नधा रत कए जाते ह।

ऊपर दए गए कथन म से कौन-सा/से सह है/ह?

(a) केवल 1 और 2

(b) केवल 2

(c) केवल 3

(d) केवल 2 और 3

उ र: d

या या:

कथन 1 गलत है : एक कृत तट य े बंधन प रयोजना 7,500 कलोमीटर क तटरे खा और दे श म


तटरे खा प रवतन और समु के तर म व ृ ध के भाव का अ ययन करने का एक यास है । यह
दे श के तट य और समु पयावरण क सरु ा एवं संर ण के उ दे य से 2010 से 2020 तक
गज
ु रात, ओ डशा और पि चम बंगाल के चि हत े म व व बक [संयु त रा पयारण काय म,
नह ं] वारा सहायता ा त एक प रयोजना है । इसम एक कृत तट य े बंधन योजना
(आईसीजेडएम) का वकास, म ट के कटाव क रोकथाम, व ृ ारोपण, म ोव व ृ ारोपण क पंि तयाँ
(शे टरबे ट), पा रि थ तक तं क नगरानी के लए आव यकताओं को मजबत
ू करना, जैव
व वधता संर ण और तट य समद
ु ाय क थायी आजी वका शा मल है। व व बक सॉ ट लोन /
अंतरा य वकास संघ (आईडीए) साख का योगदान दे ता है , िजसम कुल प रयोजना लागत का
लगभग 78% शा मल है। यह एक कृत तट य े बंधन प रयोजना, जलवायु प रवतन के संदभ म
वशेष मह व रखती है , य क जलवायु प रवतन पर अंतर्- सरकार पैनल (आईपीसीसी) के नि चत
न कष म से एक वैि वक तापन के प रणाम व प समु के तर म व ृ ध से संबं धत है।
कथन 2 और 3 सह ह: भारतीय रा य महासागर सच
ू ना सेवा क , जो प ृ वी व ान मं ालय के
तहत एक वाय नकाय है , संभा वत म य े , महासागर रा य पव
ू ानुमान, सन
ु ामी पव
ू चेतावनी,
सम पत महासागर मॉड लंग, अवलोकन, गणना सु वधाओं और समु आँकड़ क के मा यम से
ारं भक चेतावनी, उ च तरं ग से संबं धत, तफ
ू ान इ या द से संबं धत चेतावनी जार करता रहा है।
भारतीय रा य महासागर सच
ू ना सेवा क , ने तट य सभ
ु े यता सच
ू कांक (सीवीआई) तैयार करने के
लए 1:1,00,000 पैमान पर 156 मान च वाला एक मान च लाने के लए रा य के तर पर परू े
भारतीय तट के लए तट य भे यता मू यांकन कया है। जब क मान च भारतीय तट के लए
भौ तक और भव
ू ै ा नक मापदं ड के आधार पर भ व य के समु - तर म व ृ ध के कारण तट य
जो खम का नधारण करते ह, तट य सभ
ु े यता सच
ू कांक (सीवीआई), सापे जो खम का उपयोग
करता है क भौ तक प रवतन समु के तर म व ृ ध के प म ह गे, जो क - वार क सीमा ;
लहर क ऊंचाई; तट य ढलान; तट य ऊंचाई; तटरे खा प रवतन दर; भ-ू आकृ त व ान; और सापे
समु - तर प रवतन क ऐ तहा सक दर आ द जैसे मापदं ड के आधार पर नधा रत कए जाते ह।

ोत:https://pib.gov.in/newsite/erelcontent.aspx?relid=60204

https://pib.gov.in/PressReleasePage.aspx?PRID=1812037

https://www.thehindu.com/news/national/telangana/incois-prepares-coastal-vulnerability-
index/article38396960.ece

83. हाइ ोजन के संबध


ं म न न ल खत कथन पर वचार क िजये:

1. नील ( ल)ू हाइ ोजन, धूसर ( )े हाइ ोजन का एक व छ और स ता वक प है।

2. धूसर ( )े हाइ ोजन, आज व व म उ पा दत कुल हाइ ोजन का लगभग 95% ह सा है।

3. गल
ु ाबी हाइ ोजन, को ह रत हाइ ोजन माना जाता है , य क इसके उ पादन के दौरान कोई काबन
डाई ऑ ससाइड का उ सजन नह ं होता है ।

4. रा य हाइ ोजन मशन, का ल य 2025 तक 25 म लयन टन ह रत हाइ ोजन का उ पादन


करना है ।

5. हाइ ोजन म भार के अनस


ु ार, कसी भी सामा य धन क ऊजा साम ी सबसे अ धक होती है ,
ले कन इसम मा ा के अनस
ु ार ऊजा क मा ा सबसे कम होती है ।

ऊपर दए गए कथन म से कौन-से सह ह?

(a) केवल 1, 3 और 4
(b) केवल 2, 3, और 5

(c) केवल 2, 4 और 5

(d) केवल 1, 2, 3 और 5

उ र: b

या या:

कथन 1 गलत है : नील ( ल)ू हाइ ोजन, धूसर ( )े हाइ ोजन के समान है , सवाय इसके क
अ धकांश काबन डाई ऑ ससाइड उ सजन, काबन का सं हण और का भंडारण उपयोग करके (जमीन
म सं ह त) कया जाता है । नील ( ल)ू हाइ ोजन, धूसर ( े) हाइ ोजन का एक व छकर वक प है ,
ले कन यह महं गा है , य क इसम काबन सं हण तकनीक का उपयोग कया जाता है।

कथन 2 और 3 सह ह: हाइ ोजन का उपयोग मु य प से पे ोरसायन और उवरक उ योग म कया


जाता है और इसका उ पादन बड़े पैमाने पर ाकृ तक गैस से होता है , िजससे भार मा ा म काबन
डाइऑ साइड का न कषण है । इसके न कषण क वध क कृ त के आधार पर, हाइ ोजन को
तीन े णय म वग कृत कया जाता है , अथात ् धूसर, नीला और ह रत। धूसर ( े) हाइ ोजन,
ाकृ तक गैस या कोयले जैसे जीवा म धन का उपयोग करके उ पा दत क जाती है । हाइ ोजन आज
व व म उ पा दत, कुल हाइ ोजन का लगभग 95% भाग है । दो मु य उ पादन व धयां भाप मीथेन
सध
ु ार और कोयला गैसीकरण ह। जब क, गल
ु ाबी हाइ ोजन परमाणु ऊजा वारा संचा लत
व युतपघटन के मा यम से उ प न हाइ ोजन को संद भत करता है । गल
ु ाबी हाइ ोजन को
सामा यतः ह रत हाइ ोजन माना जाता है , य क यह उ पादन के दौरान कोई काबन डाई
ऑ ससाइड उतसिजत नह ं करता है ।

कथन 4 गलत है : रा य हाइ ोजन मशन क घोषणा 2020-2021 के बजट भाषण के दौरान क
गई थी और इसक औपचा रक प से शु आत भारत के 75व वतं ता दवस पर धान मं ी वारा
क गई थी। मशन का उ दे य सरकार को अपने जलवायु ल य को परू ा करने और भारत को ह रत
हाइ ोजन क बनाने म सहायता करना है। इससे 2030 तक 50 लाख टन ह रत हाइ ोजन के
उ पादन के ल य को परू ा करने और अ य ऊजा मता के संबं धत वकास म सहायता मलेगी।

कथन 5 सह है : हाइ ोजन, ऊजा के एक मह वपण


ू ोत के प म उभर रहा है, य क इसम
काब नक पदथ क मा ा शू य होती है और यह हाइ ोकाबन के वपर त ऊजा का एक गैर-
दष
ू णकार ोत है , िजसम शु ध काबन साम ी 75 से 85 तशत क सीमा म है। हाइ ोजन,
बजल क तरह, एक ऊजा वाहक है , िजसे कसी अ य पदाथ से उ पा दत कया जाना चा हए।
हाइ ोजन का उ पादन - जल, जीवा म धन, या बायोमास स हत व भ न ोत से कया जा सकता
है और ऊजा या धन के ोत के प म इसका उपयोग कया जा सकता है। हाइ ोजन म भार के
अनस
ु ार, कसी भी सामा य धन क ऊजा साम ी सबसे अ धक होती है (गैसोल न से लगभग तीन
गन
ु ा अ धक), ले कन इसम मा ा के हसाब से ऊजा क मा ा सबसे कम होती है (गैसोल न से
लगभग चार गन
ु ा कम)।

ोत:https://static.pib.gov.in/WriteReadData/specificdocs/documents/2022/mar/doc202232127201.
pdfhttps://energyeducation.ca/encyclopedia/Types_of_hydrogen_fuel#cite_ref-hyd2_14-1

84. उ चतम यायालय के सलाहकार े ा धकार के संदभ म, न न ल खत कथन पर वचार


क िजए:

1. उ चतम यायालय को सावज नक मह व के कसी भी न पर रा प त को अपनी सलाह दे ना


अ नवाय होता है , ले कन कसी भी पूव-संवैधा नक सं ध से उ प न ववाद पर यह आव यक नह ं
है ।
2. अनु छे द 143 के तहत उ चतम यायालय क सलाह न तो या यक नणय है और न ह रा पत
पर बा यकार है।
उपयु त कथन म कौन सा/से सह है/ह?

(a) केवल 1
(b) केवल 2
(c) 1 और 2 दोन
(d) न तो 1, न ह 2
उ र : (b)

या या:

कथन 1 गलत है : अनु छे द 143 के तहत, रा प त को दो े णय म उ चतम यायालय क राय लेने


के लए अ धकृत करता है :

 सावज नक मह व के कसी मु दे पर व धक न पर या जो उठने क संभावना है।


 कसी भी पव
ू -संवैधा नक सं ध, समझौते, सनद या अ य समान ि थ तय से संबं धत कसी भी
ववाद पर।
पहले मामले म, उ चतम यायालय रा प त को अपनी राय दे ने से मना कर सकता है । ले कन, दस
ू रे
मामले म, उ चतम यायालय को रा प त को अपनी राय दे ना अ नवाय है ।
कथन 2 सह है: अनु छे द 143 के तहत उ चतम यायालय वारा रा प त को य त क गई राय
केवल सलाहकार है , या यक नणय नह ं। इस लए, उ चतम यायालय क सलाह या राय रा पत
के लए बा यकार नह ं है । रा प त उ चतम यायालय वारा द गई राय का पालन कर सकते ह या
नह ं भी।

ोत: एम. ल मीकांत, भारत क राज यव था, 6वां सं करण, अ याय- 26

85. भारत म सव च यायालय के मूल अ धका रता के संदभ म न न ल खत म से कौन-सा कथन


सह है ?

(a) उ चतम यायालय के पास अपील के मा यम से के और कसी भी रा य के बीच ववाद क


सुनवाई करने का अन य े ा धकार है ।
(b) सव च यायालय का मल
ू े ा धकार कसी भी पव
ू -संवैधा नक सं ध या समझौते से उ प न
ववाद तक व ता रत नह ं है ।
(c) सव च यायालय अपने मूल अ धकार े के तहत अंतर-रा यीय जल ववाद से संबं धत
मामल का फैसला कर सकता है ।
(d) अनु छे द 131 के तहत, उ चतम यायालय रा य एज सय वारा कसी भी भारतीय नाग रक के
मौ लक अ धकार के उ लंघन से संबं धत कसी भी मामले पर वचार कर सकता है ।
उ र : (b)

या या:

वक प (a) और (d) गलत ह: उ चतम यायालय के पास संघीय मामल पर सं वधान के अनु छे द 131
के तहत अन य े ा धकार है । यहां 'अन य' का अथ है क उ चतम यायालय के अलावा भारत म
कोई अ य यायालय ऐसे ववाद का फैसला नह ं कर सकता है और 'मल
ू ' का अथ है , ऐसे ववाद को
पहल बार म सन
ु ने क शि त, अपील के मा यम से नह ं। िजसम -

 क और एक या एक से अ धक रा य के बीच; या
 क और कोई रा य या रा य का एक तरफ होना और एक या एक से अ धक रा य का दस
ू र
तरफ होना; या
 दो या दो अ धक रा य के बीच।
इसके अलावा, उ चतम यायालय के अन य मूल े ा धकार के संबंध म, दो बंदओ
ु ं पर यान दया
जाना चा हए। सबसे पहले, ववाद म एक न (चाहे कानून या त य का) शा मल होना चा हए, िजस पर
व धक अ धकार न हत हो । राजनी तक कृ त के न को इससे बाहर रखा गया है । दस
ू रा, क या
रा य के खलाफ कसी नाग रक वारा उ चतम यायालय के सम लाए गए कसी भी मुकदमे पर
इसके तहत वचार नह ं कया जा सकता है ।

वक प (b) सह है ले कन वक प (c) गलत है : सं वधान के अनु छे द 131 के तहत उ चतम यायालय


का मूल े ा धकार, न न ल खत तक व ता रत नह ं है :

 कसी भी पव
ू -सं वधान सं ध, समझौते, सनद या इसी तरह के अ य साधन से उ प न ववाद।
 कसी सं ध, समझौते इ या द से बाहर उ प न होने वाला ववाद, जो वशेष प से यव था करता
हो क े ा धकार ऐसे ववाद तक सी मत नह ं है ।
 अंतररा यीय जल ववाद।
 व आयोग के संदभ वाले मामले ।
 क और रा य के बीच कुछ खच और पशन का समझौता ।
 क और रा य के बीच वा णि यक कृ त का सामा य ववाद।
 क के खलाफ रा य के कसी नुकसान क भरपाई।

न है स: वक प (a) म उि ल खत है "अपील के मा यम से" अपील वारा कुछ कभी भी अन य


नह ं हो सकता है इस लए वक प (a) को हटाया जा सकता है ।

ोत: ल मीकांत, भारत क राज यव था, 6वां सं करण, अ याय- 26

86. रा य या यक नयुि त आयोग (एनजेएसी) के संदभ म न न ल खत म से कौन-सा कथन सह


नह ं है ?

(a) इसे 99व सं वधान संशोधन अ ध नयम, 2014 के मा यम से सं वधान म शा मल कया गया
था।
(b) अनु छे द 124ए म एनजेएसी क संरचना के ावधान ह।
(c) क य कानन
ू और याय मं ी एनजेएसी के पदे न अ य ह।
(d) एनजेएसी के लए सं वधान संशोधन को सव च यायालय एडवोके स ऑन- रकॉड
एसो सएशन और अ य बनाम भारत संघ के मामले म सव च यायालय वारा र द कर दया
गया था।
उ र : (c)

या या:
वक प (a) सह है : रा य या यक नयुि त आयोग (एनजेएसी) को 2014 के 99 व सं वधान
संशोधन अ ध नयम के मा यम से सं वधान म शा मल कया गया था। आयोग का गठन कया गया
था:

a) भारत के मु य यायाधीश, उ चतम यायालय के यायाधीश , उ च यायालय के मु य


यायाधीश और उ च यायालय के अ य यायाधीश के प म नयुि त के लए यि तय
क सफा रश करना;
b) उ च यायालय के मु य यायाधीश और अ य यायाधीश को एक उ च यायालय से
कसी अ य उ च यायालय म थानांतरण क सफा रश करना; और
c) अनश
ु ं सत यि त क मता और अखंडता को सु नि चत करना है ।
वक प (b) सह है : एक नया अनु छे द, अनु छे द 124A, (जो एनजेएसी क संरचना के लए दान
करता है ) सं वधान म शा मल कया गया था।

वक प (c) सह नह ं है : 99 व सं वधान संशोधन अ ध नयम, 2014 के अनस


ु ार, एनजेएसी म
न न ल खत सद य शा मल होने चा हए:

 भारत के मु य यायाधीश (अ य , पदे न)


 भारत के मु य यायाधीश के बगल म सव च यायालय के दो अ य व र ठ यायाधीश –
पदे न
 क य कानून और याय मं ी, पदे न
 दो ति ठत यि त (भारत के मु य यायाधीश, भारत के धानमं ी और लोकसभा म वप
के नेता वाल स म त वारा ना मत या जहां वप का ऐसा कोई नेता नह ं है , तो, लोकसभा म
सबसे बड़े वप ी दल के नेता), बशत क दो ति ठत यि त, एक यि त अनस
ु ू चत जा त
या अनुसू चत जनजा त या ओबीसी या अ पसं यक समुदाय या एक म हला से होगा।
यात यि तय को तीन वष क अव ध के लए ना मत कया जाएगा और वे पन
ु : नामांकन
के लए पा नह ं ह गे।
वक प (d) सह है : एनजेएसी के लये 99व संवैधा नक संशोधन अ ध नयम को सव च यायालय
एडवोके स ऑन- रकॉड एसो सएशन और अ य बनाम भारत संघ के मामले म सव च यायालय
वारा दनांक 16-10-2015 के अपने फैसले म र द कर दया गया है ।

ोत: एम ल मीकांत : भारत क राज यव था, 6वां सं करण, अ याय -26।


87. अनु छे द 137 के तहत समी ा करने क सव च यायालय क शि त के बारे म न न ल खत म
से कौन-सा कथन सह है ?

(a) यह वधायी अ ध नयम और क और रा य सरकार के कायकार आदे श क संवैधा नकता


क जांच कर सकता है ।
(b) वह अपने फैसल और आदे श क समी ा कर सकता है ।
(c) यह अपील के मा यम से उ च यायालय वारा पा रत सभी नणय और आदे श क समी ा
कर सकता है ।
(d) यह उन सभी अंतररा य कानन
ू और सं धय क समी ा कर सकता है िजन पर भारत ने
ह ता र कए ह।
उ र : (b)

या या:

वक प (a) गलत है : या यक समी ा क और रा य दोन सरकार के वधायी अ ध नयम और


कायकार आदे श क संवैधा नकता क जांच करने के लए सव च यायालय क शि त है। जांच करने
पर, य द वे सं वधान का उ लंघन (अ धकार े से बाहर) पाए जाते ह, तो उ ह सव च यायालय
वारा अवैध, असंवैधा नक और अमा य (अमा य) घो षत कया जा सकता है । नतीजतन, सरकार
वारा उ ह लागू नह ं कया जा सकता है । सव च यायालय क यह शि त अनु छे द 13 और 32 से
आती है ।

वक प (b) सह है : भारत के सं वधान का अनु छे द 137 सव च यायालय को अपने कसी भी नणय


या आदे श क समी ा करने क शि त दान करता है। हालां क यह शि त अनु छे द 145 के तहत
सव च यायालय वारा बनाए गए नयम के साथ-साथ संसद वारा अ ध नय मत कसी भी कानून
के ावधान के अधीन है । इसके अलावा, सव च यायालय नयम, 2013 के अनुसार, फैसले के 30
दन के भीतर पुन वचार या चका दायर क जानी चा हए।

वक प (c) गलत है: भारत म सभी यायालय और अ धकरण पर उ चतम यायालय का यापक
अपील य े ा धकार भी है । अपील य े ा धकार के तहत, सव च यायालय अपील के मा यम से
उ च यायालय वारा दए गए मामल क सन
ु वाई कर सकता है । सं वधान म अनु छे द 132, 133,
134 और 134ए के तहत इसका ावधान कया गया है ।
वह अपने ववेकानुसार सं वधान के अनु छे द 136 के अधीन भारत के रा य े म कसी यायालय या
अ धकरण वारा पा रत या कए गए कसी कारण या वषय म कसी नणय, वा य या आदे श से
अपील करने क वशेष अनम
ु त भी दे सकेगा।

वक प (d) गलत है भारतीय यायपा लका को अंतरा य कानून के मु द से संबं धत घरे लू मामल
का नणय करके अंतरा य कानून म भारत के दा य व क या या करने क शि त है। यह कथन
गलत है य क कसी भी अंतरा य सं धय क समी ा करने म यायपा लका क कोई भू मका नह ं
है , हालां क भारत के रा प त कसी भी सं ध या समझौते पर उ प न होने वाले कसी भी ववाद पर
सव च यायालय क राय ले सकते ह।

ोत: एम ल मीकांत, भारत क राज यव था, 6वां सं करण, अ याय -26।

88. भारत के सं वधान क कस अनुसूची के तहत खनन के लए नजी पा टय को आ दवासी भू म के


ह तांतरण को शू य और शू य घो षत कया जा सकता है ?

)A) तीसर अनुसूची

)B) पांचवीं अनुसूची

)C) नौवीं अनस


ु च
ू ी

)D) बारहवीं अनुसूची

उ र: B

या या:

वक प (B) सह है : भारत के सं वधान क 5 वीं अनुसच


ू ी के तहत खनन के लए नजी पा टय को
आ दवासी भू म के ह तांतरण को शू य और शू य घो षत कया जा सकता है । वतमान म, 10 रा य
आं दे श, छ ीसगढ़, गुजरात, हमाचल दे श, झारखंड, म य दे श, महारा , ओ डशा, राज थान
और तेलंगाना म पांचवीं अनुसूची े ह। सं वधान क पांचवीं अनुसूची अनुसू चत े के साथ-साथ
असम, मेघालय, परु ा और मजोरम रा य के अलावा कसी भी रा य म रहने वाले अनस
ु ू चत
जनजा तय के शासन और नयं ण से संबं धत है। एससी नणय: समथा बनाम आं दे श रा य
और अ य (1997) खनन या अ य संबं धत ग त व धय के लए नजी पा टय को आ दवासी भू म का
ह तांतरण पांचवीं अनुसूची के तहत शू य और शू य घो षत कया जा सकता है।

ोत: यूपीएससी सीएसई पीवाई यू 2019

89. भारत म सव च यायालय के संदभ म, न न ल खत कथन पर वचार क िजए:

1. उ चतम यायालय अपने ववेक पर सै य अदालत को छोड़कर कसी भी अदालत वारा दए गए


कसी भी फैसले के खलाफ अपील करने क वशेष अनम
ु त दे सकता है।
2. सं वधान के अनु छे द 21 के अंतगत सव च यायालय को दोषी वारा दायर येक उपचारा मक
या चका पर सन
ु वाई करना आव यक है ।
3. भारत म सव च यायालय अपने पछले नणय से बा य नह ं है और उसके नणय सामद
ु ा यक
क याण के हत म इससे अलग हो सकता है ।
उपयु त कथन म से कौन-सा/से सह है/ह?

(a) केवल 1 और 2
(b) केवल 1 और 3
(c) केवल 3
(d) केवल 2 और 3
उ र : (b)

या या:

कथन 1 सह है: अनु छे द 136 के तहत अपनी शि त का योग करते समय, उ चतम यायालय दे श
म कसी भी अदालत या यन
ू ल (सै य यन
ू ल और कोट माशल को छोड़कर) वारा पा रत कसी
भी मामले म कसी भी फैसले से अपील करने के लए अपने ववेक से वशेष अनम
ु त दे ने के लए
अ धकृत है ।

कथन 2 गलत है : अं तम दोष स ध के खलाफ पन


ु वचार या चका खा रज होने के बाद सध
ु ारा मक
या चका दायर क जा सकती है । यह सु नि चत करने के लए है क याय म वफलता न हो, और
या के द ु पयोग को रोका जा सके। येक सुधारा मक या चका पर उ चतम यायालय वारा
पा अशोक हुरा बनाम अशोक हुरा और अ य, 2002 म नधा रत स धांत के आधार पर नणय लया
जाता है । इस मामले म उ चतम यायालय ने फैसला सन
ु ाया है क यरू े टव पट शन पर वचार कया
जा सकता है य द या चकाकता यह स ध करता है क ाकृ तक याय के स धांत का उ लंघन हुआ
था, और आदे श पा रत करने से पहले अदालत वारा उसक बात नह ं सुनी गई थी। यह भी वीकार
कया जाएगा जहां एक यायाधीश उन त य का खल
ु ासा करने म वफल रहा जो पूवा ह क आशंका
को बढ़ाते ह। इसके अलावा, उ चतम यायालय ने माना है क यरू े टव या चकाएं नय मत होने के
बजाय दल
ु भ होनी चा हए, और सावधानी के साथ वचार कया जाना चा हए। एक यरू े टव पट शन
का एक व र ठ वक ल वारा माणन होना चा हए, जो इस पर वचार करने के लए पया त आधार
बताता है। य द उपल ध हो तो इसे पहले तीन व र ठतम यायाधीश क पीठ और संबं धत नणय
पा रत करने वाले यायाधीश क पीठ को प रचा लत कया जाना चा हए। जब अ धकांश यायाधीश
यह न कष नकालते ह क मामले क सुनवाई क आव यकता है , तभी इसे उसी पीठ के सम
यथासंभव सच
ू ीब ध कया जाना चा हए। इसके अलावा, भारतीय सं वधान के कसी भी अनु छे द के
तहत ' यूरे टव पट शन' श द का उ लेख नह ं है ।

कथन 3 सह है: सं वधान के अनु छे द 137 के तहत, उ चतम यायालय को अपने वयं के फैसले या
आदे श क समी ा करने क शि त है । इस कार, उ चतम यायालय अपने पछले फैसले से बा य
नह ं है और याय या सामुदा यक क याण के हत म इससे अलग नणय ले सकता है । सं ेप म,
उ चतम यायालय एक आ म-सुधार एजसी है । उदाहरण के लए, केशवानंद भारती मामले (1973) म,
उ चतम यायालय गोलक नाथ मामले (1967) म अपने पछले फैसले से अलग था।

ोत:

ल मीकांत भारत क राज यव था, 6वां सं करण, अ याय- 26;

https://indianexpress.com/article/what-is/what-is-curative-petition-december-2012-
gangrape-case-sc-6204629/

90. न न ल खत म से कौन-सा/से कथन सह है /ह?

1. ाम यायालय और लोक अदालत दोन को ाम तर पर कसी भी आपरा धक और द वानी


मामल से नपटने का अ धकार है ।
2. लोक अदालत वारा पा रत नणय अं तम और प कार पर बा यकार है , जब क ाम यायालय
वारा पा रत आदे श को एक ड माना जाता है और कृ त म अं तम नह ं माना जाता है ।
नीचे दए गए कूट का योग करके सह उ र का चयन क िजये:
(a) केवल 1
(b) केवल 2
(c) 1 और 2 दोन
(d) न तो 1, न ह 2

उ र : (b)

या या:

कथन 1 गलत है ाम यायालय अ ध नयम, 2008 के अनुसार, ाम यायालय एक मोबाइल


यायालय होगा और आपरा धक और द वानी यायालय दोन क शि तय का योग करे गा। ाम
यायालय आपरा धक मामल , द वानी मक
ु दम , दाव या ववाद पर वचार करे गा जो अ ध नयम क
पहल अनुसूची और दस
ू र अनुसूची म न द ट ह, न क सभी मामले। जब क, लोक अदालत न केवल
अदालत के सम लं बत मामल से नपट सकती है , बि क उन आपरा धक मामल स हत पूव-
मुकदमेबाजी चरण म ववाद से भी नपट सकती है जो केवल शमनीय अपराध (compoundable
offences) ह। दस
ू रे श द म, जो अपराध कसी भी कानन
ू के तहत गैर- शमनीय अपराध ह, वे लोक
अदालत के दायरे से बाहर आते ह।

कथन 2 सह है: लोक अदालत क सं था को व धक सेवा ा धकरण अ ध नयम, 1987 के तहत


वैधा नक दजा दया गया है । लोक अदालत का अ ध नणय स वल यायालय क ड या कसी अ य
यायालय का आदे श माना जाएगा। लोक अदालत वारा दया गया येक नणय ववाद के सभी प
के लए अं तम और बा यकार होगा। लोक अदालत के फैसले के खलाफ कसी भी अदालत म कोई
अपील नह ं क जाएगी। ाम यायालय अ ध नयम 2008 म यह अ धदे श दया गया है क ाम
यायालय को प कार के बीच सुलह तक लाकर जहां तक संभव हो ववाद को सल
ु झाने का यास
करना चा हए और इस योजन के लए, नयु त कए जाने वाले सल
ु हकताओं का उपयोग करे गा। ाम
यायालय वारा पा रत नणय और आदे श को एक ड माना जाएगा और इसके न पादन म दे र से
बचने के लए, ाम यायालय व रत यास करे गा। आपरा धक मामल म अपील स यायालय म
होगी, िजसे ऐसी अपील दायर करने क तार ख से छह मह ने क अव ध के भीतर सन
ु ा और नपटाया
जाएगा। द वानी मामल म अपील िजला यायालय म होगी, िजसे अपील दायर करने क तार ख से छह
मह ने क अव ध के भीतर सन
ु ा और नपटाया जाएगा।
ोत: ल मीकांत, भारत क राज यव था, 6वां सं करण, अ याय- 36

91. भारत के सव च यायालय के स दभ म, न न ल खत कथन पर वचार क िजए:

1. सं वधान रा प त को द ल के अलावा अ य थान को सव च यायालय क पीठ के प म


नयु त करने के लए अ धकृत करता है ।
2. अनु छे द 143 के तहत संवध
ै ा नक मामल या रा प त वारा कए गए संदभ का नणय कम
से कम पांच यायाधीश वाल पीठ वारा कया जाता है ।
3. सव च यायालय म सभी कायवाह अं ेजी म होनी चा हए, ले कन या चकाओं को अं ेजी के
साथ हंद म अनुम त द जा सकती है ।
उपयु त कथन म कौन-सा/से कथन सह है/ह?

(a) केवल 1 और 2
(b) केवल 2
(c) केवल 2 और 3
(d) केवल 1 और 3
उ र : (b)

या या:

कथन 1 गलत है : सं वधान द ल को सव च यायालय क पीठ के प म घो षत करता है । सं वधान


भारत के मु य यायाधीश को सव च यायालय क पीठ के प म द ल के अलावा अ य थान को
नयु त करने के लए अ धकृत करता है , न क रा प त को । ले कन वह रा प त के अनम
ु ोदन से ह
इस संबंध म नणय ले सकता है ।

कथन 2 सह है: अनु छे द 143 के तहत रा प त वारा कए गए संवैधा नक मामल या संदभ का


नणय कम से कम पांच यायाधीश वाल पीठ वारा कया जाता है । अ य सभी मामल का फैसला
एकल पीठ और खंडपीठ वारा कया जाता है । ये फैसले खल
ु अदालत वारा सुनाए जाते ह। सभी
नणय बहुमत से होते ह, ले कन य द अलग-अलग होते ह, तो यायाधीश असहम तपूण नणय या राय
दे सकते ह।

कथन 3 गलत है : जब तक संसद अ यथा दान नह ं करती है , तब तक सव च यायालय क सभी


कायवाह अं ेजी म होनी चा हए। संसद ने हंद के उपयोग के लए कोई ावधान नह ं कया है ,
इस लए सव च यायालय केवल उ ह ं या चकाओं या अपील क अनुम त दे ता है जो केवल अं ेजी म
ह।

ोत: एम ल मीकांत, भारत क राज यव था, 6वां सं करण, अ याय -26।

92. उ च यायालय के यायाधीश के सदभ म न न ल खत कथन पर वचार क िजए:

1. वे रा यपाल वारा नयु त कए जाते ह ले कन रा प त के सादपयत ह।


2. उ ह उसी र त से और उसी आधार पर हटाया जा सकता है जैसे सव च यायालय के
यायाधीश।
3. एक ति ठत याय वद को सव च यायालय के यायाधीश के प म नयु त कया जा
सकता है ले कन उ च यायालय के यायाधीश के प म नह ं।
उपयु त म कौन-सा/से कथन सह है/ह?

(a) केवल 1 और 2
(b) केवल 2 और 3
(c) केवल 3
(d) 1, 2 और 3
उ र : (b)

या या:

कथन 1 गलत है : उ च यायालय के यायाधीश क नयुि त रा प त वारा क जाती है । भारत के


मु य यायाधीश और संबं धत रा य के रा यपाल के परामश के बाद रा प त वारा मु य यायाधीश क
नयिु त क जाती है । अ य यायाधीश क नयिु त के लए संबं धत उ च यायालय के मु य
यायाधीश से भी परामश लया जाता है ।

इ ह कायकाल क सुर ा ा त है (हालां क सं वधान म कोई नि चत कायकाल दान नह ं कया गया है )


और 62 वष क आयु तक पद धारण करते ह।

कथन 2 सह है: कसी उ च यायालय के यायाधीश को रा प त के आदे श वारा उसके पद से हटाया जा


सकता है । रा प त तभी हटाने का आदे श जार कर सकते ह जब संसद के उसी स म हटाने के लए ताव
पेश कया गया हो। ताव को संसद के येक सदन के वशेष बहुमत वारा सम थत कया जाना चा हए
(अथात ्, उस सदन क कुल सद यता का बहुमत और उस सदन के उपि थत और मतदान करने वाले
सद य के कम से कम दो- तहाई बहुमत)। हटाने के दो आधार द ु यवहार या अ मता ह। इस कार, एक
उ च यायालय के यायाधीश को उसी र त और उसी आधार पर हटाया जा सकता है जैसे क सव च
यायालय के यायाधीश को।

कथन 3 सह है: उ च यायालय के यायाधीश के प म नयु त कए जाने वाले यि त के पास


न न ल खत यो यताएं होनी चा हए:

 उसे भारत का नाग रक होना चा हए।


 उसे दस वष तक भारत के रा य े म या यक पद धारण करना चा हए; या
 उ ह दस साल तक उ च यायालय (या उ रा धकार म उ च यायालय ) का वक ल होना
चा हए।
सं वधान म कसी ति ठत याय वद क उ च यायालय के यायाधीश के प म नयिु त का कोई
ावधान नह ं है । ले कन एक ति ठत याय वद सव च यायालय का यायाधीश बन सकता है ।

ोत: एम ल मीकांत, भारत क राज यव था, 6वां सं करण, अ याय -34।

93. वधान प रषद के संदभ म न न ल खत कथन पर वचार क िजए:

1. वधान प रषद के सद य आं शक प से मनोनीत और आं शक प से अ य प से चन


ु े
जाते ह।
2. वधान प रषद क अ धकतम सं या वधानसभा क कुल सं या का एक तहाई और यूनतम
सं या 40 नधा रत क गई है ।
3. 1/12 सद य को श क और नातक वारा तीन साल के लये चन
ु ा जाता है ।
उपयु त म कौन-सा/से कथन सह है/ह?

(a) केवल 1 और 2
(b) केवल 2 और 3
(c) केवल 3
(d) 1, 2 और 3
उ र : (d)

या या:
कथन 2 सह है: वधान सभा के सद य के वपर त, वधान प रषद के सद य अ य प से चन
ु े जाते
ह। वधान प रषद क अ धकतम सं या वधानसभा क कुल सं या का एक तहाई और यूनतम सं या
40 नधा रत क गई है। इसका मतलब है क प रषद का आकार संबं धत रा य क वधानसभा के आकार
पर नभर करता है ।

यह रा य के वधायी मामल म सीधे नवा चत सदन ( वधानसभा) क वर यता सु नि चत करने के लए


कया जाता है । य य प सं वधान ने अ धकतम और यूनतम सीमा नधा रत क है , ले कन प रषद क
वा त वक सं या संसद वारा नधा रत क जाती है ।

कथन 1 और 3 सह ह: चन
ु ाव का तर का: वधान प रषद के सद य क कुल सं या म से:

1. 1/3 सद य रा य म थानीय नकाय जैसे नगर पा लकाओं, िजला बोड आ द के सद य


वारा चन
ु े जाते ह,
2. 1/12 सद य रा य म रह रहे तीन वष से नातक वारा चन
ु े जाते ह,
3. 1/12 सद य रा य म रह रहे तीन वष से अ यापन कर रहे अ यापक चन
ु ते ह, ये मा य मक
व यालय से कम के नह ं होने चा हए,
4. 1/3 रा य क वधान सभा के सद य वारा उन यि तय म से चन
ु े जाते ह जो वधानसभा के
सद य नह ं ह, और
5. शेष को रा यपाल वारा उन यि तय म से ना मत कया जाता है िजनके पास सा ह य,
व ान, कला, सहकार आंदोलन और समाज सेवा का वशेष ान या यावहा रक अनभ
ु व है ।
इस कार, वधान प रषद के सद य क कुल सं या का 5/6 अ य प से नवा चत होता है और 1/6
रा यपाल वारा ना मत कया जाता है। सद य को एकल ह तांतरणीय वोट के मा यम से आनुपा तक
त न ध वक णाल के अनुसार चन
ु ा जाता है। कसी भी मामले म रा यपाल के मनोनयन क
ामा णकता या औ च य को अदालत म चन
ु ौती नह ं द जा सकती है ।

ोत: एम ल मीकांत, भारत क राज यव था, 6वां सं करण, अ याय -33।

94. पूण याय करने के लए आदे श या ड पा रत करने क सव च यायालय क अंत न हत शि त


इनम से कस अनु छे द से मलती है ?

(a) अनु छे द 136


(b) अनु छे द 137
(c) अनु छे द 142
(d) अनु छे द 143
उ र : (c)

या या:

वक प (c) सह है : अनु छे द 142 के मा यम से भारत का सं वधान भारत के सव च यायालय को इस


तरह के " ड या आदे श को पा रत करने के लए सश त बनाता है जो पण
ू याय करने के लए आव यक
हो सकता है "। हमारे सं वधान के अनु छे द-142 म ावधान है क पूण याय करने क या म उ चतम
यायालय के पास आव यकतानुसार कोई भी नणय, ड या आदे श पा रत करने क शि त है । सव च
यायालय वारा दया गया ऐसा कोई भी आदे श भारत के पूरे े म लागू होगा। ऐसे आदे श के वतन का
साधन उस कानून वारा नधा रत कया जाएगा जो इसके बारे म संसद वारा पा रत कया गया है । य द
सव च यायालय वारा पा रत आदे श को लागू करने के लए संसद वारा पा रत कोई मौजद
ू ा कानन
ू नह ं
है , तो रा प त का आदे श इसे नधा रत करे गा। इस अनु छे द म, सव च यायालय कसी यि त क
उपि थ त के लए आदे श भी दे सकता है , द तावेज क खोज और तु त के लए कह सकता है , और
इसक अवमानना के लए जांच और दं डत भी कर सकता है ।

ये शि तयाँ सव च यायालय को द गई ह ता क सव च यायालय अ य यायालय को कसी भी कार


क े ा धकार संबंधी क ठनाइय से बचा सके और याय को सु नि चत करने के लये यायालय को हा न
न होने दे ।

ोत:

https://blog.ipleaders.in/inherent-powers-supreme-court-high-court-civil-court-criminal-court/

एम ल मीकांत भारत क राज यव था, छठा सं करण, अ याय -26

95. भारत हर साल 15 म लयन टन लाि टक अप श ट उ प न करता है , ले कन इसम से केवल एक


चौथाई ह अ छ ठोस अप श ट बंधन णाल क कमी के कारण पन
ु नवीनीकरण कया जाता है।
इससे लड फल और कमजोर सामािजक-आ थक प रि थ तय पर बोझ होता है।

लाि टक अप श ट बंधन के संदभ म, न न ल खत कथन पर वचार कर:


1. लाि टक अप श ट बंधन नयम , 2016 के तहत, यह ाथ मक िज मेदार नमाता, आयातक
और ांड मा लक पर है क जो बाजार म उनके वारा पेश कए गए बहु तर य लाि टक पाउच या
पैकेिजंग जो उपयोग म लाये गए है को सं हत करे ।
2. व ता रत नमाता िज मेदार केवल उपभो ता लाि टक पैकेिजंग कचरे पर लागू होती है ।
3. ईपीआर दशा नदश 2022 के अनस
ु ार व ता रत नमाता िज मेदार दशा नदश अ धशेष
व ता रत नमाता िज मेदार माणप क ब और खर द के लए अनम
ु त दे ते ह।

ऊपर दए गए कथन म से कौन-से सह ह?


(a) केवल 1 और 2
(b) केवल 2 और 3
(c) 1 और 3 केवल
(d) 1, 2 और 3
उ र: C
या या:
कथन 1 सह है: लाि टक अप श ट बंधन नयम, 2016, लाि टक कचरे के जनरे टर को जनादे श
लाि टक कचरे क पीढ़ को कम करने के लए कदम उठाने के लए, लाि टक कचरे को कूड़े के
लए नह ं, ोत पर कचरे के अलग -अलग भंडारण को सु नि चत करने के लए और अलग -अलग
कचरे को अलग कर दया नयम के अनुसार। नयम लाि टक कचरे का बंधन करने के लए
थानीय नकाय , ाम पंचायत , अप श ट जनरे टर, खुदरा व े ताओं और सड़क व े ताओं क
िज मेदा रय को भी अ नवाय करते ह। लाि टक अप श ट बंधन नयम, 2016 ने नमाता, आयातक
और ांड मा लक पर उ पादक िज मेदार को बढ़ाया। उपयोग कए गए बहु तर य लाि टक पाउच
या पाउच या पैकेिजंग के सं ह के लए ाथ मक िज मेदार उ पादक , आयातक और ांड मा लक
क है जो बाजार म उ पाद का प रचय दे ते ह।उ ह अपने उ पाद के कारण उ प न लाि टक कचरे
को वापस इक ठा करने के लए एक णाल था पत करने क आव यकता है।
कथन 2 गलत है : व ता रत उ पादक क िज मेदार (EPR) शासन लाि टक अप श ट बंधन
नयम , 2016 म काया वयन के तहत है , 2016 के नयम के तहत, व ता रत नमाता िज मेदार पव
ू -
उपभो ता और बाद के उपभो ता लाि टक पैकेिजंग कचरे दोन पर लागू होगी।
कथन 3 सह है : ईपीआर, पयावरण, वन और जलवायु प रवतन मं ालय, भारत सरकार के काया वयन
या को कारगर बनाने के लए, लाि टक अप श ट बंधन नयम म अपने चौथे संशोधन म,
2022, व ता रत नमाता के लए दशा नदश को सू चत कया। नयम के अनस
ु च
ू ी II म लाि टक
पैकेिजंग '। इन दशा नदश ने पहल बार, अ धशेष व ता रत नमाता िज मेदार माण प क
पीढ़ का ताव दया। वे पछले वष व ता रत नमाता िज मेदार ल य और दा य व के खलाफ
आगे ले जाने और ऑफसेट करने के लए अ धशेष व ता रत नमाता िज मेदार माणप क ब
और खर द क अनम
ु त दे ते ह।
ोत: https://eprplastic.cpcb.gov.in/#/plastic/home

96. डग ग के संदभ म, न न ल खत कथन पर वचार कर:


1. वे मु य प से सी ास बेड पर संप न द ु नया म सबसे बड़े शाकाहार समु तनधार ह।
2. वे भारतीय और पि चमी शांत महासागर के गम अ ांश म पाए जाते ह।
3. वे वाइ ड लाइफ ( ोटे शन) अ ध नयम, 1972 के अनस
ु च
ू ी 1 के तहत संर त ह।
4. भारत का पहला डग ग संर ण रजव च लका झील के पास ओ डशा म था पत कया गया
है ।

ऊपर दए गए कथन म से कौन-सा/से सह है/ह?


(a) केवल 1, 2 और 3
(b) 1, 2 और 4 केवल
(c) 1, 3 और 4 केवल
(d) 2, 3 और 4 केवल

उ र: a

या या:
टे टमट 1 सह है : डग स द ु नया म सबसे बड़े शाकाहार समु तनधार ह जो मु य प से
सी ास बेड पर संप न होते ह, जो महासागर का एक मख
ु काबन संक है । दग
ु ग स ती से एक
समु तनपायी है । आमतौर पर 'समु गाय ' के प म जाना जाता है । यह मु य प से समु
घास पर चराई करता है और इस लए अपना अ धकांश समय समु घास के ब तर म बताता है।
तट य वकास या औ यो गक ग त व धय के कारण डग ग को समु घास के नवास थान के
नुकसान या गरावट से खतरा है जो जल दष
ू ण का कारण बनता है । य द खाने के लए पया त
समु घास नह ं है तो डग ग सामा य प से जनन नह ं करता है । यह उनके उथले पानी के समु
आवास के संर ण को बहुत मह वपण
ू बनाता है।
कथन 2 सह है : डग ग समु गाय क एक जा त है जो भारतीय और पि चमी शांत महासागर के
गम अ ांश म पाई जाती है । यह डग गडे प रवार का एकमा सद य है , और इसके नकटतम
जी वत र तेदार मैने स ह। भारतीय और पि चमी शांत महासागर के उथले तट य जल म समु
घास पर डग स चरते ह।
कथन 3 सह है : नवास थान के कारण डग ग क आबाद गरावट पर है। इसक सरु ा क ि थ त
है :
o वाइ ड लाइफ (संर ण) अ ध नयम, 1972 - अनस
ु च
ू ी 1
o iucn लाल सच
ू ी - कमजोर
o CITES - प र श ट 1

कथन 4 गलत है : भारत के पहले 'दग


ु ग संर ण रजव' को पॉक बे (त मलनाडु के तट से दरू ) म
सू चत कया गया था। इस कदम का उ दे य डग ग जा तय और उसके समु आवास क र ा
करना था। डग ग को संर त करने से सी ास बेड क र ा और सध
ु ार करने म मदद मलेगी और
अ धक वायम
ु ड
ं ल य काबन को अनु मत कया जाएगा। सी ास बेड भी कई यावसा यक प से
मू यवान मछ लय और समु जीव के लए जनन और खला मैदान ह। इस लए, हजार फशर
प रवार सीधे अपनी आय के लए डग ग आवास पर नभर करते ह।
ोत:

https://www.worldwildlife.org/species/dugong
https://indianexpress.com/article/cities/chennai/indias-first-dugong-conservation-reserve-notified-
in-tamil-nadu-says-govt-8165098/

97. 'सीबेड 2030 ोजे ट' के संदभ म, न न ल खत म से कौन सा कथन सह है ?


1. प रयोजना को 2017 म संयु त रा महासागर स मेलन म लॉ च कया गया था।
2. इस प रयोजना के तहत, यूने को ने 2030 तक सीबेड के 100% को मैप करने का ल य
नधा रत कया है ।
3. यह प रयोजना संयु त रा वारा महासागर दशक क पहल का एक मख
ु काय म है।

4. यह प रयोजना संयु त रा सतत वकास ल य 17 के साथ संरे खत है , िजसका उ दे य


काया वयन के साधन को मजबत
ू करना है और सतत वकास के लए वैि वक साझेदार को
पन
ु ज वत करना है ।
नीचे दए गए कूट का योग कर सह उ र चु नए:

(a) 1 और 3 केवल
(b) 1, 2 और 4 केवल
(c) 4 केवल
(d) केवल 2 और 3
उ र: a

या या:

कथन 1 सह है : सीबेड 2030 जापान के न पॉन फाउं डेशन और महासागर के सामा य बा थमे क
चाट (GEBCO) के बीच एक सहयोगी प रयोजना है। इसका उ दे य 2030 तक व व महासागर के फश
के नि चत न शे का उ पादन करने के लए सभी उपल ध बा थमे क डेटा को एक साथ लाना है
और इसे सभी के लए उपल ध कराना है । यह प रयोजना जून 2017 म संयु त रा (UN) महासागर
स मेलन म शु क गई थी।
कथन 2 गलत है: जब सीबेड 2030 को पांच साल पहले लॉ च कया गया था, तो केवल 6% सीबेड
को आधु नक मानक पर मैप कया गया था, आज यह आंकड़ा 20% है। स मेलन म, यन
ू े को ने
घोषणा क है क अ य संयु त रा नकाय के सहयोग से और इसके सद य रा य और नजी े
के समथन के साथ, वतमान म 20% क तल
ु ना म कम से कम 80% सीबेड को 2030 तक मैप कया
जाएगा। सीबेड मै पंग म तेजी लाने क मता हाल के तकनीक नवाचार वारा और बढ़ाई गई है ।
जब क सोनार तकनीक का उपयोग 1960 के दशक के बाद से सीबेड को कैन करने के लए कया
गया है , सोनास के पास अब कई बीम ह जो उ ह कई बंदओ
ु ं पर पानी क ऊंचाई को मापने म
स म बनाते ह और एक ह बार म कई दशाओं म, जो समय को काफ बचाते ह।
कथन 3 सह है : सीबेड 2030 महासागर के दशक का एक मख
ु काय म है, संयु त रा वारा
सरकार , नजी े , वै ा नक और नाग रक समाज को सह- डजाइन और सह- डल वर प रवतनकार
ान के नेत ृ व वाले काय को जट
ु ाने के लए एक मख
ु पहल रवस करने के लए रवस महासागर
के वा य म गरावट और समु वातावरण के थायी बंधन म बदलाव। IOC-UNESCO संयु त
रा का नकाय है जो महासागर के दशक के काया वयन का नेत ृ व करता है
कथन 4 गलत है: सीबेड डेटा यह नधा रत करने के लए मल
ू भत
ू है क महासागर कैसे काम करता
है । ने वगेशन से परे , समु तल का आकार समु के मलबे और इसक सतह और धाराओं पर दष
ू ण
के आंदोलन म एक बड़ी भू मका नभाता है । गहराई के बारे म ान थायी म य बंधन म
अंत ि ट दान कर सकता है । महासागर अ ल करण भी सीधे गहराई से जड़
ु ा हुआ है; कुछ े
अ धक रासाय नक प रवतन का अनभ
ु व कर सकते ह और दस
ू र क तल
ु ना म व थ पा रि थ तक
तं को बनाए रखने म कम स म हो सकते ह। इस कार, सीबेड 2030 को संयु त रा के सतत
वकास ल य 14 के साथ जोड़ा जाता है , िजसका उ दे य महासागर , समु और समु संसाधन का
संर ण और नरं तर उपयोग करना है ।
ोत:
https://seabed2030.org/aboutus#:~:text=Seabed%202030%20is%20a%20collaborative,make%20it%
20available%20to%20all

98. ' वा य और प रवार क याण मं ालय, भारत सरकार ने दे श म डिजटल वा य बु नयाद ढांचे
के एक करण के लए आव यक सहायता दान करने के उ दे य से आयु मान भारत डिजटल मशन
तैयार कया है ।' नीचे दए गए कथन म से कौन सा है / आयु मान भारत डिजटल मशन के संदभ
म सह है ?
1. आयु मान भारत डिजटल मशन नेशनल हे थ पॉ लसी, 2017 से उपजा है ।
2. एक यि तगत वा य रकॉड एक इले ॉ नक अनु योग है जो रो गय को उनक वा य
जानकार को बनाए रखने और बं धत करने म मदद करता है ।
3. एक कुशल, सल
ु भ, समावेशी, स ती, समय पर और सरु त तर के से सावभौ मक वा य कवरे ज
आयु मान भारत डिजटल मशन क अं तम ि ट है ।

ऊपर दए गए कथन म से कौन-सा/से सह है/ह?


(a) 1 और 2 केवल
(b) 2 और 3 केव
(c) केवल 1 और 3
(d) 1, 2 और 3

उ र: d
या या:

कथन 1 सह है : वा य और प रवार क याण मं ालय, भारत सरकार ने दे श म डिजटल वा य


बु नयाद ढांचे के एक करण के लए आव यक सहायता दान करने के उ दे य से आयु मान भारत
डिजटल मशन तैयार कया है । यह दरू दश पहल, रा य वा य नी त, 2017 से उपजी, भारत म
वा य सेवा को डिजटाइज़ करने का इरादा रखता है । रा य वा य ा धकरण (NHA) को
डजाइ नंग रणनी त, तकनीक बु नयाद ढांचे का नमाण और आयु मान भारत डिजटल मशन के
काया वयन क भू मका के साथ स पा गया है।
कथन 2 सह है : मशन के तहत, एक यि तगत वा य रकॉड एक इले ॉ नक एि लकेशन है
िजसके मा यम से मर ज एक नजी, सरु त और गोपनीय वातावरण म अपनी वा य जानकार
(और दस
ू र के लए अ धकृत ह) को बनाए रख सकते ह और बं धत कर सकते ह। नाग रक
वा य आईडी को दे खे गए वा य दे खभाल सु वधाओं के साथ वा य आईडी बनाकर यि तगत
वा य रकॉड का अनुदै य ि टकोण ा त कर सकते ह।
कथन 3 सह है: इसक ि ट एक रा य डिजटल वा य पा रि थ तक तं बनाना है जो एक
कुशल, सल
ु भ, समावेशी, स ती, समय पर और सरु त तर के से सावभौ मक वा य कवरे ज का
समथन करता है । यह डेटा, सच
ू ना और बु नयाद ढांचे क सेवाओं क एक व तत
ृ ंख
ृ ला के
ावधान के मा यम से एक सहज ऑनलाइन लेटफॉम को भी लागू करता है , जो क खल
ु ,े अंतर-
आधा रत, मानक -आधा रत डिजटल स टम का व धवत लाभ उठाता है , और वा य से संबं धत
यि तगत जानकार क सरु ा, गोपनीयता और गोपनीयता सु नि चत करता है।

ोत: https://www.nhp.gov.in/national-digital-health-mission-(ndhm)_pg

99. समाचार म अ सर सन
ु ा जाने वाला श द 'डॉ सएि संग' न न ल खत म से कौन सा ग त व ध
से मेल खाती है?
(a) उपयोगकता के ाउज़र को ै क करना और पॉप-अप या बैनर व ापन को द शत करने के
इरादे से इ तहास डाउनलोड करना।
(b) दभ
ु ावनापण
ू इरादे से इंटरनेट पर दस
ू र क यि तगत जानकार का शत करना।
(c) लगभग सब कुछ ै क करना जो एक यि त अपने कं यूटर पर करता है।
(d) दभ
ु ावनापण
ू लंक या अटै चमट वाले वैध संदेश के प म छ न मेल वत रत करना।

उ र: b

या या:
वक प (a) गलत है : एडवेयर उपयोगकता के ाउज़र को ै क करने और पॉप-अप या बैनर व ापन
को द शत करने के इरादे से इ तहास डाउनलोड करने के लए संद भत करता है । वे उपयोगकताओं
को खर दार करने का लालच दे ते ह।
वक प (b) सह है: डॉि सं गंग इंटरनेट पर दस
ू र क यि तगत जानकार को दभ
ु ावनापण
ू इरादे से
का शत करने का काय है जो उ ह उ पीड़न और साइबर हमल का शकार बना सकता है । यह एक
यि त को भावना मक और मान सक प से भा वत कर सकता है। मेटा के ओवरसाइट बोड ने
फेसबक
ु को फरवर , 2022 म कड़े डॉि संग नयम जार करने का सझ
ु ाव दया।
वक प (c) गलत है: लगभग सब कुछ ै क करना जो एक यि त अपने कं यट
ू र पर करता है, एक
ऐसा काय है िजसे क लॉगस (Keyloggers) के प म जाना जाता है। इसम ईमेल, खोले गए वेबपेज,
ो ाम और क ो स शा मल ह।
वक प (d) गलत है : दभ
ु ावनापण
ू लंक या अटै चमट वाले वैध संदेश के प म छ न मेल को
फ़ शंग अटै क के प म जाना जाता है। यह उपयोगकताओं को अनसन
ु ा करने के लए मैलवेयर
न पादन यो य फ़ाइल वत रत करता है ।

ोत: https://www.bbc.com/news/technology-60302683

100. भारत-यूएई संबध


ं के संदभ म, न न ल खत कथन पर वचार कर:
1. व वष 2021-22 म, संयु त अरब अमीरात भारत का तीसरा सबसे बड़ा यापार भागीदार है।
2. यए
ू ई भारत का सबसे बड़ा नयात गंत य है ।
3. भारतीय यए
ू ई म दस
ू रा सबसे बड़ा जातीय समद
ु ाय है
ऊपर दए गए कथन म से कौन-सा/से सह है/ह?
(a) केवल 1
(b) 1 और 2 केवल
(c) केवल 3
(d) 1, 2 और 3

उ र: a

या या:

वक प (a) सह है: भारत और यए


ू ई ने स दय से यापार लंक साझा कए ह। यापार, जो
पारं प रक व तुओं जैसे क दनांक, मोती और मछ लय पर हावी था, ने संयु त अरब अमीरात म तेल
क खोज के बाद एक तेज बदलाव कया। बढ़ते भारत-यूएई आ थक और वा णि यक संबध
ं दोन दे श
के बीच तेजी से व वधता और वप ीय संबध
ं को गहराई से व वधता और गहनता क ि थरता
और ताकत म योगदान करते ह। दोन प आपसी लाभ के लए इन संबध
ं को और मजबत
ू करने
के लए यास कर रहे ह। भारत-यूएई यापार 2019-20 म UAE म 59 ब लयन अमे रक डॉलर का
मू य था , जो चीन और अमे रका के बाद वष 2019-20 के लए भारत का तीसरा सबसे बड़ा
यापा रक भागीदार था। यह वृ 2021-22 म भी जार है । इसके अलावा, यए
ू ई भारत का दस
ू रा
सबसे बड़ा नयात गंत य है (हमारे बाद)वष 2019-20 के लए लगभग यए
ू स $ 29 ब लयन क रा श
के साथ। यए
ू ई के लए, भारत वष 2019 के लए दस
ू रा सबसे बड़ा यापा रक भागीदार है , िजसक
रा श लगभग 41.43 ब लयन अमे रक डॉलर (गैर-तेल यापार) है । इसके अलावा, भारतीय भारत म
सबसे बड़े जातीय समद
ु ाय का गठन करते ह, िजसम लगभग 30% जनसं या सभी सेवाओं के सभी
े म भारतीय का गठन करती है।

ोत: https://www.indembassyuae.gov.in/bilateral-eco-com-relation.php

You might also like